PEDS NCLEX Questions

¡Supera tus tareas y exámenes ahora con Quizwiz!

A 4-year-old child is brought to the emergency department. The child has a "froglike" croaking sound on inspiration, is agitated, and is drooling. The child insists on sitting upright. The priority action by the nurse is to A. examine the child's oropharynx and report the assessment to the healthcare provider. B. make the child lie down and rest quietly. C. auscultate the child's lungs and make preparations for placement in a mist tent. D. notify the healthcare provider immediately and be prepared to assist with a tracheostomy or intubation

D

A humidified atmosphere is recommended for a young child with an upper respiratory tract infection because it A. liquefies secretions. B. improves oxygenation. C. promotes ventilation. D. soothes inflamed mucous membrane.

D

Cystic fibrosis may affect one system or multiple systems of the body. What is the primary factor responsible for possible multiple clinical manifestations? A. Atrophic changes in the mucosal wall of the intestines B. Hypoactivity of the autonomic nervous system C. Hyperactivity of the apocrine glands D. Mechanical obstruction caused by increased viscosity of exocrine gland secretions

D

Cystic fibrosis must often affects multiple systems of the body. The primary factor responsible for possible multiple clinical manifestations is which of the following? A. Hyperactivity of the sweat glands B. Hyperactivity of parasympathetic nervous system C. Sweat chloride test >60 mEq/L D. Increased viscosity of mucous gland secretions

D

The nurse is interviewing the parents of a 4-month-old infant brought to the hospital emergency department. The infant is dead on arrival, and no attempt at resuscitation is made. The parents state that the baby was found in the crib with a blanket over the head, lying face down in bloody fluid from the nose and mouth. The parents indicate no problems when the infant was placed in the crib asleep. Which of the following causes of death does the nurse suspect? A. Suffocation B. Child abuse C. Infantile apnea D. Sudden infant death syndrome (SIDS)

D

The school nurse is called to the cafeteria because a child "has eaten something he is allergic to." The child is in severe respiratory distress. The first action by the nurse is to A. determine what the child has eaten. B. administer diphenhydramine (Benadryl) PO stat. C. move the child to the nurse's office or hallway. D. have someone call for an ambulance and paramedic rescue squad or 9-1-1.

D

What clinical manifestation would the nurse expect when a pneumothorax occurs in a neonate who is undergoing mechanical ventilation? A. Barrel chest B. Wheezing C. Thermal instability D. Nasal flaring and retractions

D

Which of the following statements best represents infectious mononucleosis? A. Herpes simplex type 2 B. Leukopenia is often paired with the diagnosis C. Amoxicillin is used to treat the pharyngitis D. Physical assessment and blood tests are used as test results to establish the diagnosis

D

26. The nurse is caring for an adolescent with osteosarcoma being admitted to undergo chemotherapy. The adolescent had a right above-the-knee amputation 2 months ago and has been experiencing "phantom limb pain." Which prescribed medication is appropriate to administer to relieve phantom limb pain? a. Amitriptyline (Elavil) b. Hydrocodone (Vicodin) c. Oxycodone (OxyContin) d. Alprazolam (Xanax)

a. Amitriptyline (Elavil) ANS: A Amitriptyline (Elavil) has been used successfully to decrease phantom limb pain. Opioids such as Vicodin or OxyContin would not be prescribed for this pain. A benzodiazepine, Xanax, would not be prescribed for this type of pain. DIF: Cognitive Level: Apply REF: p. 836 TOP: Integrated Process: Nursing Process: Implementation MSC: Area of Client Needs: Physiologic Integrity: Pharmacologic and Parenteral Therapies

2. The nurse is assessing a child after a cardiac catheterization. Which complication should the nurse be assessing for? a. Cardiac arrhythmia b. Hypostatic pneumonia c. Heart failure d. Rapidly increasing blood pressure

a. Cardiac arrhythmia ANS: A Because a catheter is introduced into the heart, a risk exists of catheter-induced dysrhythmias occurring during the procedure. These are usually transient. Hypostatic pneumonia, heart failure, and rapidly increasing blood pressure are not risks usually associated with cardiac catheterization. DIF: Cognitive Level: Apply REF: p. 739 TOP: Integrated Process: Nursing Process: Assessment MSC: Area of Client Needs: Physiologic Integrity: Reduction of Risk Potential

26. The school nurse is informed that a child with human immunodeficiency virus (HIV) will be attending school soon. Which is an important nursing intervention? a. Carefully follow universal precautions. b. Determine how the child became infected. c. Inform the parents of the other children. d. Reassure other children that they will not become infected.

a. Carefully follow universal precautions. ANS: A Universal precautions are necessary to prevent further transmission of the disease. It is not the role of the nurse to determine how the child became infected. Informing the parents of other children and reassuring children that they will not become infected is a violation of the child's right to privacy. DIF: Cognitive Level: Apply REF: p. 807 TOP: Integrated Process: Nursing Process: Implementation MSC: Area of Client Needs: Physiologic Integrity: Reduction of Risk Potential

13. Why is meperidine (Demerol) not recommended for children in sickle cell crisis? a. May induce seizures b. Is easily addictive c. Not adequate for pain relief d. Given by intramuscular injection

a. May induce seizures ANS: A A metabolite of meperidine, normeperidine, is a central nervous system stimulant that produces anxiety, tremors, myoclonus, and generalized seizures when it accumulates with repetitive dosing. Patients with sickle cell disease are particularly at risk for normeperidine-induced seizures. Meperidine is no more addictive than other narcotic agents. Meperidine is adequate for pain relief. It is available for IV infusion. DIF: Cognitive Level: Understand REF: p. 795 TOP: Integrated Process: Nursing Process: Planning MSC: Area of Client Needs: Physiologic Integrity: Pharmacologic and Parenteral Therapies

24. The nurse is taking care of an adolescent with osteosarcoma. The parents ask the nurse about treatment. The nurse should make which accurate response about treatment for osteosarcoma? a. Treatment usually consists of surgery and chemotherapy. b. Amputation of the affected extremity is rarely necessary. c. Intensive irradiation is the primary treatment. d. Bone marrow transplantation offers the best chance of long-term survival.

a. Treatment usually consists of surgery and chemotherapy. ANS: A The optimal therapy for osteosarcoma is a combination of surgery and chemotherapy. Intensive irradiation and bone marrow transplantation are usually not part of the therapeutic management. DIF: Cognitive Level: Understand REF: p. 836 TOP: Integrated Process: Nursing Process: Implementation MSC: Area of Client Needs: Physiologic Integrity: Physiologic Adaptation

34. The nurse is conducting teaching for an adolescent being discharged to home after a renal transplant. The adolescent needs further teaching if which statement is made? a. "I will report any fever to my primary health care provider." b. "I am glad I only have to take the immunosuppressant medication for two weeks." c. "I will observe my incision for any redness or swelling." d. "I won't miss doing kidney dialysis every week."

b. "I am glad I only have to take the immunosuppressant medication for two weeks." ANS: B The immunosuppressant medications are taken indefinitely after a renal transplant, so they should not be discontinued after two weeks. Reporting a fever and observing an incision for redness and swelling are accurate statements. The adolescent is correct in indicating dialysis will not need to be done after the transplant. PTS: 1 DIF: Cognitive Level: Apply REF: 925 TOP: Integrated Process: Teaching/Learning MSC: Area of Client Needs: Physiologic Integrity: Reduction of Risk Potentia

49. Parents of a child undergoing an endoscopy to rule out peptic ulcer disease (PUD) from H. pylori ask the nurse, "If H. pylori is found, will my child need another endoscopy to know that it is gone?" Which is the nurse's best response? a. "Yes, the only way to know the H. pylori has been eradicated is with another endoscopy." b. "We can collect a stool sample and confirm that the H. pylori has been eradicated." c. "A blood test can be done to determine that the H. pylori is no longer present." d. "Your child will always test positive for H. pylori because after treatment it goes into remission but can't be completely eradicated."

b. "We can collect a stool sample and confirm that the H. pylori has been eradicated." ANS: B An upper endoscopy is the procedure initially performed to diagnose PUD. A biopsy can determine the presence of H. pylori. Polyclonal and monoclonal stool antigen tests are an accurate, noninvasive method to confirm H. pylori has been eradicated after treatment. A blood test can identify the presence of the antigen to this organism, but because H. pylori was already present, it would not be as accurate as a stool sample to determine whether it has been eradicated. H. pylori can be treated and, once the treatment is complete, the stool sample can determine that it was eradicated. DIF: Cognitive Level: Apply REF: p. 716 TOP: Integrated Process: Teaching/Learning MSC: Area of Client Needs: Physiologic Integrity: Reduction of Risk Potential

4. The nurse is preparing to care for an infant returning from pyloromyotomy surgery. Which prescribed orders should the nurse anticipate implementing? (Select all that apply.) a. NPO for 24 hours b. Administration of analgesics for pain c. Ice bag to the incisional area d. IV fluids continued until tolerating PO e. Clear liquids as the first feeding

b. Administration of analgesics for pain d. IV fluids continued until tolerating PO e. Clear liquids as the first feeding ANS: B, D, E Feedings are usually instituted soon after a pyloromyotomy surgery, beginning with clear liquids and advancing to formula or breast milk as tolerated. IV fluids are administered until the infant is taking and retaining adequate amounts by mouth. Appropriate analgesics should be given around the clock because pain is continuous. Ice should not be applied to the incisional area as it vasoconstricts and would reduce circulation to the incisional area and impair healing. DIF: Cognitive Level: Apply REF: p. 728 TOP: Integrated Process: Nursing Process: Planning MSC: Area of Client Needs: Physiologic Integrity: Reduction of Risk Potential

13. A mother calls the clinic nurse about her 4-year-old son who has acute diarrhea. She has been giving him the antidiarrheal drug loperamide (Imodium A-D). The nurse's response should be based on what knowledge about this drug? a. Not indicated b. Indicated because it slows intestinal motility c. Indicated because it decreases diarrhea d. Indicated because it decreases fluid and electrolyte losses

a. Not indicated ANS: A Antidiarrheal medications are not recommended for the treatment of acute infectious diarrhea. These medications have adverse effects and toxicity, such as worsening of the diarrhea because of slowing of motility and ileus, or a decrease in diarrhea with continuing fluid losses and dehydration. Antidiarrheal medications are not recommended in infants and small children. DIF: Cognitive Level: Analyze REF: p. 701 TOP: Integrated Process: Teaching/Learning MSC: Area of Client Needs: Physiologic Integrity: Pharmacologic and Parenteral Therapies

45. The nurse is caring for a boy with probable intussusception. He had diarrhea before admission but, while waiting for administration of air pressure to reduce the intussusception, he passes a normal brown stool. Which nursing action is the most appropriate? a. Notify practitioner b. Measure abdominal girth c. Auscultate for bowel sounds d. Take vital signs, including blood pressure

a. Notify practitioner ANS: A Passage of a normal brown stool indicates that the intussusception has reduced itself. This is immediately reported to the practitioner, who may choose to alter the diagnostic-therapeutic care plan. The first action would be to report the normal stool to the practitioner. DIF: Cognitive Level: Apply REF: p. 728 TOP: Integrated Process: Nursing Process: Implementation MSC: Area of Client Needs: Physiologic Integrity: Reduction of Risk Potential

22. The nurse is conducting an admission assessment on a school-age child with acute renal failure. Which are the primary clinical manifestations the nurse expects to find with this condition? a. Oliguria and hypertension b. Hematuria and pallor c. Proteinuria and muscle cramps d. Bacteriuria and facial edema

a. Oliguria and hypertension ANS: A The principal feature of acute renal failure is oliguria; hypertension is a nonspecific clinical manifestation. Hematuria and pallor, proteinuria and muscle cramps, and bacteriuria and facial edema are not principal features of acute renal failure. PTS: 1 DIF: Cognitive Level: Understand REF: 919 TOP: Integrated Process: Nursing Process: Assessment MSC: Area of Client Needs: Physiologic Integrity: Physiologic Adaptation

1. The nurse is conducting a staff in-service on renal ultrasounds. Which statement describes this diagnostic test? a. Computed tomography uses external radiation to visualize the renal system. b. Visualization of the renal system is accomplished without exposure to radiation or radioactive isotopes. c. Contrast medium and x-rays allow for visualization of the renal system. d. External radiation for x-ray films is used to visualize the renal system, before, during, and after voiding.

a. Computed tomography uses external radiation to visualize the renal system.

9. Which is instituted for the therapeutic management of minimal change nephrotic syndrome? a. Corticosteroids b. Antihypertensive agents c. Long-term diuretics d. Increased fluids to promote diuresis

a. Corticosteroids ANS: A Corticosteroids are the first line of therapy for minimal change nephrotic syndrome. Response is usually seen within 7 to 21 days. Antihypertensive agents and long-term diuretic therapy are usually not necessary. A diet that has fluid and salt restrictions may be indicated. PTS: 1 DIF: Cognitive Level: Apply REF: 914 TOP: Integrated Process: Nursing Process: Implementation MSC: Area of Client Needs: Physiologic Integrity: Pharmacologic and Parenteral Therapy

38. The nurse is caring for a neonate with a suspected tracheoesophageal fistula. What nursing care should be included? a. Elevate the head but give nothing by mouth. b. Elevate the head for feedings. c. Feed glucose water only. d. Avoid suctioning unless infant is cyanotic.

a. Elevate the head but give nothing by mouth.

17. The nurse is admitting a child with a Wilms tumor. Which is the initial assessment finding associated with this tumor? a. Abdominal swelling b. Weight gain c. Hypotension d. Increased urinary output

a. Abdominal swelling ANS: A The initial assessment finding with a Wilms (kidney) tumor is abdominal swelling. Weight loss, not weight gain, may be a finding. Hypertension occasionally occurs with a Wilms tumor. Urinary output is not increased, but hematuria may be noted. DIF: Cognitive Level: Understand REF: p. 820 TOP: Integrated Process: Nursing Process: Assessment MSC: Area of Client Needs: Physiologic Integrity: Physiologic Adaptation

33. Which of the following types of seizures may be difficult to detect? a. Absence b. Generalized c. Simple partial d. Complex partial

a. Absence ANS: A Absence seizures may go unrecognized because little change occurs in the child's behavior during the seizure. Generalized, simple partial, and complex partial seizures all have clinical manifestations that are observable. DIF: Cognitive Level: Understand REF: p. 896 TOP: Integrated Process: Nursing Process: Assessment MSC: Area of Client Needs: Physiologic Integrity: Physiologic Adaptation

1. Which condition in a child should alert a nurse for increased fluid requirements? a. Fever b. Mechanical ventilation c. Congestive heart failure d. Increased intracranial pressure (ICP)

a. Fever ANS: A Fever leads to great insensible fluid loss in young children because of increased body surface area relative to fluid volume. Respiratory rate influences insensible fluid loss and should be monitored in the mechanically ventilated child. Congestive heart failure is a case of fluid overload in children. Increased ICP does not lead to increased fluid requirements in children. DIF: Cognitive Level: Understand REF: p. 689 TOP: Integrated Process: Nursing Process: Assessment MSC: Area of Client Needs: Physiologic Integrity: Reduction of Risk Potential

3. The nurse is conducting an assessment on a school-age child with urosepsis. Which assessment finding should the nurse expect? a. Fever with a positive blood culture b. Proteinuria and edema c. Oliguria and hypertension d. Anemia and thrombocytopenia

a. Fever with a positive blood culture ANS: A Symptoms of urosepsis include a febrile UTI coexisting with systemic signs of bacterial illness; blood culture reveals presence of urinary pathogen. Proteinuria and edema are symptoms of minimal change nephrotic syndrome (MCNS). Oliguria and hypertension are symptoms of acute glomerulonephritis (AGN). Anemia and thrombocytopenia are symptoms of hemolytic uremic syndrome (HUS). PTS: 1 DIF: Cognitive Level: Analyze REF: 846 TOP: Integrated Process: Nursing Process: Assessment MSC: Area of Client Needs: Physiologic Integrity: Physiologic Adaptation

11. The nurse closely monitors the temperature of a child with minimal change nephrotic syndrome. The purpose of this assessment is to detect an early sign of which possible complication? a. Infection b. Hypertension c. Encephalopathy d. Edema

a. Infection ANS: A Infection is a constant source of danger to edematous children and those receiving corticosteroid therapy. An increased temperature could be an indication of an infection. Temperature is not an indication of hypertension or edema. Encephalopathy is not a complication usually associated with minimal change nephrotic syndrome. The child will most likely have neurologic signs and symptoms.

12. A young child is brought to the emergency department with severe dehydration secondary to acute diarrhea and vomiting. What should therapeutic management of this child begin with? a. Intravenous (IV) fluids b. ORS c. Clear liquids, 1 to 2 ounces at a time d. Administration of antidiarrheal medication

a. Intravenous (IV) fluids ANS: A In children with severe dehydration, IV fluids are initiated. ORS is acceptable therapy if the dehydration is not severe. Diarrhea is not managed by using clear liquids by mouth. These fluids have a high carbohydrate content, low electrolyte content, and high osmolality. Antidiarrheal medications are not recommended for the treatment of acute infectious diarrhea. DIF: Cognitive Level: Apply REF: p. 706 TOP: Integrated Process: Nursing Process: Implementation MSC: Area of Client Needs: Physiologic Integrity: Physiologic Adaptation

3. Which type of dehydration is defined as "dehydration that occurs in conditions in which electrolyte and water deficits are present in approximately balanced proportion"? a. Isotonic dehydration b. Hypotonic dehydration c. Hypertonic dehydration d. All types of dehydration in infants and small children

a. Isotonic dehydration ANS: A Isotonic dehydration is the correct term for this definition and is the most frequent form of dehydration in children. Hypotonic dehydration occurs when the electrolyte deficit exceeds the water deficit, leaving the serum hypotonic. Hypertonic dehydration results from water loss in excess of electrolyte loss and is usually caused by a proportionately larger loss of water or a larger intake of electrolytes. This definition is specific to isotonic dehydration. DIF: Cognitive Level: Understand REF: p. 694 TOP: Integrated Process: Nursing Process: Assessment MSC: Area of Client Needs: Physiologic Integrity: Reduction of Risk Potential

34. The nurse is admitting a child with rheumatic fever. Which therapeutic management should the nurse expect to implement? a. Administering penicillin b. Avoiding salicylates (aspirin) c. Imposing strict bed rest for 4 to 6 weeks d. Administering corticosteroids if chorea develops

a. Administering penicillin ANS: A The goal of medical management is the eradication of the hemolytic streptococci. Penicillin is the drug of choice. Salicylates can be used to control the inflammatory process, especially in the joints, and reduce the fever and discomfort. Bed rest is recommended for the acute febrile stage, but it does not need to be strict. The chorea is transient and will resolve without treatment. DIF: Cognitive Level: Apply REF: p. 768 TOP: Integrated Process: Nursing Process: Implementation MSC: Area of Client Needs: Physiologic Integrity: Pharmacologic and Parenteral Therapies

24. The nurse is caring for a child admitted with acute abdominal pain and possible appendicitis. Which is appropriate to relieve the abdominal discomfort? a. Place in Trendelenburg position. b. Allow to assume position of comfort. c. Apply moist heat to the abdomen. d. Administer a saline enema to cleanse bowel.

b. Allow to assume position of comfort. ANS: B The child should be allowed to take a position of comfort, usually with the legs flexed. The Trendelenburg position will not help with the discomfort. In any instance in which appendicitis is a possibility, there is a danger in administering a laxative or enemas or applying heat to the area. Such measures stimulate bowel motility and increase the risk of perforation. DIF: Cognitive Level: Apply REF: p. 709 TOP: Integrated Process: Nursing Process: Planning MSC: Area of Client Needs: Physiologic Integrity: Physiologic Adaptation

6. Parents call the clinic and report that their toddler has had acute diarrhea for 24 hours. The nurse should further ask the parents if the toddler has which associated factor that is causing the acute diarrhea? a. Celiac disease b. Antibiotic therapy c. Immunodeficiency d. Protein malnutrition

b. Antibiotic therapy ANS: B Acute diarrhea is a sudden increase in frequency and change in consistency of stools and may be associated with antibiotic therapy. Celiac disease is a problem with gluten intolerance and may cause chronic diarrhea if not identified and managed appropriately. Immunodeficiency would occur with chronic diarrhea. Protein malnutrition or kwashiorkor causes chronic diarrhea from lowered resistance to infection. DIF: Cognitive Level: Apply REF: p. 691 TOP: Integrated Process: Nursing Process: Assessment MSC: Area of Client Needs: Physiologic Integrity: Physiologic Adaptation

35. The nurse is teaching parents of a child with chronic renal failure (CRF) about the use of recombinant human erythropoietin (rHuEPO) subcutaneous injections. Which statement indicates the parents have understood the teaching? a. "These injections will help with the hypertension." b. "We're glad the injections only need to be given once a month." c. "The red blood cell count should begin to improve with these injections." d. "Urine output should begin to improve with these injections."

c. "The red blood cell count should begin to improve with these injections." ANS: C Anemia in children with CRF is related to decreased production of erythropoietin. Recombinant human erythropoietin (rHuEPO) is being offered to these children as thrice-weekly or weekly subcutaneous injections and is replacing the need for frequent blood transfusions. The parents understand the teaching if they say that the red blood cell count will begin to improve with these injections. PTS: 1 DIF: Cognitive Level: Apply REF: 916 | 923 TOP: Integrated Process: Teaching/Learning MSC: Area of Client Needs: Physiologic Integrity: Reduction of Risk Potential

18. The nurse is teaching the parent about the diet of a child experiencing severe edema associated with acute glomerulonephritis. Which information should the nurse include in the teaching? a. "You will need to decrease the number of calories in your child's diet." b. "Your child's diet will need an increased amount of protein." c. "You will need to avoid adding salt to your child's food." d. "Your child's diet will consist of low-fat, low-carbohydrate foods."

c. "You will need to avoid adding salt to your child's food." ANS: C For most children, a regular diet is allowed, but it should contain no added salt. The child should be offered a regular diet with favorite foods. Severe sodium restrictions are not indicated. PTS: 1 DIF: Cognitive Level: Apply REF: 860 TOP: Integrated Process: Teaching/Learning MSC: Area of Client Needs: Physiologic Integrity: Basic Care and Comfort

14. Constipation has recently become a problem for a school-age girl. She is healthy except for seasonal allergies that are being treated with antihistamines. What should the nurse suspect caused the constipation? a. Diet b. Allergies c. Antihistamines d. Emotional factors

c. Antihistamines DIF: Cognitive Level: Analyze REF: p. 702 TOP: Integrated Process: Nursing Process: Evaluation MSC: Area of Client Needs: Physiologic Integrity: Pharmacologic and Parenteral Therapies

21. A histamine-receptor antagonist such as cimetidine (Tagamet) or ranitidine (Zantac) is ordered for an infant with GER. What is the purpose of this medication? a. Prevent reflux b. Prevent hematemesis c. Reduce gastric acid production d. Increase gastric acid production

c. Reduce gastric acid production ANS: C The mechanism of action of histamine-receptor antagonists is to reduce the amount of acid present in gastric contents and perhaps prevent esophagitis. Preventing reflux or hematemesis and increasing gastric acid production are not the modes of action of histamine-receptor antagonists. DIF: Cognitive Level: Understand REF: p. 707 TOP: Integrated Process: Nursing Process: Evaluation MSC: Area of Client Needs: Physiologic Integrity: Pharmacologic and Parenteral Therapies

What nursing intervention would be most appropriate for a 10-year-old child with type I diabetes in order to meet their needs (as described by Erikson). a. Explain carefully to the mother the need to rigidly adhere to dietary modifications. b. Allow the child to eat whatever he or she wants and administer insulin to maintain optimum glucose levels. c. Allow the child to perform his or her own Accuchecks and administration of insulin. d. Perform Accuchecks four times a day and at bedtime.

c. Allow the child to perform his or her own Accuchecks and administration of insulin.

34. What should be included in caring for the newborn with a cleft lip and palate before surgical repair? a. Gastrostomy feedings b. Keeping infant in near-horizontal position during feedings c. Allowing little or no sucking d. Providing satisfaction of sucking needs

d. Providing satisfaction of sucking needs ANS: D Using special or modified nipples for feeding techniques helps meet the infant's sucking needs. Gastrostomy feedings are usually not indicated. Feeding is best accomplished with the infant's head in an upright position. The child requires both nutritive and nonnutritive sucking. DIF: Cognitive Level: Apply REF: p. 723 TOP: Integrated Process: Nursing Process: Implementation MSC: Area of Client Needs: Physiologic Integrity: Reduction of Risk Potential

21. Which is the most common cause of acute renal failure in children? a. Pyelonephritis b. Tubular destruction c. Urinary tract obstruction d. Severe dehydration

d. Severe dehydration

39. Which type of hernia has an impaired blood supply to the herniated organ? a. Hiatal hernia b. Incarcerated hernia c. Omphalocele d. Strangulated hernia

d. Strangulated hernia ANS: D A strangulated hernia is one in which the blood supply to the herniated organ is impaired. Hiatal hernia is the intrusion of an abdominal structure, usually the stomach, through the esophageal hiatus. Incarcerated hernia is a hernia that cannot be reduced easily. Omphalocele is the protrusion of intra-abdominal viscera into the base of the umbilical cord. The sac is covered with peritoneum, not skin. DIF: Cognitive Level: Understand REF: p. 728 TOP: Integrated Process: Nursing Process: Assessment MSC: Area of Client Needs: Physiologic Integrity: Reduction of Risk Potential

9. The nurse is talking to a parent with a child who has a latex allergy. Which statement by the parent would indicate a correct understanding of the teaching? a. "My child will have an allergic reaction if he comes in contact with yeast products." b. "My child may have an upset stomach if he eats a food made with wheat or barley." c. "My child will probably develop an allergy to peanuts." d. "My child should not eat bananas or kiwis."

d. "My child should not eat bananas or kiwis." ANS: D There are cross-reactions between latex allergies and a number of foods such as bananas, avocados, kiwi, and chestnuts. Children with a latex allergy will not develop allergies to other food products such as yeast, wheat, barley, or peanuts. DIF: Cognitive Level: Analyze REF: p. 990 TOP: Integrated Process: Nursing Process: Evaluation MSC: Area of Client Needs: Physiologic Integrity: Reduction of Risk Potential

A 16 year old female comes for a sport's physical in the clinic. During the nursing history, the teenage states she is bothered that she "towers over her companions and everyone is staring at her at school." What would be the most therapeutic response by the nurse? a. "Just ignore the other kids at school." b. "You are so lucky to be tall. You can play basketball or whatever you want." c. "This will resolve itself in time." d. "Tell me more about how this embarasses you."

d. "Tell me more about how this embarasses you."

1. Which child should the nurse document as being anemic? a. 7-year-old child with a hemoglobin of 11.5 g/dl b. 3-year-old child with a hemoglobin of 12 g/dl c. 14-year-old child with a hemoglobin of 10 g/dl d. 1-year-old child with a hemoglobin of 13 g/dl

d. 1-year-old child with a hemoglobin of 13 g/dl ANS: D Anemia is a condition in which the number of red blood cells, or hemoglobin concentration, is reduced below the normal values for age. Anemia is defined as a hemoglobin level below 10 or 11 g/dl. The child with a hemoglobin of 10 g/dl would be considered anemic. The normal hemoglobin for a child after 2 years of age is 11.5 to 15.5 g/dl. DIF: Cognitive Level: Understand REF: p. 789 TOP: Integrated Process: Nursing Process: Assessment MSC: Area of Client Needs: Physiologic Integrity: Physiologic Adaptation

3. The nurse is performing a Glasgow Coma Scale on a school-age child with a head injury. The child opens eyes spontaneously, obeys commands, and is oriented to person, time, and place. Which is the score the nurse should record? a. 8 b. 11 c. 13 d. 15

d. 15 ANS: D The Glasgow Coma Scale (GCS) consists of a three-part assessment: eye opening, verbal response, and motor response. Numeric values of 1 through 5 are assigned to the levels of response in each category. The sum of these numeric values provides an objective measure of the patient's level of consciousness (LOC). A person with an unaltered LOC would score the highest, 15. The child who opens eyes spontaneously, obeys commands, and is oriented is scored at a 15. DIF: Cognitive Level: Understand REF: p. 873 TOP: Integrated Process: Nursing Process: Assessment

21. Hyperglycemia associated with diabetic ketoacidosis is defined as a blood glucose measurement equal to or greater than _____ mg/dl. a. 100 b. 120 c. 180 d. 200

d. 200 ANS: D Diabetic ketoacidosis is a state of relative insulin insufficiency and may include the presence of hyperglycemia, a blood glucose level greater than or equal to 200 mg/dl. The values 100 mg/dl, 120 mg/dl, and 180 mg/dl are too low for the definition of ketoacidosis. DIF: Cognitive Level: Understand REF: p. 940 TOP: Integrated Process: Nursing Process: Assessment MSC: Area of Client Needs: Physiologic Integrity: Physiologic Adaptation

29. The nurse is discussing with a child and family the various sites used for insulin injections. Which site usually has the fastest rate of absorption? a. Arm b. Leg c. Buttock d. Abdomen

d. Abdomen ANS: D The abdomen has the fastest rate of absorption but the shortest duration. The arm has a fast rate of absorption but short duration. The leg has a slow rate of absorption but a long duration. The buttock has the slowest rate of absorption and the longest duration. DIF: Cognitive Level: Apply REF: p. 931 TOP: Integrated Process: Nursing Process: Implementation MSC: Area of Client Needs: Physiologic Integrity: Pharmacologic and Parenteral Therapies

16. Which therapeutic management treatment is implemented for children with Hirschsprung disease? a. Daily enemas b. Low-fiber diet c. Permanent colostomy d. Surgical removal of affected section of bowel

d. Surgical removal of affected section of bowel ANS: D Most children with Hirschsprung disease require surgical rather than medical management. Surgery is done to remove the aganglionic portion of the bowel, relieve obstruction, and restore normal bowel motility and function of the internal anal sphincter. Preoperative management may include enemas and low-fiber, high-calorie, high-protein diet, until the child is physically ready for surgery. The colostomy that is created in Hirschsprung disease is usually temporary. DIF: Cognitive Level: Understand REF: p. 703 TOP: Integrated Process: Nursing Process: Implementation MSC: Area of Client Needs: Physiologic Integrity: Physiologic Adaptation

6. The nurse is admitting a newborn with hypospadias to the nursery. The nurse expects which finding in this newborn? a. Absence of a urethral opening is noted. b. Penis appears shorter than usual for age. c. The urethral opening is along the dorsal surface of the penis. d. The urethral opening is along the ventral surface of the penis.

d. The urethral opening is along the ventral surface of the penis. ANS: D Hypospadias is a congenital condition in which the urethral opening is located anywhere along the ventral surface of the penis. The urethral opening is present, but not at the glans. Hypospadias refers to the urethral opening, not to the size of the penis. Urethral opening along ventral surface of penis is known as epispadias.

26. Which clinical manifestation would be seen in a child with chronic renal failure? a. Hypotension b. Massive hematuria c. Hypokalemia d. Unpleasant "uremic" breath odor

d. Unpleasant "uremic" breath odor ANS: D Children with chronic renal failure have a characteristic breath odor resulting from the retention of waste products. Hypertension may be a complication of chronic renal failure. With chronic renal failure, little or no urinary output occurs. Hyperkalemia is a concern in chronic renal failure. PTS: 1 DIF: Cognitive Level: Understand REF: 922 TOP: Integrated Process: Nursing Process: Assessment MSC: Area of Client Needs: Physiologic Integrity: Physiologic Adaptation

33. A preschool child is being admitted to the hospital with dehydration and a urinary tract infection (UTI). Which urinalysis result should the nurse expect with these conditions? a. WBC <1; specific gravity 1.008 b. WBC <2; specific gravity 1.025 c. WBC >2; specific gravity 1.016 d. WBC >2; specific gravity 1.030

d. WBC >2; specific gravity 1.030 ANS: D WBC count in a routine urinalysis should be <1 or 2. Over that amount indicates a urinary tract inflammatory process. The urinalysis specific gravity for children with normal fluid intake is 1.016 to 1.022. When the specific gravity is high, dehydration is indicated. A low specific gravity is seen with excessive fluid intake, distal tubular dysfunction, or insufficient antidiuretic hormone secretion. PTS: 1 DIF: Cognitive Level: Analyze REF: 907 TOP: Integrated Process: Nursing Process: Assessment MSC: Area of Client Needs: Physiologic Integrity: Physiologic Adaptation

30. The nurse is caring for an adolescent who has just started dialysis. The child seems always angry, hostile, or depressed. The nurse should recognize that this is most likely related to: a. neurologic manifestations that occur with dialysis. b. physiologic manifestations of renal disease. c. adolescents having few coping mechanisms. d. adolescents often resenting the control and enforced dependence imposed by dialysis.

d. adolescents often resenting the control and enforced dependence imposed by dialysis. ANS: D Older children and adolescents need control. The necessity of dialysis forces the adolescent into a dependent relationship, which results in these behaviors. These are a function of the child's age, not neurologic or physiologic manifestations of the dialysis. Feelings of anger, hostility, and depression are functions of the child's age, not neurologic or physiologic manifestations of the dialysis. Adolescents do have coping mechanisms, but they need to have some control over their disease management. PTS: 1 DIF: Cognitive Level: Analyze REF: 923 TOP: Integrated Process: Nursing Process: Evaluation MSC: Area of Client Needs: Health Promotion and Maintenance

16. A mother asks the nurse what would be the first indication that acute glomerulonephritis is improving. The nurse's best response should be that the: a. blood pressure will stabilize. b. the child will have more energy. c. urine will be free of protein. d. urinary output will increase.

d. urinary output will increase. ANS: D An increase in urinary output may signal resolution of the acute glomerulonephritis. If blood pressure is elevated, stabilization usually occurs with the improvement in renal function. The child having more energy and the urine being free of protein are related to the improvement in urinary output. PTS: 1 DIF: Cognitive Level: Apply REF: 860 TOP: Integrated Process: Teaching/Learning MSC: Area of Client Needs: Physiologic Integrity: Physiologic Adaptation

30. The nurse is reviewing first aid with a group of school nurses. Which statement made by a participant indicates a correct understanding of the information? a. "If a child loses a tooth due to injury, I should place the tooth in warm milk." b. "If a child has recurrent abdominal pain, I should send him or her back to class until the end of the day." c. "If a child has a chemical burn to the eye, I should irrigate the eye with normal saline." d. "If a child has a nosebleed, I should have the child sit up and lean forward."

d. "If a child has a nosebleed, I should have the child sit up and lean forward." ANS: D If a child has a nosebleed, the child should lean forward, not lie down. A tooth should be placed in cold milk or saliva for transporting to a dentist. Recurrent abdominal pain is a physiologic problem and requires further evaluation. If a chemical burn occurs in the eye, the eye should be irrigated with water for 20 minutes. DIF: Cognitive Level: Apply REF: p. 805 TOP: Integrated Process: Teaching/Learning MSC: Area of Client Needs: Physiologic Integrity: Reduction of Risk Potential

58. What would be the priority intervention when a 10-year-old girl comes to the nurse's office because of a headache, and the nurse notices various stages of bruising on her inner upper arms? 1. Call her mother and ask her if acetaminophen can be given for the headache. 2. Call mom and ask about the bruises. Ask the girl what happened to her arms, and have her describe the headache. 3. Inquire about her headache and the bruising on her arms; file mandatory reporting forms. 4. Call her mother to pick her up from school, and complete required school nurse visit forms.

2

6. Which of the following would be the priority intervention the nurse should initiate for a child suspected of having varicella (chickenpox)? 1. Contact precautions. 2. Contact and droplet respiratory precautions. 3. Droplet respiratory precautions. 4. Universal precautions and standard precautions.

2

34. The nurse knows that young infants are at risk for injury from SBS because: 1. Anterior fontanel is open. 2. Insufficient musculoskeletal support and a disproportionate head-to-body ratio. 3. Immature vascular system with veins and arteries that are more superficial. 4. Immature myelination of the nervous system.

2. Insufficient musculoskeletal support and a disproportionate head places the infant at risk because the head cannot be supported during a shaking episode.

A child with asthma is having pulmonary function tests. What explains the purpose of the peak expiratory flow rate (PEFR)? A. Confirms the diagnosis of asthma B. Determines the cause of asthma C. Identifies the "triggers" of asthma D. Assesses the severity of asthma

D

8. Which of the following are specific toxicities to gentamycin? 1. Hepatatoxicity. 2. Ototoxcity. 3. Anaphylaxis. 4. Neurological.

Nephrotoxicity and ototoxicity are the most significant adverse effects.

Which nursing intervention best supports the concept of atraumatic care for a hospitalized child? a. Allowing parents to visit once every shift. b. Encouraging six year old to be brave during an IV start. c. Allowing adolescent to keep the hospital door shut. d. Asking parents of baby to wait outside treatment room door during spinal tap.

c. Allowing adolescent to keep the hospital door shut.

25. What would be the nurse's best response if the foster mother of a 15-month-old with an unknown immunization history comes to the clinic requesting immunizations? 1. "Your foster child will not receive any immunizations today." 2. "Your foster child will receive the measles, mumps, rubella (MMR); and Hib; IPV; and hepatitis B vaccines." 3. "Your foster child could have harmful effects if we revaccinate with prior vaccines." 4. "Your foster child will receive only the Hib and DTaP vaccines today."

4

40. The best nursing intervention for a child with phantom pain after an amputation would be which of the following? 1. Tell the child that the pain does not exist. 2. Request a PCA pump from the physician for pain management. 3. Encourage the child to rub the stump. 4. Provide Elavil to help with pain.

4. Elavil is a medication for nerve pain that is helpful with relieving phantom pain.

32. When caring for a patient in pain, which activity is appropriate to delegate to unlicensed nursing personnel? 1. Coaching the patient during painful procedures. 2. Assessment using a self-report pain scale. 3. Evaluating pain after giving medication. 4. Bathing the patient and hygiene measures.

4. The nursing assistant may help the patient in hygiene matters, including bathing.

44. The nurse is caring for a child with CP. The nurse knows that since the 1960s the incidence of CP has: 1. Increased. 2. Decreased. 3. Remained the same. 4. Has decreased due to early misdiagnosis.

44. 1. The incidence of CP has increased partly due to the increased survival of extreme low-birth-weight and premature infants.

20. What is the most common cause of acute renal failure in children? a. Pyelonephritis b. Tubular destruction c. Urinary tract obstruction d. Inadequate perfusion

D. Inadequate perfusion the most common cause of acute renal failure in children is poor perfusion that may respond to restoration of fluid volume. Pylonephritis and tubular destruction are not common causes of acute renal failure. Obstructive uropathy may cause renal failure but it is not the most common case.

54. The mother of a 6-month-old states that since yesterday, the child cries when anyone touches her arm. Which would be the priority assessment after the airway, breathing, and circulation had been assessed and found stable? 1. Ask the mother if she knows what happened. 2. Assess child for other signs of potential physical abuse. 3. Prepare for radiological diagnostic studies. 4. Establish intravenous access, and draw blood for diagnostic testing.

1

With which age group should the nurse use "magical thinking" as a developmental strategy when administering medications? A. Infant B. Toddler C. Preschool D. School-age

C. Preschool

38. Which would be the most appropriate teaching to the parents of a female child in the emergency department who is awake, alert, and has no respiratory distress after a near-drowning experience? 1. "She will most likely be discharged, and you should watch for any cough or trouble breathing." 2. "She will need to have a preventive tube for breathing and ventilation to ensure her lungs are clear." 3. "She will be fine but sometimes antibiotics are started as a preventive." 4. "She will most likely be admitted for at least 24 hours and observed for respiratory distress or any swelling of the brain."

4

44. Which nursing intervention would be of highest priority for a 2-year-old suspected of ingestion of his grandmother's digoxin? 1. Provide supplemental oxygen. 2. Establish intravenous access. 3. Draw blood for a STAT digoxin level. 4. Provide continuous cardiac monitoring.

4

A 4-year-old child diagnosed with leukemia is hospitalized for chemotherapy. The child is fearful of the hospitalization. Which nursing intervention should be implemented to alleviate the child's fears? A. Encourage the child's parents to stay with the child. B. Encourage play with other children of the same age. C. Advise the family to visit only during the scheduled visiting hours. D. Provide a private room, allowing the child to bring favorite toys from home.

A. Encourage the child's parents to stay with the child.

5. The nurse is caring for a 6-month-old infant with a diagnosis of hydrocephalus. Which of the following signs best indicates increased ICP in this child? 1. Sunken anterior fontanel. 2. Complaints of blurred vision. 3. High-pitched cry. 4. Increased appetite. 5. Sleeping more than usual

3,5

1. A 6-month-old male is at his well-child checkup. The nurse weighs him, and his mother asks if his weight is normal for his age. The nurse's best response is: 1. "At 6 months his weight should be approximately three times his birth weight." 2. "Each child gains weight at his or her own pace." 3. "At 6 months his weight should be approximately twice his birth weight." 4. "At 6 months a child should weigh about 10 lb more than his or her birth weight."

3. "At 6 months his weight should be approximately twice his birth weight."

Apnea of infancy has been diagnosed in an infant scheduled for discharge with home monitoring. Part of the infant's discharge teaching plan should include A. Cardiopulmonary resuscitation (CPR) B. Administration of intravenous (IV) fluids C. Foreign airway obstruction removal using the Heimlich maneuver D. Advice that the infant not be left with caretakers other than the parents

A

The most appropriate nursing intervention for a child following a tonsillectomy is to A. watch for continuous swallowing. B. encourage gargling to reduce discomfort. C. position the child on the back for sleeping. D. apply warm compresses to the throat.

A

Asthma is classified into four categories: mild intermittent, mild persistent, moderate persistent, and severe persistent. Clinical features used to determine these categories include (Select all that apply) A. lung function. B. associated allergies. C. frequency of symptoms. D. frequency and severity of exacerbations.

A, D

Ringworm, frequently found in schoolchildren, is caused by which of the following? a. Virus b. Fungus c. Allergic reaction d. Bacterial infection

ANS: B Ringworm is caused by a group of closely related filamentous fungi that invade primarily the stratum corneum, hair, and nails. They are superficial infections that live on, not in, the skin.

Because the absorption of fat-soluble vitamins is decreased in cystic fibrosis, which vitamin supplementation is necessary? A. C, D B. A, E, K C. A, D, E, K D. C, folic acid

C

The nurse is assessing a child with croup and a sore throat in the ED. The child is drooling and agitated. The nurse knows that examining the child's throat using a tongue depressor might precipitate which of the following? A. Profuse coughing B. Inspiratory stridor C. Complete obstruction D. Increased agitation

C

29. Which is the most common causative agent of bacterial endocarditis? a. Staphylococcus albus b. Streptococcus hemolyticus c. Staphylococcus albicans d. Streptococcus viridans

d. Streptococcus viridans ANS: D S. viridans is the most common causative agent in bacterial (infective) endocarditis. Staphylococcus albus, Streptococcus hemolyticus, and Staphylococcus albicans are not common causative agents. DIF: Cognitive Level: Remember REF: p. 765 TOP: Integrated Process: Nursing Process: Assessment MSC: Area of Client Needs: Physiologic Integrity: Physiologic Adaptation

9. The parents of a 12-month-old male with HIV are concerned about his receiving routine immunizations. What will the nurse tell them about immunizations? 1. "Your child will not receive routine immunizations today." 2. "Your child will receive the recommended vaccines today; regular immunizations help prevent childhood illnesses." 3. "Your child is not severely immunocompromised, but I would still be concerned about his receiving them. Let's not give them today." 4. "Your child may develop infections if he gets his routine immunizations. Your child will not be immunized today."

1

11. Expected nursing assessments of a newborn with suspected cystic fibrosis would include: 1. Observe frequency and nature of stools. 2. Provide chest physical therapy. 3. Observe for weight gain. 4. Assess parent's compliance with fluid restrictions.

1,3

60. Which statement is true of shaken baby syndrome? SATA 1. There may be absence of external signs of injury. 2. Shaken babies usually do not have retinal hemorrhage. 3. Shaken babies usually do not have signs of a subdural hematoma. 4. Shaken babies have signs of external head injury.

1,3,4

10. Which of the following factors are associated with SCFE? Select all that apply. 1. Obesity. 2. Female gender. 3. African descent. 4. Age of 5 to 10 years. 5. Pubertal hormonal changes. 6. Endocrine disorders.

10. 1. Obesity increases the risk of SCFE by stressing the physeal plate. 5. SCFE is most common during pubertal hormone changes. 6. SCFE is associated with endocrine disorders.

14. An experienced nurse notes that a patient is developing a rash shortly after the first dose of an intravenous antibiotic. Which team member should be called first? 1. Physician. 2. Pharmacist. 3. Charge nurse. 4. Unit manager.

14. 1. Development of an allergic reaction, which is most likely happening in this situation, requires prompt treatment to prevent complications such as anaphy laxis. The physician should be called first so that prompt treatment can be ordered.

19. Chelation therapy is used to prevent organ damage from the presence of too much iron in the body as a result of frequent transfusions. Which of the following should a nurse anticipate to be prescribed in chelation therapy? 1. Dalteparin sodium (Fragmin). 2. Deferoxamine (Desferal). 3. Diclofenac (Voltaren). 4. Diltiazem (Cardizem).

2. Deferoxamine (Desferal) is an antidote for acute iron toxicity.

22. Where should the top of the crutch bar be in relation to the axilla?

22. The crutch bar should not put pressure on nerves in the axilla.

23. Select the numbers of inches lateral to the heel where a crutch should be placed. 1. 1 to 3. 2. 4 to 5. 3. 6 to 8. 4. 9 to 10.

3. This position provides the best protection for balance and stability.

61. A 5-year-old is admitted to the pediatric unit. The child has an infusion of dextrose 5% via a line with a volume control chamber on a pump. The nurse knows that this system is used for administration of intravenous solutions for which of the following reasons? 1. Prevents accidental fluid overload. 2. Reduces the potential for bacterial infection. 3. Makes administering of intravenous fluids easier. 4. Is less costly.

61. 1. The volume control chamber functions as a safety device. No more than 2 hours of solution is placed in the chamber at a time. If the pump should be pro grammed incorrectly, the child would get only 2 hours of fluids; then the pump would alarm. The nurse is thereby alerted that the pump was programmed incorrectly.

The parent of a child with cystic fibrosis calls the clinic nurse to report that the child has developed tachypnea, tachycardia, dyspnea, pallor, and cyanosis. The nurse should tell the parent to bring the child to the clinic because these symptoms are suggestive of A. pneumothorax. B. bronchodilation. C. carbon dioxide retention. D. increased viscosity of sputum.

A

A 5-year-old child is brought the Emergency Department with abrupt onset of sore throat, pain with swallowing, fever, and sitting upright and forward. Acute epiglottitis is suspected. What are the most appropriate nursing interventions? (Select all that apply.) A. Vital signs B. Throat culture C. Medical history D. Assessment of breath sounds E. Emergency airway equipment readily available

A, C, D, E

A mother of a 3-year-old asks a clinic nurse about appropriate and safe toys for the child. The nurse should tell the mother that the most appropriate toy for a 3-year-old is which? A. A wagon B. A golf set C. A farm set D. A jack set with marbles

A. A wagon Rationale: Toys for the toddler must be strong, safe, and too large to swallow or place in the ear or nose. Toddlers need supervision at all times. Push-pull toys, large balls, large crayons, large trucks, and dolls are some of the appropriate toys. A farm set, a golf set, and jacks with marbles may contain items that the child could swallow.

An immediate intervention to teach parents for when an infant chokes on a piece of food would be to A. have infant lie quietly while a call is placed for emergency help. B. position infant in a head-down, face-down position and administer five quick back slaps. C. administer mouth-to-mouth resuscitation. D. give some water by a cup to relieve the obstruction.

B

A nurse is assigned to care for a hospitalized toddler. The nurse plans care, knowing that what should be the highest priority? A. Providing a consistent caregiver B. Protecting the toddler from injury C. Adapting the toddler to the hospital routine D. Allowing the toddler to participate in play and divisional activities

B. Protecting the toddler from injury Rationale: The toddler is at high risk for injury as a result of developmental abilities and an unfamiliar environment. Although consistency, adaptation, and diversionare important, protection from injury is the highest priority.

The nurse is instructing the caregiver of a child about reprimanding the child. The nurse recognizes that additional teaching is needed if the caregiver makes which statement to the child? A. "I like it when you obey." B. "I need you to listen to me." C. "You need to stop hitting your sister." D. "I don't like it when you hit your sister."

C. "You need to stop hitting your sister." Rationale: When reprimanding children, the person reprimanding should focus only on the misbehavior, not on the child. "I" messages rather than "you" messages should be used to express personal feelings without accusation. An "I" message attacks the behavior, not the child.

A 16-year-old is admitted to the hospital for acute appendicitis and an appendectomy is performed. Which nursing intervention is most appropriate to facilitate normal growth and development postoperatively? A. Encourage the child to rest and read. B. Encourage the parents to room in with the child. C. Allow the family to bring in the child's favorite computer games. D. Allow the child to interact with others in his or her same age group.

D. Allow the child to interact with others in his or her same age group. Rationale: Adolescents often are not sure whether they want their parents with them when they are hospitalized. Because of the importance of their peer group, separation from friends is a source of anxiety. Ideally, the members of the peer group will support their ill friend. Options 1, 2, and 3 isolate the child from the peer group.

1. Parents of a toddler with hypopituitarism ask the nurse, "What can we expect with this condition?" The nurse should respond with which statement? a. Growth is normal during the first 3 years of life. b. Weight is usually more retarded than height. c. Skeletal proportions are normal for age. d. Most of these children have subnormal intelligence.

c. Skeletal proportions are normal for age ANS: C In children with hypopituitarism, the skeletal proportions are normal. Growth is within normal limits for the first year of life. Height is usually more delayed than weight. Intelligence is not affected by hypopituitarism. DIF: Cognitive Level: Apply REF: p. 911 TOP: Integrated Process: Teaching/Learning MSC: Area of Client Needs: Physiologic Integrity: Physiologic Adaptation.

32. What can a nurse do to reinforce a 5-year-old's intellectual initiative when he asks about his upcoming surgery? 1. Answer the child's questions about his upcoming surgery in simple terms. 2. Provide the child with a book that has vivid illustrations about his surgery. 3. Tell the child he should wait and ask the doctor his questions. 4. Tell the child that she will answer his questions at a later time.

1. Answer the child's questions about his upcoming surgery in simple terms. The child is taking the initiative to ask questions, as all preschoolers do, and the nurse should always answer those questions as appropriately and accurately as possible.

57. Which statement by the mother would lead the nurse to suspect sexual abuse in a 4- year-old girl? 1. Masturbation. 2. Increased temper tantrums. 3. She is not grateful. 4. She does not demonstrate loyalty.

2

11. The nurse is caring for a 6-month-old infant diagnosed with meningitis. When she places the infant in the supine position and flexes his neck, she notes that the infant flexes his knees and hips. The nurse knows that this is referred to as: 1. Brudzinski sign. 2. Cushing triad. 3. Kernig sign. 4. Nuchal rigidity.

11. 1. Brudzinski sign occurs when the child responds to a flexed neck with an involuntary flexion of the hips and/or knees.

Impetigo ordinarily results in which of the following? a. No scarring b. Pigmented spots c. Slightly depressed scars d. Atrophic white scars

ANS: A Impetigo tends to heal without scarring unless a secondary infection occurs.

The management of a child who has just been stung by a bee or wasp should include the application of which of the following? a. Cool compresses b. Warm compresses c. Antibiotic cream d. Corticosteroid cream

ANS: A Bee or wasp stings are initially treated by carefully removing stinger, cleansing with soap and water, applying cool compresses, and using common household agents such as lemon juice or a paste made with aspirin and baking soda.

5. The nurse in the neonatal intensive care unit is caring for an infant with myelomeningocele scheduled for surgical repair in the morning. Which early signs of infection should the nurse monitor on this infant? (Select all that apply.) a. Temperature instability b. Irritability c. Lethargy d. Bradycardia e. Hypertension

ANS: A, B, C The nurse should observe an infant with unrepaired myelomeningocele for early signs of infection, such as temperature instability (axillary), irritability, and lethargy. Bradycardia and hypertension are not early signs of infection in infants. DIF: Cognitive Level: Analyze REF: p. 989 TOP: Integrated Process: Nursing Process: Evaluation MSC: Area of Client Needs: Physiologic Integrity: Reduction of Risk Potential

2. Which assessment findings should the nurse note in a school-age child with Duchenne muscular dystrophy (DMD)? (Select all that apply.) a. Lordosis b. Gower sign c. Kyphosis d. Scoliosis e. Waddling gait

ANS: A, B, E Difficulties in running, riding a bicycle, and climbing stairs are usually the first symptoms noted in Duchenne muscular dystrophy. Typically, affected boys have a waddling gait and lordosis, fall frequently, and develop a characteristic manner of rising from a squatting or sitting position on the floor (Gower sign). Lordosis occurs as a result of weakened pelvic muscles, and the waddling gait is a result of weakness in the gluteus medius and maximus muscles. Kyphosis and scoliosis are not assessment findings with DMD. DIF: Cognitive Level: Apply REF: p. 992 TOP: Integrated Process: Nursing Process: Assessment MSC: Area of Client Needs: Physiologic Integrity: Reduction of Risk Potential

3. A school-age child has been admitted to the hospital with an exacerbation of nephrotic syndrome. Which clinical manifestations should the nurse expect to assess? (Select all that apply.) a. Weight loss b. Facial edema c. Cloudy smoky brown-colored urine d. Fatigue e. Frothy-appearing urine

ANS: B, D, E A child with nephrotic syndrome will present with facial edema, fatigue, and frothy-appearing urine (proteinuria). Weight gain, not loss, is expected because of the fluid retention. Cloudy smoky brown-colored urine is seen with acute glomerulonephritis but not with nephrotic syndrome because there is no gross hematuria associated with nephrotic syndrome. PTS: 1 DIF: Cognitive Level: Apply REF: 914 TOP: Integrated Process: Nursing Process: Assessment MSC: Area of Client Needs: Physiologic Integrity: Reduction of Risk Potential

Which of the following is an important nursing consideration when caring for a child with impetigo contagiosa? a. Apply topical corticosteroids to decrease inflammation. b. Carefully remove dressings so as not to dislodge undermined skin, crusts, and debris. c. Carefully wash hands and maintain cleanliness when caring for an infected child. d. Examine child under a Wood lamp for possible spread of lesions.

ANS: C A major nursing consideration related to bacterial skin infections, such as impetigo contagiosa, is to prevent the spread of the infection and complications. This is done by thorough hand washing before and after contact with the affected child.

Which of the following explains physiologically the edema formation that occurs with burns? a. Vasoconstriction b. Decreased capillary permeability c. Increased capillary permeability d. Decreased hydrostatic pressure within capillaries

ANS: C With a major burn, an increase in capillary permeability occurs, allowing plasma proteins, fluids, and electrolytes to be lost. Maximal edema in a small wound occurs about 8 to 12 hours after injury. In larger injuries, the maximal edema may not occur until 18 to 24 hours.

The most appropriate time to perform bronchial postural drainage is A. immediately before all aerosol therapy. B. before meals and at bedtime. C. Immediately on arising and at bedtime. D. thirty minutes after meals and at bedtime

B

It is important that a child with acute streptococcal pharyngitis be treated with antibiotics to prevent A. otitis media. B. diabetes insipidus. C. nephrotic syndrome. D. acute rheumatic fever

D

One of the goals for children with asthma is to prevent respiratory tract infection because infections A. lessen effectiveness of medications. B. encourage exercise-induced asthma. C. increase sensitivity to allergens. D. can trigger an episode or aggravate asthmatic state.

D

The nurse is preparing to care for a 5-year-old who has been placed in traction following a fracture of the femur. The nurse plans care, knowing that which is the most appropriate activity for this child? A. A radio B. A sports video C. Large picture books D. Crayons and a coloring book

D. Crayons and a coloring book Rationale: In the preschooler, play is simple and imaginative, and includes activities such as crayons and coloring books, puppets, felt and magnetic boards, and Play-Doh. A radio or sports video are most appropriate for the adolescent. Large picture books are most appropriate for the infant.

Which would be the highest expected growth and development occurrence expected at 12 months of age for an infant who has had appropriate growth assessed at each well-child visit? A. Imitates sounds B. Smiles spontaneously C. Sits steadily unsupported D. Walks holding on to someone's hand

D. Walks holding on to someone's hand Rationale: Growth and development are sequential and predictable. One task builds on another. Mastery of a lower level task must occur before higher-level tasks are completed. At 12 months a child can walk holding on to someone's hand. Smiling, imitating sounds, and sitting steadily unsupported begins at 6 months of age.

1. The nurse is caring for an infant with a suspected urinary tract infection. Which clinical manifestations should be expected? (Select all that apply.) a. Vomiting b. Jaundice c. Failure to gain weight d. Swelling of the face e. Back pain f. Persistent diaper rash

a. Vomiting c. Failure to gain weight f. Persistent diaper rash ANS: A, C, F Vomiting, failure to gain weight, and persistent diaper rash are clinical manifestations observed in an infant with a UTI. PTS: 1 DIF: Cognitive Level: Understand REF: 909 TOP: Integrated Process: Nursing Process: Assessment MSC: Area of Client Needs: Physiologic Integrity: Physiologic Adaptation

15. A child is admitted with acute glomerulonephritis. The nurse should expect the urinalysis during this acute phase to show: a. bacteriuria, hematuria. b. hematuria, proteinuria. c. bacteriuria, increased specific gravity. d. proteinuria, decreased specific gravity.

b. hematuria, proteinuria ANS: B Urinalysis during the acute phase characteristically shows hematuria and proteinuria. Bacteriuria and changes in specific gravity are not usually present during the acute phase. PTS: 1 DIF: Cognitive Level: Understand REF: 860 TOP: Integrated Process: Nursing Process: Assessment MSC: Area of Client Needs: Physiologic Integrity: Physiologic Adaptation.

17. The nurse is teaching parents about signs of digoxin (Lanoxin) toxicity. Which is a common sign of digoxin toxicity? a. Seizures b. Vomiting c. Bradypnea d. Tachycardia

b. Vomiting ANS: B Vomiting is a common sign of digoxin toxicity. Seizures are not associated with digoxin toxicity. The child will have a slower heart rate, not respiratory rate. The heart rate will be slower, not faster. DIF: Cognitive Level: Understand REF: p. 754 TOP: Integrated Process: Teaching/Learning MSC: Area of Client Needs: Physiologic Integrity: Pharmacologic and Parenteral Therapies

8. Which is a parasite that causes acute diarrhea? a. Shigella organisms b. Salmonella organisms c. Giardia lamblia d. Escherichia coli

c. Giardia lamblia ANS: C G. lamblia is a parasite that represents 10% of non-dysenteric illness in the United States. Shigella, Salmonella, and E. coli are bacterial pathogens. DIF: Cognitive Level: Understand REF: p. 697 TOP: Integrated Process: Nursing Process: Assessment MSC: Area of Client Needs: Physiologic Integrity: Physiologic Adaptation

23. The nurse is caring for a child with acute renal failure. Which clinical manifestation should the nurse recognize as a sign of hyperkalemia? a. Dyspnea b. Seizure c. Oliguria d. Cardiac arrhythmia

d. Cardiac arrhythmia ANS: D Hyperkalemia is the most common threat to the life of the child. Signs of hyperkalemia include electrocardiograph anomalies such as prolonged QRS complex, depressed ST segments, peaked T waves, bradycardia, or heart block. Dyspnea, seizure, and oliguria are not manifestations of hyperkalemia. PTS: 1 DIF: Cognitive Level: Apply REF: 920 TOP: Integrated Process: Nursing Process: Assessment MSC: Area of Client Needs: Physiologic Integrity: Physiologic Adaptation

5. Myelosuppression, associated with chemotherapeutic agents or some malignancies such as leukemia, can cause bleeding tendencies because of a(n): a. decrease in leukocytes. b. increase in lymphocytes. c. vitamin C deficiency d. decrease in blood platelets..

d. decrease in blood platelets. ANS: D The decrease in blood platelets secondary to the myelosuppression of chemotherapy can cause an increase in bleeding. The child and family should be alerted to avoid risk of injury. Decrease in leukocytes, increase in lymphocytes, and vitamin C deficiency would not affect bleeding tendencies. DIF: Cognitive Level: Apply REF: p. 828 TOP: Integrated Process: Nursing Process: Planning MSC: Area of Client Needs: Physiologic Integrity: Physiologic Adaptation

29. A 3-year-old is hospitalized for an ASD repair. The parents have decided to go home for a few hours to spend time with her siblings. The child asks when her mommy and daddy will be back. The nurse's best response is: 1. "Your mommy and daddy will be back after your nap." 2. "Your mommy and daddy will be back at 6:00 p.m." 3. "Your mommy and daddy will be back later this evening." 4. "Your mommy and daddy will be back in 3 hours."

1. "Your mommy and daddy will be back after your nap." Preschoolers understand time in relation to events.

19. A 5-year-old female has been diagnosed with a seizure disorder. Her teacher noticed that she has been having episodes where she drops her pencil and simply appears to be daydreaming. This is most likely called: 1. An absence seizure. 2. An akinetic seizure. 3. A non-epileptic seizure. 4. A simple spasm seizure.

19. 1. Absence seizures occur frequently and last less than 30 seconds. The child ex periences a brief loss of consciousness where she may have a change in activ ity. These children rarely fall, but they may drop an object. The condition is often confused with daydreaming.

13. Which of the following would the nurse assess in a child diagnosed with osteomyelitis? Select all that apply. 1. Unwillingness to move affected extremity. 2. Severe pain. 3. Fever. 4. Following a closed fracture of an extremity. 5. Redness and swelling at the site.

13. 1. Pain in an extremity leads to resistance to movement. 2. Pain is frequently severe in osteomyelitis. 3. Fever is present in the acute phase of the illness. 5. Redness and swelling occur because of the infection.

17. Which signs and symptoms would the nurse expect to assess in a newborn with congenital hypothyroidism? 1. Preterm, diarrhea, and tachycardia. 2. Post-term, constipation, and bradycardia. 3. High-pitched cry, colicky, and jittery. 4. Lethargy, diarrhea, and tachycardia.

2

45. Individually identifiable health information may not be: 1. Faxed. 2. Mailed. 3. Copied. 4. Sold.

4. Selling information, such as lists of names and addresses, is disallowed by law, because the purpose of sharing the information is not the provision of care.

10. The mother of a newborn asks the nurse when the infant will receive the first hepatitis B immunization. Which is the nurse's best response? 1. "Babies receive the hepatitis B vaccine only if their mother is hepatitis B-positive." 2. "The first dose of the hepatitis B vaccine will be given prior to discharge today." 3. "The first dose of hepatitis B vaccine is given at 1 year of age." 4. "Babies receive their first hepatitis B vaccine at 6 months of age."

2. "The first dose of the hepatitis B vaccine will be given prior to discharge today." The first dose of hepatitis B vaccine is recommended between birth and 2 months. In most hospitals, newborns are given the vaccine prior to discharge.

49. A 10-month-old with heart failure weighs 10 kg. Digoxin (Lanoxin) is prescribed as 10 mcg/kg/day to be given every 12 hours. How much is given as each dose? 1. 10 mcg. 2. 50 mcg. 3. 100mcg. 4. 500 mcg.

2. 10 kg 10 mcg/kg/day 100 mcg 2 doses a day 50 mcg/dose.

45. A child with cystic fibrosis is placed on an oral antibiotic to be given four times a day for 14 days. Which of the following schedules is the most appropriate for the child? 1. 8 a.m., 12 p.m., 4 p.m., 8 p.m. 2. 7 a.m., 1 p.m., 7 p.m. 12 midnight. 3. 9 a.m., 1 p.m., 5 p.m., 9 p.m. 4. 10 a.m., 2 p.m., 6 p.m., 10 p.m.

2. Antibiotics should be scheduled to maintain therapeutic blood levels and not interfere with the child's sleep. This schedule allows for dosing every 6 hours during the day and allows the child to get 7 hours of uninterrupted sleep.

35. An infant is born with a sac protruding through the spine. The sac contains CSF, a portion of the meninges, and nerve roots. The nurse knows that this is referred to as: 1. Meningocele. 2. Myelomeningocele. 3. Spina bifida occulta. 4. Anencephaly.

2. A myelomeningocele is a sac that con tains a portion of the meninges, the CSF, and the nerve roots.

11. A child has been receiving prednisone for the past 3 weeks, and the parent wants to stop the medication. What is the nurse's best response? 1. "There should be no problem in stopping the medication since the child's symptoms have gone away." 2. "It is dangerous for steroids to be withdrawn immediately." 3. "Your child may develop severe psychological symptoms when prednisone is stopped." 4. Stopping the prednisone will require serum blood work."

2. Abrupt cessation of long-term steroid therapy can cause acute adrenal insuffi ciency that could lead to death. Long term steroid use can cause shrinkage of the adrenal gland, which decreases the production of the hormone.

37. A child is receiving continuous morphine by patient-controlled analgesia pump (basal and bolus) to control pain. Which side effect is the nurse's greatest concern? 1. Sedation. 2. Respiratory depression. 3. Nausea and vomiting. 4. Constipation.

2. Airway and breathing complications are the most serious side effects.

2. The nurse is caring for a 3-year-old female with an altered state of consciousness. The nurse determines that the child is oriented by asking the child to: 1. Name the president of the United States. 2. Identify her parents and state her own name. 3. State her full name and phone number. 4. Identify the current month but not the date.

2. Asking the 3-year-old to identify her parents and state her name is a devel opmentally appropriate way to assess orientation.

51. After the parent leaves the side rail down, a hospitalized toddler falls from the crib and suffers a skull fracture. Earlier that day, the nurse had discussed the importance of side rails with that parent and recorded the discussion in the nursing notes. Which element of malpractice is missing in this case? 1. Relationship with the patient. 2. Breach of duty of care. 3. Injury. 4. Damage.

2. Breach of duty of care is missing in this case, because the nurse met the stan dard of care. The nurse educated the parent on the importance of side rail safety before the incident occurred.

26. A premature infant with chronic lung disease is going home with complex care, in cluding oxygen, tracheostomy suction, and gastrostomy feedings. Which of the fol lowing discharge planning activities will be most effective in promoting continuity of care? 1. Send the parents to meet staff at the nursing agency that will be providing care in the home. 2. Plan a team conference at the hospital before discharge to include parents and hospital and home health staff. 3. Ask the parents to meet with the hospital respiratory therapy staff to discuss adapting the home environment to meet equipment needs. 4. Teach parents how to care for the child by utilizing hospital equipment and protocols.

2. By including parents, hospital staff, and home-care staff, a joint team con ference is the best option to promote continuity of care.

50. A 3-year-old male with CP has just been fitted for braces and is beginning physical therapy to assist with ambulation. His parents ask why he needs the braces when he was crawling without any assistive devices. Select the nurse's best response: 1. "The CP has progressed, and he now needs more assistance to ambulate." 2. "As your child ages and grows, the CP can manifest in different ways, and different muscle groups can need more assistance." 3. "Most children with CP need braces to help with ambulation." 4. "We have found that when children with CP use braces, they are less likely to fall."

2. CP can be manifested in different ways as the child grows. It does not progress,

50. A 16-year-old male is hospitalized for cystic fibrosis. He will be an inpatient for 2 weeks while he receives IV antibiotics. Which action taken by the nurse will most enhance his psychosocial development? 1. Fax the teen's teacher, and have her send in his homework. 2. Encourage the teen's friends to visit him in the hospital. 3. Encourage the teen's grandparents to visit frequently. 4. Tell the teen he is free to use his phone to call or text friends.

2. Encourage the teen's friends to visit him in the hospital.

26. Which stressor is common in hospitalized toddlers? Select all that apply. 1. Social isolation. 2. Interrupted routine. 3. Sleep disturbances. 4. Self-concept disturbances. 5. Fear of being hurt.

2. Interrupted routine. 3. Sleep disturbances. 5. Fear of being hurt.

13. The nurse is caring for a child who has just been admitted to the pediatric floor with a diagnosis of bacterial meningitis. When reviewing the child's plan of care, which of the following orders would the nurse question? 1. Maintain isolation precautions until 24 hours after receiving intravenous antibiotics. 2. Intravenous fluids at 11/2 times regular maintenance. 3. Neurological checks every 4 hours. 4. Administer acetaminophen for temperatures higher than 38°C (100.4°F).

2. Intravenous fluids at 11/2 times regular maintenance could cause fluid overload and lead to increased ICP.

45. A child with Ewing sarcoma is undergoing a limb salvage procedure. Which of the following statements indicates the parents understand the procedure? 1. "Our child will have a bone graft to save the limb." 2. "Our child will need follow-up lengthening procedures." 3. "Our child will need shorter shirt sleeves." 4. "Our child will not need chemotherapy."

2. Limb salvage requires the lengthening procedures to encourage the bone to continue to grow so the child will not have a short limb.

8. Which toy is the best choice for a 12-month-old? 1. Baby doll. 2. Musical rattle. 3. Board book. 4. Colorful beads.

2. Musical rattle. A musical rattle is the perfect toy for this child. Infants have short attention spans and enjoy auditory and visual stimulation.

34. After a school bus accident, four elementary school children are delivered by ambulance to the emergency department. Only one emergency physician is on duty. Which child should be directed to the physician first by the triage nurse? 1. The child who is crying uncontrollably and tries to move off the stretcher. 2. The child with severe abdominal pain, anxious and responsive, blood pressure 100/60; heart rate 120; RR 28. 3. The child with severe pain and distorted alignment of the right lower leg, indicating a possible fracture. 4. The child who is unresponsive, with fixed and dilated pupils, blood pressure 58/44; heart rate 60; RR 10.

2. Presentation indicates possible ab dominal injury and internal bleeding in early shock. With immediate atten tion, the injuries may be treatable; therefore, this child should be seen first.

44. Which activity can the nurse provide for a 9-year-old to encourage a sense of industry? 1. Allow the child to choose what time to take his medication. 2. Provide the child with the homework his teacher has sent. 3. Allow the child to assist with his bath. 4. Allow the child to help with his dressing change.

2. Provide the child with the homework his teacher has sent. child can achieve a sense of industry by completing his homework and staying on track with his classmates.

22. The nurse is working in the emergency room when an ambulance arrives with a 9-year-old male who has been having a generalized seizure for 35 minutes. The paramedics have provided blow-by oxygen and monitored vital signs. The patient does not have intravenous access yet. Which of the following medications should the nurse anticipate administering first? 1. Establish an intravenous line, and administer intravenous lorazepam. 2. Administer rectal diazepam. 3. Administer an oral glucose gel to the side of the child's mouth. 4. Place a nasogastric tube, and administer oral diazepam.

2. Rectal diazepam is first administered in an attempt to stop the seizure long enough to establish an IV, and then IV medication is administered.

58. A child is admitted to the pediatric unit with the diagnosis of SLE. On assessment, the nurse expects the child to have: 1. Leukemia. 2. Malar rash. 3. Weight gain. 4. Heart failure.

2. The "butterfly," or malar, rash is the most common manifestation of SLE.

9. A child is being evaluated in the emergency room for a possible diagnosis of meningitis. The nurse is assisting with the lumbar puncture and notes that the CSF is cloudy. The nurse is aware that cloudy CSF most likely means: 1. Viral meningitis. 2. Bacterial meningitis. 3. No infection, as CSF is usually cloudy. 4. Sepsis.

2. The CSF in bacterial meningitis is usually cloudy.

15. The nurse is providing education concerning Reye syndrome to a mothers' group. She knows that further education is needed when a mother states: 1. "I will have my children immunized against varicella and influenza." 2. "I will make sure not to give my child any products containing aspirin when my child is ill." 3. "Because I do not give my child aspirin, my child will probably never get Reye syndrome, but if that happens, it will be a very mild case." 4. "Children with Reye syndrome are admitted to the hospital."

2. The administration of aspirin or products containing aspirin have been associated with the development of Reye syndrome.

30. A 9-year-old is in a spica cast and complains of pain 1 hour after receiving intravenous opioid analgesia. What should the nurse do first? 1. Give more pain medication. 2. Perform a neuromuscular assessment. 3. Call the surgeon for orders. 4. Tell the child to wait another hour for the medication to work.

2. The nurse looks for the source of the pain by performing a neuromuscular assessment.

24. A child with a heart defect is placed on a maintenance dose of digoxin (Lanoxin) elixir. The dose is 0.07 mg/kg/day, and the child's weight is 16 lb. The medication is to be given two times a day. The nurse prepares how much digoxin (Lanoxin) to be given to the child? 1. 0.25 mg. 2. 0.37 mg. 3. 0.5 mg. 4. 2.5 mg.

24. 1. 0.25 mg.

20. The school nurse is called to the preschool classroom to evaluate a child. He has been noted to have periods where he suddenly falls and appears to be weak for a short time after the event. The preschool teacher asks what she should do. Select the nurse's best response. 1. "Have the parents follow up with his pediatrician as this is likely an atonic seizure." 2. "Find out if there have been any new stressors in his life, as it could be attention-seeking behavior." 3. "Have the parents follow up with his pediatrician as this is likely an absence seizure." 4. "The preschool years are a time of rapid growth, and many children appear clumsy. It would be best to watch him, and see if it continues."

20. 1. An atonic seizure is characterized by a loss of muscular tone, whereby the child may fall to the ground.

20. Which parts of the body should the nurse assess on a child in a spica cast? See the following figure, and use the relevant label(s). B A C D

20. C, D. The nurse needs to assess areas under the cast for drainage through the cast and assess neurocirculatory status of the feet.

21. A child comes to the clinic for diphtheria, pertussis, and tetanus and inactivated po liovirus vaccines. The child has a temperature of 101°F (38.3°C). The nurse should take which of the following actions? 1. Withhold the vaccines, and reschedule when the child is afebrile. 2. Administer Tylenol, and give the vaccine. 3. Give the vaccine, and instruct the parent to give Tylenol every 4 hours for the next 2 days. 4. Have the physician order an antibiotic and give the vaccine.

21. 1. Because fever is a side effect of the vaccine, the immunization should be withheld as it would be difficult to determine if the fever was due to the vaccine or another febrile illness. Immunizations can be given when the child has a low-grade fever.

28. A spinal curve of less than _________degrees that is nonprogressive does not require treatment for scoliosis.

28. 20. A 20-degree spinal curve that is non progressive will not disfigure or interfere with normal functioning, so it is not treated with bracing or surgery.

43. Which would be the most appropriate intervention for a 4 year-old boy brought to the emergency department after ingesting a small watch battery? 1. No treatment would be needed; assess and monitor airway, breathing, circulation, and abdominal pain. 2. Ask the mother the time of the ingestion; if it was more than 2 hours ago, it will probably pass in his bowel movement. 3. Assess and monitor airway, breathing, circulation, and abdominal pain; anticipate admission and prepare for surgical intervention. 4. Discuss childproofing measures needed in the home with a 4-year-old child; provide anticipatory guidance concerning other possible poisonous ingestions.

3

50. Which nursing plan would be most successful if the nurse has to administer activated charcoal to a 5-year-old boy? 1. Have his parents tell him he has to drink it while providing a movie to distract him. 2. Tell him it is candy, it tastes good, do not let him look at it, and tell him he will get a toy after he takes it all down. 3. Mix it with a sweetening flavoring, provide a straw, give in an opaque cup with a cover. 4. Have his mother take some first to show the boy it does not taste too bad, and then administer it to him quickly.

3

53. Which statement would be most therapeutic to a child the nurse suspects has been abused? 1. "Who did this to you? This is not right." 2. "This is wrong that your mother did not protect you." 3. "This is not your fault; you are not to blame for this." 4. "I will not tell anyone."

3

55. Which assessment of a 2-year-old with burns on his feet would cause suspicion of child abuse? 1. Splash marks on his right lower leg. 2. Burns noted on right arm. 3. Symmetrical burns on both feet. 4. Burns mainly noted on right foot.

3

16. The nurse is using the FLACC scale to rate the pain level in a 9-month-old. Which is the nurse's best response to the father's question of what the FLACC scale is? 1. "It estimates a child's level of pain utilizing vital sign information." 2. "It estimates a child's level of pain based on parents' perception." 3. "It estimates a child's level of pain utilizing behavioral and physical responses." 4. "It estimates a child's level of pain utilizing a numeric scale from 0 to 5."

3. "It estimates a child's level of pain utilizing behavioral and physical responses." The FLACC scale utilizes behavioral and physical responses of the child to measure the child's level of pain. The scale utilizes facial expression, leg position, activity, intensity of cry, and level of consolability.

45. The mother of 11-year-old fraternal twins tells the nurse at their well-child checkup that she is concerned because her daughter has gained more weight and height than her twin brother. The mother is concerned that there is something wrong with her son. The nurse's best response is: 1. "I understand your concern. I will talk with the physician, and we can draw some lab work." 2. "Let me ask you whether your son has been ill lately." 3. "It is normal for girls to grow a little taller and gain more weight than boys at this age." 4. "It is normal for you to be concerned, but I am sure your son will catch up with your daughter eventually."

3. "It is normal for girls to grow a little taller and gain more weight than boys at this age."

9. A nurse is administrating vancomycin intravenously to a child and sets the pump to infuse the medication over 90 minutes. Which of the following adverse reactions is the nurse trying to prevent? 1. Vomiting. 2. Headache. 3. Flushing of the face, neck, and chest. 4. Hypertension.

3. "Red man syndrome" or "red neck syndrome" is flushing of the face, neck, and upper chest associated with too rapid an infusion of vancomycin. This can be prevented with infusing the vancomycin over 90 to 120 minutes and pretreating the patient with Ben adryl (diphenhydramine) prior to the infusion.

3. An infant returned from GI surgery 4 hours ago. The parent refuses pain medication for the baby and states, "The baby is crying because of hunger. Can I offer a bottle?" How should the nurse best advocate for the infant? 1. Review the results of the observational pain scale with the parent, and explain why the infant must have nothing by mouth. 2. Use nonpharmacological measures first to see if the pain rating of 8 (out of 10) decreases. 3. Ask the parent to use the observational pain tool to measure the infant's pain. 4. Call the physician to obtain an order to feed the infant.

3. 1. As the advocate for the infant and par ent, the nurse has the responsibility to educate the parent about the infant's condition after surgery, the rationale for having nothing by mouth, and the assessment and treatment of pain. By being informed, the parent can make educated assessments and decisions, collaborating with the nurse to meet the infant's needs.

3. Which of the following conditions can occur in untreated DDH? Select all that apply. 1. Duck gait. 2. Pain. 3. Osteoarthritis in adulthood. 4. Osteoporosis in adulthood. 5. Increased flexibility of the hip joint in adulthood.

3. 1. Due to abnormal hip joint function, the patient's gait is stiff and waddling. 2. Due to abnormal femoral head place ment, the patient may experience pain and decreased flexibility in adulthood. 3. Due to abnormal femoral head place ment, the patient may experience os teoarthritis in the hip joint in adulthood.

23. Amoxicillin (Amoxil) 250 mg by mouth every 8 hours is prescribed for a child who weighs 42 lb. The nurse knows that the safe pediatric dosage is 25 to 90 mg/kg/day. The nurse determines that: 1. The prescribed dose is too low. 2. The prescribed dose is too high. 3. The prescribed dose is within a safe range. 4. There is not enough information to determine the safe dose.

3. 750 mg/day is between 477.25 mg/day and 954.5 mg/day. The nurse first must determine what the safe dosage range is.

27. Which nursing action would help foster a hospitalized 3-year-old's sense of autonomy? 1. Let the child choose what time to take the oral antibiotics. 2. Allow the child to have a doll for medical play. 3. Allow the child to administer her own dose of Keflex (cephalexin) via oral syringe. 4. Let the child watch age-appropriate videos.

3. Allow the child to administer her own dose of Keflex (cephalexin) via oral syringe. Allowing preschoolers to participate in actions of which they are capable is an excellent way to enhance their autonomy.

57. A nurse is caring for an 11-month-old who has received atropine to treat sinus bradycardia. The nurse knows that a common adverse reaction to atropine is which of the following? 1. Diarrhea. 2. Increased urine output. 3. No tears when crying. 4. Lethargy.

3. Atropine dries up secretions and also lessens the response of ciliary and iris sphincter muscles in the eye, causing mydriasis.

48. Which of the following is a toxic reaction in a child taking digoxin (Lanoxin)? 1. Weight gain. 2. Tachycardia. 3. Nausea and vomiting. 4. Seizures.

3. Digoxin (Lanoxin) toxicity in infants and children may present with nausea, vomiting, anorexia, or a slow, irregular, apical heart rate.

2. How can the nurse best facilitate the trust relationship between infant and parents while the infant is hospitalized? The nurse should: 1. Encourage the parents to remain at their child's bedside as much as possible. 2. Keep parents informed about all aspects of their child's condition. 3. Encourage the parents to hold their child as much as possible. 4. Advise the parents to participate actively in their child's care.

3. Encourage the parents to hold their child as much as possible. Having parents hold their child while in the hospital is an excellent means of building the trust relationship. Infants are most secure when they are being held, patted, and spoken to.

23. Which should the nurse do to prevent separation anxiety in a hospitalized toddler? 1. Assume the parental role when parents are not able to be at the bedside. 2. Encourage the parents to always remain at the bedside. 3. Establish a routine similar to that of the child's home. 4. Rotate nursing staff so the child becomes comfortable with a variety of nurses.

3. Establish a routine similar to that of the child's home. It is very important to try to maintain a child's home routine both when par- ents are present and when they have to leave the hospital. This will increase the child's sense of security and decrease anxiety. Providing consistent nursing care is impor- tant, not rotating staff.

1. The nurse is caring for a child who has been in an MVA. The child continues to fall asleep unless her name is called or she is gently shaken. The nurse knows that this state of consciousness is referred to as: 1. Coma. 2. Delirium. 3. Obtunded. 4. Confusion.

3. Obtunded describes a state of conscious ness in which the child has a limited re sponse to the environment and can be aroused by verbal or tactile stimulation.

20. The mother of a child 2 years 6 months has arranged a play date with the neighbor and her child 2 years 9 months. During the play date the two mothers should expect that the children will do which of the following? 1. Share and trade their toys while playing. 2. Play with one another with little or no conflict. 3. Play alongside one another but not actively with one another. 4. Only play with one or two items, ignoring most of the other toys.

3. Play alongside one another but not actively with one another. Toddlers engage in parallel play. They often play alongside another child, but they rarely engage in activities with the other child. Toddlers do not share their possessions well. One of their favorite words is "mine."

44. A child in the emergency room is being treated with albuterol (Proventil) aerosol mist treatments for an acute asthma attack. She requires treatment every 2 hours. Which of the following adverse effects of the medication should the nurse expect? 1. Lethargy and bradycardia. 2. Decreased blood pressure and dizziness. 3. Nervousness and tachycardia. 4. Increased blood pressure and fatigue.

3. Potential side effects of this medication are stimulation of the central nervous system and cardiovascular system. Tachycardia is the most frequent side effect of albuterol (Proventil).

19. The pediatric unit is re-evaluating the procedure used by staff to suction tracheostomies. Which activity demonstrates followership activities, appropriate to a recent nursing graduate? 1. Rewrite the hospital policy and procedure for tracheostomy suctioning. 2. Work with information technology to plan new computer screens for charting. 3. Provide input on the feasibility and effectiveness of the new procedure. 4. Hold in-services to educate staff on the techniques and rationale for the new procedure.

3. Testing a new procedure falls within the scope of a new practitioner and is an appropriate followership activity.

41. Which statement would indicate to the nurse that a school-age child is not developmentally on track for age? 1. The child is able to follow a four- to five-step command. 2. The child started wetting the bed on admission to the hospital. 3. The child has an imaginary friend named Kelly. 4. The child enjoys playing board games with her sister.

3. The child has an imaginary friend named Kelly.

30. After administrating a narcotic, the nurse will monitor the child for pain relief. Which of the following assessments should be a priority in addition to pain relief? 1. Respirations. 2. Bowel sounds. 3. Blood pressure. 4. Oxygen saturation.

30. 1. The primary purpose of administrating an opioid analgesic is to relieve pain. Side effects placing the child at great est risk are respiratory depression and decreased level of consciousness.

35. The nurse is providing care for an adolescent with complex needs after surgical correction of a severe bowel obstruction. On entering the room, the nurse prioritizes care and decides to complete which task first? 1. Change the central intravenous line dressing, which is loose and gaping. 2. Empty the ileostomy bag, which is moderately full of liquid stool. 3. Change the gauze dressing around the Jackson-Pratt drain. 4. Check for correct positioning of the nasogastric tube in the stomach.

35. 1. The best method for preventing con tamination of the central intravenous line site is to complete this procedure first. This lessens the likelihood of spreading microorganisms from the GI track to the central line. The possible complication of sepsis is most life threatening.

35. What condition is the harness used for in the following figure?_________________

35. It is used for DDH. The Pavlik harness is used to treat DDH in neonates.

25. Which comment should the parent of a 2-year-old expect from the toddler about a new baby brother? 1. "When the baby takes a nap, will you play with me?" 2. "Can I play with the baby?" 3. "The baby is so cute. I love him." 4. "It is time to put him away so we can play."

4. "It is time to put him away so we can play." This is a typical statement that would be made by a toddler. Toddlers are very egocentric and do not consider the needs of the other child.

53. Which of the following is the most common adverse reaction from erythromycin? 1. Weight gain. 2. Constipation. 3. Mouth sores. 4. Nausea and vomiting.

4. Common adverse reactions to erythro mycin include nausea, vomiting, diar rhea, abdominal pain, and anorexia. Erythromycin should be given with a full glass of water and after meals. Because these gastrointestinal adverse reactions occur commonly, it may be necessary to give erythromycin with food.

57. The diabetes clinic conducts a disease management program for children with type I diabetes. Which test is the most valid indicator of compliance with the diabetes regimen? 1. Fasting blood glucose level. 2. Fingerstick glucose for 24 hours. 3. Urine ketone strip. 4. HgA1C assay.

4. HgA1C reflects the average blood glu cose level over 3 months, making it the most valid indicator.

5. Which statement accurately describes the best method for assessing a 12-month-old? 1. The nurse should assess the child on the examining table. 2. The nurse should assess the child in a head-to-toe sequence. 3. The nurse should have the child's mother assist in holding her down. 4. The nurse should assess the child while she is in her mother's lap.

4. The nurse should assess the child while she is in her mother's lap. Infants are most secure when in prox- imity to the parent. The parent's lap is an excellent place to assess the child.

33. A child who has been diagnosed with conjunctivitis is ordered to have eye ointment applied three times a day. Which of the following should the nurse do first? 1. Remove any discharge from the affected eye. 2. Ensure the ointment is at room temperature. 3. Hold the tip of the eye ointment parallel to the eye. 4. Wash hands.

4. The procedure for instilling eye oint ment begins with washing hands fol lowed by donning clean gloves.

47. An infant with short bowel syndrome will be discharged home on total parenteral nutrition (TPN) and gastrostomy feedings. What should be included in the discharge teaching? a. Prepare family for impending death. b. Teach family signs of central venous catheter infection. c. Teach family how to calculate caloric needs. d. Secure TPN and gastrostomy tubing under diaper to lessen risk of dislodgment.

ANS: B During TPN therapy, care must be taken to minimize the risk of complications related to the central venous access device, such as catheter infections, occlusions, or accidental removal. This is an important part of family teaching. The prognosis for patients with short bowel syndrome depends in part on the length of residual small intestine. It has improved with advances in TPN. Although parents need to be taught about nutritional needs, the caloric needs and prescribed TPN and rate are the responsibility of the health care team. The tubes should not be placed under the diaper due to risk of infection. DIF: Cognitive Level: Apply REF: p. 727 TOP: Integrated Process: Teaching/Learning MSC: Area of Client Needs: Physiologic Integrity: Pharmacologic and Parenteral Therapies

28. Calcium carbonate is given with meals to a child with chronic renal disease. The purpose of this is to: a. prevent vomiting. b. bind phosphorus. c. stimulate appetite. d. increase absorption of fat-soluble vitamins.

ANS: B Oral calcium carbonate preparations combine with phosphorus to decrease gastrointestinal absorption and the serum levels of phosphate. Serum calcium levels are increased by the calcium carbonate, and vitamin D administration is necessary to increase calcium absorption. Calcium carbonate does not prevent vomiting, stimulate appetite, or increase the absorption of fat-soluble vitamins. PTS: 1 DIF: Cognitive Level: Apply REF: 922 TOP: Integrated Process: Nursing Process: Implementation MSC: Area of Client Needs: Physiologic Integrity: Pharmacologic and Parenteral Therapy

10. A nurse is teaching nursing students the physiology of congenital heart defects. Which defect results in decreased pulmonary blood flow? a. Atrial septal defect b. Tetralogy of Fallot c. Ventricular septal defect d. Patent ductus arteriosus

ANS: B Tetralogy of Fallot results in decreased blood flow to the lungs. The pulmonic stenosis increases the pressure in the right ventricle, causing the blood to go from right to left across the ventricular septal defect. Atrial and ventricular septal defects and patent ductus arteriosus result in increased pulmonary blood flow. DIF: Cognitive Level: Understand REF: p. 743 TOP: Integrated Process: Nursing Process: Assessment MSC: Area of Client Needs: Physiologic Integrity: Physiologic Adaptation

John's mother tells the nurse that he keeps scratching the areas where he has poison ivy. The nurse's response should be based on which of the following? a. Poison ivy does not itch and needs further investigation. b. Scratching the lesions will not cause a problem. c. Scratching the lesions will cause the poison ivy to spread. d. Scratching the lesions may cause them to become secondarily infected.

ANS: D Poison ivy is a contact dermatitis that results from exposure to the oil urushiol in the plant. Every effort is made to prevent the child from scratching, since the lesions can become secondarily infected.

A high-protein diet for the child with major burns is ordered to: a. promote growth. b. improve appetite. c. diminish risks of stress-induced hyperglycemia. d. avoid protein breakdown.

ANS: D The diet must provide sufficient calories to meet the increased metabolic needs and enough protein to avoid protein breakdown.

A 6-month-old infant is admitted to the hospital. The nurse weighs the infant and notes that the infant weighs 14 pounds. Which statement by the mother indicates that further teaching is needed? A. "His weight for his age is just right." B. "I am so glad he is gaining the correct amount of weight for his age." C. "I will have to increase his milk intake because he is not gaining enough weight." D. "He weighed 7 pounds when he was born so he is at the correct weight for his age."

C. "I will have to increase his milk intake because he is not gaining enough weight." Rationale: Newborns double their birth weight at 5 to 6 months of age and triple it by 1 year. Therefore, options 1, 2, and 4 are correct statements. Option 3 indicates the need for further teaching.

The nurse prepares to take the blood pressure of a school-age child. To obtain an accurate measurement, how should the nurse position the blood pressure cuff? A. One half of the distance between the antecubital fossa and the shoulder B. One third of the distance between the antecubital fossa and the shoulder C. Two thirds of the distance between the antecubital fossa and the shoulder D. One quarter of the distance between the antecubital fossa and the shoulder

C. Two thirds of the distance between the antecubital fossa and the shoulder Rationale: The size of the blood pressure cuff is important. Cuffs that are too small will cause falsely elevated values, and those that are too large will cause inaccurate low values. The cuff should cover two thirds of the distance between the antecubital fossa and the shoulder.

The pediatric nurse is caring for a hospitalized toddler. What does the nurse determine is the most appropriate play activity for the toddler? A. Listening to music B. Playing peek-a-boo C. Hand sewing a picture D. Playing with a push-pull toy

D. Playing with a push-pull toy Rationale: The toddler has increased use of motor skills and enjoys manipulating small objects such as toy people, cars, and animals. Push-pull toys are appropriate for this age. Listening to music is most appropriate for an adolescent. Playing peek-a-boo is most appropriate for an infant. Hand sewing a picture is most appropriate for a school-age child.

12. Which clinical manifestations in an infant would be suggestive of spinal muscular atrophy (Werdnig-Hoffmann disease)? a. Hyperactive deep tendon reflexes b. Hypertonicity c. Lying in the frog position d. Motor deficits on one side of body

c. Lying in the frog position

30. To help the adolescent deal with diabetes, the nurse must consider which characteristic of adolescence? a. Desire to be unique b. Preoccupation with the future c. Need to be perfect and similar to peers d. Need to make peers aware of the seriousness of hypoglycemic reactions

...ANS: C Adolescence is a time when the individual wants to be perfect and similar to peers. Having diabetes makes adolescents different from their peers. Adolescents do not wish to be unique; they desire to fit in with the peer group and are usually not future oriented. Forcing peer awareness of the seriousness of hypoglycemic reactions would further alienate the adolescent with diabetes. The peer group would focus on the differences. DIF: Cognitive Level: Analyze REF: p. 940 TOP: Integrated Process: Teaching/Learning MSC: Area of Client Needs: Health Promotion and Maintenance: Growth and Development

15. Which intervention should the nurse implement for a newborn diagnosed with galactosemia? 1. Eliminate all milk and lactose-containing foods. 2. Encourage breastfeeding as long as possible. 3. Encourage lactose-containing formulas. 4. Avoid feeding soy-protein formula to the newborn.

1

2. The nurse received a report on a new admission: a 3-year-old with Kawasaki disease. Understanding the etiology and major complications of Kawasaki disease, the priority nursing intervention would be: 1. Continuous cardiovascular and oxygen saturation monitoring. 2. Vital signs every 4 hours until stable. 3. Strict intake and output monitoring hourly. 4. Begin aspirin therapy after fever has resolved.

1

24. Which would be the nurse's best response if a mother asks if her baby still needs the Hib vaccine because he already had Hib? 1. "Yes, it is recommended that the baby still get the Hib vaccine." 2. "No, if he has had Hib, he will not need to receive the vaccine." 3. "Let me take a nasal swab first; if it is negative, he will receive the Hib vaccine." 4. "The physician will order a blood test, and depending on results, your child may need the vaccine."

1

32. A 10-month-old female is carried into the emergency department by her parents after she fell down 15 stairs in her walker. Which would be your highest priority nursing intervention? 1. Assess airway while simultaneously maintaining cervical spine precautions. 2. Assess airway, breathing, and circulation simultaneously. 3. Prepare for diagnostic radiological testing to check for any injuries. 4. Obtain venous access and draw blood for testing.

1

35. An adolescent male comes in for his yearly physical. Which would be the most appropriate injury prevention/safety teaching for him? 1. Inquire which are his favorite sports and discuss his knowledge and application of appropriate safety principles. 2. Tell him to be careful performing sports activities because every sport has the potential for injury. 3. Tell him not to let his friends encourage him to drink or smoke or take any drugs. 4. Ask his mother what sports he plays and if he wears a helmet with contact sports.

1

4. The mother of an 18-month-old is discharged from the hospital after the child has a febrile seizure secondary to exanthem subitum (roseola). On discharge, the mother asks the nurse if her 6-year-old twins will get sick. Which teaching about the transmission of roseola would be most accurate? 1. The child should be isolated in the home until the vesicles have dried. 2. The child does not need to be isolated from the older siblings. 3. Administer acetaminophen to the older siblings to prevent seizures. 4. Monitor older children for seizure development.

1

41. Which response about safety measures is the most appropriate advice to the 2-year old girl's mother who had her older home remodeled to reduce the lead level? 1. "Wash and dry the child's hands and face before eating." 2. "Remodeling the home to remove the lead is all you need to do." 3. "It is best to use hot water to prepare the child's food to decrease the lead." 4. "Diet does not matter in reducing lead levels in the child."

1

45. Which would be the priority nursing intervention for a child with carbon monoxide poisoning? 1. Provide supplemental 100% oxygen. 2. Provide continuous oxygen saturation monitoring. 3. Establish intravenous access. 4. Draw blood for a STAT carbon monoxide level.

1

46. Which of the following would be appropriate anticipatory guidance for a well-care visit of a 17-year-old? 1. Discuss alcohol use and potential for alcohol poisoning. 2. Discuss secondary sex characteristics that will develop. 3. Teach about anger management and safe sex. 4. Teach about peer pressure and desire for independence.

1

31. Which type of seizure involves both hemispheres of the brain? a. Focal b. Partial c. Generalized d. Acquired

. Generalized ANS: C Clinical observations of generalized seizures indicate that the initial involvement is from both hemispheres. Focal seizures may arise from any area of the cerebral cortex, but the frontal, temporal, and parietal lobes are most commonly affected. Partial seizures are caused by abnormal electric discharges from epileptogenic foci limited to a circumscribed region of the cerebral cortex. A seizure disorder that is acquired is a result of a brain injury from a variety of factors; it does not specify the type of seizure. DIF: Cognitive Level: Remember REF: p. 896 TOP: Integrated Process: Nursing Process: Assessment MSC: Area of Client Needs: Physiologic Integrity: Physiologic Adaptation

3. The nurse is evaluating the laboratory results on cerebral spinal fluid (CSF) from a 3- year-old child with bacterial meningitis. Which findings confirm bacterial meningitis? (Select all that apply.) a. Elevated white blood cell (WBC) count b. Decreased glucose c. Normal protein d. Elevated red blood cell (RBC) count

...ANS: A, B The cerebrospinal fluid analysis in bacterial meningitis shows elevated WBC count, decreased glucose, and increased protein content. There should not be RBCs evident in the CSF fluid. DIF: Cognitive Level: Analyze REF: p. 890 TOP: Integrated Process: Nursing Process: Evaluation MSC: Area of Client Needs: Physiologic Integrity: Physiologic Adaptation

11. Which finding would the nurse consider abnormal when performing a physical assessment on a 6-month-old? 1. Posterior fontanel is open. 2. Anterior fontanel is open. 3. Beginning signs of tooth eruption. 4. Able to track and follow objects.

1. Posterior fontanel is open. The posterior fontanel should close between 6 and 8 weeks of age. The anterior fontanel usually closes between 12 and 18 months. The infant usually has a first tooth erupt at about 6 months of age. The infant should be able to track objects.

12. Which of the following should be included in teaching a family about postsurgical care for SCFE? Select all that apply. 1. The patient will receive help with weight-bearing ambulation 24 to 48 hours after surgery. 2. Monitoring of pain medication to prevent drug dependence. 3. Instruction on pin site care. 4. Offering low-calorie meals to encourage weight loss. 5. Correct use of crutches by the patient. 6. Outpatient physical therapy for 6 to 8 weeks.

12. 3. The parents will be assessing pin site for infection and stability upon dis charge. Instructions on care should be demonstrated for and then by the parents. 5. Instruction on crutch usage will be given prior to discharge. Crutch walk ing will not be done during the early postoperative stage.

13. A child is going to start growth hormone therapy. Which of the following should the nurse include in the discharge teaching plan? 1. The child is expected to grow 3 to 5 inches during the first year of treatment. 2. The parents must measure the child's weight and height daily. 3. The parents must remember that once the growth hormone therapy is started, they will need to continue the therapy until the child is 21 years old. 4. There are no side effects from taking growth hormones.

13. 1. The expected growth rate with growth hormone therapy is 3 to 5 inches in the first year.

15. A child has been started on metoclopramide (Reglan) for gastric esophageal reflux disease. Which of the following should the nurse include in the teaching plan? 1. This drug increases gastrointestinal motility. 2. The drug decreases tone in the lower esophageal sphincter. 3. The drug prevents diarrhea. 4. The drug induces the release of acetylcholine.

15. 1. Metoclopramide (Reglan) is a gas trointestinal stimulant that increases motility of the gastrointestinal tract, shortens gastric emptying time, and reduces the risk of the esophagus be ing exposed to gastric content.

16. Which of the following activities would be most effective in helping a new nurse manager develop transformational leadership abilities in a rapidly changing practice setting? 1. Select a mentor and a professional support group. 2. Focus on activities that avoid change and involve minimal risk. 3. Reward or correct followers to maintain current levels of practice. 4. Use own judgment when making decisions, with little input from staff.

16. 1. A mentor can model new behavior and coach the new nurse manager while providing support during new experiences.

16. High-dose prednisone is given to a 5-year-old with leukemia as part of the treatment protocol. The nurse will monitor the child for which of the following? 1. Diabetes. 2. Deep vein thrombosis. 3. Nephrotoxicity. 4. Hepatotoxicity.

16. 1. One of the side effects of high-dose steroids can be diabetes mellitus. The child needs to be evaluated so prompt treatment can be initiated. The dia betes is self-limiting and after the steroids are discontinued should no longer be present. Other side effects include mood changes, hirsutism, trunk obesity, thin extremities, gastric bleeding, poor wound healing, hyper tension, immunosuppression, insomnia, and increased appetite.

17. A 15-year-old is immobilized after SCFE surgery. Which of the following instructions should the nurse give the parents? Select all that apply. 1. Continue upper body exercises to decrease loss of muscle strength. 2. Do not turn the teen in bed when complaining of pain. 3. Provide homework, computer games, and other activities to decrease boredom. 4. Do most activities of daily living for the teen. 5. Expect expressions of anger and hostility. 6. Continue setting limits on behavior.

17. 1. Immobilization can lead to a decrease in muscle strength. Upper body exer cises should be continued soon after surgery. 3. It is important for this patient to con tinue as many normal activities as possible. This should include school work and leisure activities. 5. Some expressions of anger and hostil ity are normal, as this adolescent is losing some independence with this immobility. 6. Continuation of setting limits on behavior is important to keep as much normalcy as possible.

17. Which of the following activities best falls within the scope of management rather than leadership? 1. Planning the staffing schedule for a 2-month period. 2. Empowering the staff to meet patient care goals for the year. 3. Encouraging staff to utilize reflective practice and self-awareness. 4. Inspiring staff to develop a shared vision of quality patient care.

17. 1. Managers coordinate and utilize re sources. Planning for staffing is an example of both coordination and utilization of staff.

18. The nurse is caring for a child with meningitis. The parents call for the nurse as "something is wrong." When the nurse arrives, she notes that the child is having a generalized tonic-clonic seizure. Which of the following should the nurse do first? 1. Administer blow-by oxygen and call for additional help. 2. Reassure the parents that seizures are common in children with meningitis. 3. Call a code and ask the parents to leave the room. 4. Assess the child's temperature and blood pressure.

18. 1. The child experiencing a seizure usually requires more oxygen as the seizure increases the body's metabolic rate and demand for oxygen. The seizure may also affect the child's air way, causing the child to be hypoxic. It is always appropriate to give the child blow-by oxygen immediately. The nurse should remain with the child and call for additional help.

18. Tasks of followership that would be expected of a new nursing graduate include which of the following? 1. Collaborates with others, honors standards, demonstrates individual accountability. 2. Envisions organizational goals, affirms values, represents the group. 3. Motivates others, develops standards, eliminates barriers to care. 4. Evaluates system processes, recommends ways to improve the system.

18. 1. These followership activities are ap propriate for a new graduate nurse.

7. Which of the following would be the priority teaching to the parent of a child diagnosed with chickenpox (varicella) who was prescribed diphenhydramine (Benadryl) for itching? 1. Give a warm bath with mild soap before lotion application. 2. Avoid Caladryl lotion while taking diphenhydramine (Benadryl). 3. Apply Caladryl lotion generously to decrease itching. 4. Give a cool shower with mild soap to decrease itching.

2

What would be the best response if the mother of a 10-year-old boy on kidney dialysis tells the nurse he has no appetite and only eats bananas? 1. "Right now his stomach is upset, and as long as he is eating something to give him strength, it is fine." 2. "Let's talk about your son and his diet." 3. "Bananas are good to eat; they are rich in needed nutrients." 4. "Did you try asking him what else he may want to eat?"

2

6. The nurse is instructing a new breastfeeding mother in the need to provide her pre- mature infant with an adequate source of iron in her diet. Which statement reflects a need for further education of the new mother? 1. "I will use only breast milk or an iron-fortified formula as a source of milk for my baby until she is at least 12 months old." 2. "My baby will need to have iron supplements introduced when she is 4 months old." 3. "I will need to add iron supplements to my baby's diet when she is 2 months old." 4. "When my baby begins to eat solid foods, I should introduce iron-fortified cereals to her diet."

3. "I will need to add iron supplements to my baby's diet when she is 2 months old." Premature infants have iron stores from the mother that last approximately 2 months, so it is important to introduce an iron supplement by 2 months of age. Full-term infants have iron stores that last approximately 4 to 6 months.

39. A 3-year-old boy has been hospitalized because he fell down the stairs. His mother is crying and states, "This is all my fault." Which is the nurse's best response? 1. "Accidents happen. You shouldn't blame yourself." 2. "Falls are one of the most common injuries in this age group." 3. "It may be a good idea to put a baby gate on the stairs." 4. "Your son should be proficient at walking down the stairs by now."

2. "Falls are one of the most common injuries in this age group." Falls are one of the most common injuries, and it may make the parent feel better to know that this is common.

4. Which statement by an infant's mother leads the nurse to believe that she needs further education about the nutritional needs of a 6-month-old? 1. "I will continue to breastfeed my son and will give him rice cereal three times a day." 2. "I will start my son on fruits and gradually introduce vegetables." 3. "I will start my son on carrots and will introduce one new vegetable every few days." 4. "I will not give my son any more than 8 ounces of baby juice per day."

2. "I will start my son on fruits and gradually introduce vegetables." Infants should be started on vegetables prior to fruits. The sweetness of fruits may inhibit infants from taking vegetables. Cereal can be introduced between 4 and 6 months of age. Infants can be given fruit juice by 6 months of age, but it is recommended not to exceed 4 to 6 ounces per day.

50. It is recommended that a child with metastatic rhabdomyosarcoma undergo a bone marrow transplant. Education regarding life-threatening side effects should discuss: 1. Diarrhea. 2. Fever. 3. Skin breakdown. 4. Tumor shrinkage.

2. Fever indicates infection that can be life-threatening after a bone marrow transplant.

8. The nurse is working in the PICU caring for an infant who has just returned from having a ventriculoperitoneal shunt placed. Which position initially will be most beneficial for this child? 1. Semi-Fowler in an infant seat. 2. Flat in the crib. 3. Trendelenburg. 4. In the crib with the head elevated to 90 degrees.

2. Flat in the crib is the position usually used initially, with the angle gradually increasing as the child tolerates.

33. After receiving the change-of-shift report, the nurse prioritizes care for the day. Which patient should the nurse assess first? 1. 1-month-old admitted 1 day ago with fever and possible sepsis. 2. 14-month-old with a tracheostomy admitted for respiratory syncytial virus bronchiolitis. 3. 18-month-old with acute viral meningitis. 4. 7-year-old 1 day after an appendectomy.

2. Following the ABCs (airway-breathing circulation), this baby has the greatest potential for a life-threatening compli cation if the tracheostomy becomes obstructed by mucus.

51. To obtain an adolescent's health information, the nurse should: 1. Interview the adolescent using direct questions. 2. Gather information during a casual conversation. 3. Interview the adolescent only in the presence of the parents. 4. Gather information only from the parents.

2. Gather information during a casual conversation.

33. A 5-year-old boy has always been one of the shortest children in class. His mother tells the school nurse that her husband is 6 tall and she is 57. What should the nurse tell the child's mother? 1. He is expected to grow about 3 inches every year from ages 6 to 9 years. 2. He is expected to grow about 2 inches every year from ages 6 to 9 years. 3. He should be seen by an endocrinologist for growth-hormone injections. 4. His growth should be re-evaluated when he is 7 years old.

2. He is expected to grow about 2 inches every year from ages 6 to 9 years.

17. A nurse is administering cyclophosphamide (Cytoxan) to a child with leukemia. Which of the following actions by the nurse would be appropriate? 1. Monitoring serum potassium levels. 2. Checking for hematuria. 3. Obtaining daily weights. 4. Getting neurological checks every 4 hours.

2. Hemorrhagic cystitis is a major side effect of cyclophosphamide (Cytoxan); by checking the urine for blood, appro priate interventions can be made.

54. The nurse is caring for a 4-month-old infant who was diagnosed with a neuroblastoma. The nurse knows that this particular child's prognosis is: 1. Excellent, as a neuroblastoma is always cured. 2. Excellent, as infants with a neuroblastoma have the best prognosis. 3. Poor, as infants with a neuroblastoma rarely survive. 4. Variable, depending on where the site of origin is.

2. Infants younger than 1 year have the best prognosis.

14. The parent of a child diagnosed with osteomyelitis asks how the child acquired the illness. What is the nurse's best response? 1. "Direct inoculation of the bone from stepping barefoot on a sharp stick." 2. "An infection from a scratched mosquito bite carried the infection through the bloodstream to the bone." 3. "The blood supply to the bone was disrupted because of the child's diabetes." 4. "An infection of the upper respiratory tract."

2. Infection through the bloodstream is the most likely cause of osteomyelitis in a child.

29. The emergency room nurse is caring for an unconscious 6-year-old girl who has had a severe closed-head injury and notes the following changes in her vital signs. Her heart rate has dropped from 120 to 55, her blood pressure has increased from 110/44 to 195/62, and her respirations are becoming more irregular. After calling the physi cian, which of the following should the nurse expect to do? 1. Call for additional help, and prepare to administer mannitol. 2. Continue to monitor the patient's vital signs, and prepare to administer a bolus of isotonic fluids. 3. Call for additional help, and prepare to administer an antihypertensive. 4. Continue to monitor the patient, and administer supplemental oxygen.

29. 1. Cushing triad is characterized by a decrease in heart rate, an increase in blood pressure, and changes in respira tions. The triad is associated with severely increased ICP. Mannitol is an osmotic diuretic that helps decrease the increased ICP.

29. A 13-year-old just returned from surgery for scoliosis. What nursing interventions are appropriate in the first 24 hours? Select all that apply. 1. Assess for pain. 2. Logroll to change positions. 3. Get the teen to the bathroom 12 to 24 hours after surgery. 4. Check neurological status. 5. Monitor blood pressure.

29. 1. General postoperative nursing inter ventions include assessing for pain. 2. Specific to scoliosis surgery, logrolling is the means of changing positions. 4. It is essential to check neurological status in a patient who just had scoliosis surgery. 5. General postoperative nursing inter ventions include assessing vital signs.

26. Which medication is most important to have available in all clinics and offices if immunizations are administered? 1. Benadryl (diphenhydramine) injection. 2. Epinephrine 1:10,000 injection. 3. Epinephrine 1:1000 injection. 4. Benadryl (diphenhydramine) liquid.

3

28. Which of the following would be the priority intervention for the newborn of a mother positive for hepatitis antigen? 1. The newborn should be given the first dose of hepatitis B vaccine by 2 months of age. 2. The newborn should receive the hepatitis B vaccine and hepatitis B immune globulin within 12 hours of birth. 3. The newborn should receive the hepatitis B vaccine and hepatitis B immune globulin within 24 hours of birth. 4. The newborn should receive hepatitis B immune globulin only within 12 hours of birth.

3

29. Which instruction would be of highest priority for the mother of an infant receiving his first oral rotavirus vaccine? 1. "Call the physician if he develops fever or cough." 2. "Call the physician if he develops fever, redness, or swelling at the injection site." 3. "Call the physician if he develops a bloody stool or diarrhea." 4. "Call the physician if he develops constipation and irritability."

3

31. When discharging a newborn, which injury prevention instruction would be of highest priority to tell the parents? 1. "Place safety locks on all medicine cabinets and household cleaning supplies." 2. "Transport the infant in the front seat when driving alone so you can see the baby." 3. "Never leave the baby unattended on a raised, unguarded area." 4. "Place safety guards in front of any heating appliance, stove, fireplace, or radiator."

3

33. Which would be the most appropriate discharge instructions for a child with a right wrist sprain 3 hours ago? 1. "You should rest, elevate the wrist above the heart, apply heat wrapped in a towel, and use the sling when walking." 2. "You can use the wrist, but stop if it hurts; elevate the wrist when not in use, and use the sling when walking." 3. "You should rest, apply ice wrapped in a towel, elevate the wrist above the heart, and use the sling when walking." 4. "You do not have to take any special precautions; do not use any movements that cause pain, and apply alternate heat and ice, each wrapped in a towel.

3

40. Which would be the best response to the mother of a 13-year-old boy who continues to ask to ride his 16-year-old cousin's all-terrain vehicle? 1. Emphasize that if he wears safety apparel and has adult supervision, it is fine. 2. Explain to the mother that he is developing increased physical skills and, if he wears safety apparel and shows maturity, it should be fine. 3. Discuss that all-terrain vehicles are not recommended for those younger than 16 years of age. 4. Discuss with the mother that this is a stage where the child is seeking independence and should be allowed to participate in new physical activities.

3

3. The nurse is preparing to assess a 6-year-old male with altered consciousness in the PICU. His parents ask if they can stay during his morning assessment. Select the nurse's best response. 1. "Your child is more likely to answer questions and cooperate with any procedures if you are not present." 2. "Most children feel more at ease when parents are present, so you are more than welcome to stay at the bedside." 3. "It is our policy to ask parents to leave during the first assessment of the shift." 4. "Many children fear that their parents will be disappointed if they do not do well with procedures, so we recommend that no parents be present at this time."

2. Parents should be encouraged to remain with their child for mutual comfort.

1. After sustaining a closed head injury, a child is admitted to the pediatric intensive care unit. The child is ordered to receive phenytoin (Dilantin) 100 mg intravenously for seizure prophylaxis. Which of the following interventions should be done when ad ministrating this drug? 1. Mix it in dextrose 5% in water and give over 1 hour. 2. Administer as an intravenous bolus using a syringe pump no faster than 50 mg/min. 3. An inline filter should not be used. 4. Monitor temperature prior to and after administration.

2. Phenytoin (Dilantin) should be given slowly (1-2 mg/kg/min) via pump. Rapid infusion may cause hypotension, ar rhythmias, and circulatory collapse.

46. Which patient is able to give informed consent for a surgical procedure in many U.S. states? 1. 13-year-old abused male. 2. 15-year-old pregnant female. 3. 16-year-old cancer patient. 4. 17-year-old college freshman.

2. Pregnant minors are considered eman cipated minors in many U.S. states and are able to give informed consent.

59. Which of the following is an important nursing intervention to teach about photo sensitivity to the parents of a child with SLE? 1. Regular clothing is appropriate for sun exposure. 2. Sunscreen application is necessary for protection. 3. Teenage patients cannot participate in outdoor sports. 4. Water is important to reduce sensitivity.

2. Sunscreen helps reduce accelerated burning due to sensitivity.

54. A child who weighs 20 kg is to receive 8 g of gamma globulin for the treatment of idiopathic thrombocytopenia purpura. The order is to give the gamma globulin over 12 hours. The concentration is 8 g in 300 mL of normal saline. How many milliliters per hour will the child receive? 1. 12 mL/hr. 2. 25 mL/hr. 3. 50 mL/hr. 4. 40 mL/hr.

2. Take the total volume (300 mL) and divide it by the number of hours (12 hours) 300 mL 12 hours 25 mL/hr

50. When making assignments for the oncoming shift on the pediatric unit, the charge nurse assigns a float RN from another unit to care for an infant with complex needs. What is the legal responsibility of the charge nurse in this situation? 1. Assurance of scope of practice. 2. Duty to orient, educate, and evaluate. 3. Patient's rights and responsibilities. 4. Determination of nurse/patient ratios.

2. The charge nurse has a duty to orient the float nurse to the pediatric unit, educate the nurse on any unfamiliar procedures, and evaluate the float nurse's competency to provide care.

31. Which reaction would a nurse expect when giving a preschooler immunizations? 1. The child remains silent and still. 2. The child cries and tells the nurse that it hurts. 3. The child tries to stall the nurse. 4. The child remains still while telling the nurse that she is hurting him.

2. The child cries and tells the nurse that it hurts. The common response of a 5-year-old is to cry and protest during an immunization. School-age children are most likely to try to stall the nurse. Teens usually remain still, and they may calmly tell the nurse that they are feeling pain during the injection.

58. A 5-year-old female has been diagnosed with a midline brain tumor. In addition to showing signs of increased ICP, she has been voiding large amounts of very dilute urine. Which of the following medications does the nurse expect to administer? 1. Mannitol. 2. Vasopressin. 3. Lasix. 4. Dopamine.

2. The child is experiencing diabetes insipidus, a common occurrence in children with midline brain tumors. Vasopressin is a hormone that is used to help the body retain water.

46. The nurse is working in the pediatric developmental clinic. Which of the children requires continued follow-up because of behaviors suspicious of CP? 1. A 1-month-old who demonstrates the startle reflex when a loud noise is heard. 2. A 6-month-old who always reaches for toys with the right hand. 3. A 14-month-old who has not begun to walk. 4. A 2-year-old who has not yet achieved bladder control during waking hours.

2. The clinical characteristic of hemiple gia can be manifested by the early pref erence of one hand. This may be an early sign of CP.

40. A 15-kg child is started on amoxicillin/clavulanate potassium (Augmentin) for treat ment of cellulitis. The dose is 40 mg/kg/day, given three times a day. The nurse has a bottle of Augmentin that indicates there is 200 mg/5 mL. How many milliliters must the nurse draw up for each dose? 1. 2.5 mL. 2. 5 mL. 3. 15 mL. 4. 20 mL.

2. The dose is calculated by multiplying the weight by the milligrams. That re sult is divided by the three doses. The milligrams are then used to determine the milliliters based on the concentra tion of the drug. 40 mg × 15 kg = 600 mg/3 doses = 200 mg/dose The concentration of the drug is 200 mg/5 mL, so the answer is 5 mL

9. When counseling the parents of a child with OI, the nurse should do which of the following? Select all that apply. 1. Discourage future children, because the condition is inherited. 2. Provide education about the child's physical limitations. 3. Give the parents a letter signed by the primary care provider explaining OI. 4. Provide information on contacting the Osteogenesis Imperfecta Foundation. 5. Encourage the parents to treat the child like their other children. 6. Encourage use of calcium to decrease risk of fractures.

2. The nurse should provide education about the child's physical limitations so that physical therapy and appropriate activity can be encouraged. 3. OI is frequently confused with child abuse. Carrying a letter stating that the child has OI and what that condition looks like can ease the stressors of an emergency department visit. 4. Osteogenesis Imperfecta Foundation is an organization that can provide infor mation and support for a family with a child with the condition.

26. An 8-year-old child is attending a Cub Scout camp picnic. He has a history of epilepsy and has had generalized seizures since the age of 3. The child falls to the ground and has a generalized seizure. Which of the following is the best action for the nurse to take during the child's seizure? 1. Administer the child's rescue dose of oral valium. 2. Loosen the child's clothing, and call for help. 3. Place an oral tongue blade in the child's mouth to prevent aspiration. 4. Carry the child to the infirmary to call 911 and start an intravenous line.

2. The nurse should remain with the child and observe the seizure. The child should be protected from his environ ment, and clothing should be loosened.

25. The nurse is providing discharge instructions to the parents of a 13-year-old girl who has been diagnosed with epilepsy. Her parents ask if there are any activities that she should avoid. Select the nurse's best response. 1. "She should avoid swimming, even with a friend." 2. "She should avoid being in a car at night." 3. "She should avoid any strenuous activities." 4. "She should not return to school right away as her peers will likely cause her to feel inadequate."

2. The rhythmic reflection of other car lights can trigger a seizure in some children.

42. A 6-month-old infant male has just been diagnosed with craniosynostosis. He is being evaluated for reconstructive surgery. The infant's father asks the nurse for more information about the surgery. Select the nurse's best response. 1. "The surgery is done for cosmetic reasons and is without many complications." 2. "The surgery is important to allow the brain to grow properly. Although most children do well, serious complications can occur, so your child will be closely observed in the intensive care unit." 3. "The surgery is important to allow the brain to grow properly. Most surgeons wait until the child is 3 years old to minimize potential complications." 4. "The surgery is mainly done for cosmetic reasons, and most surgeons wait until the child is 3 years old as the head has finished growing at that time."

2. The surgery is done to reconstruct the skull to allow the brain to grow prop erly. Because there are potential com plications associated with this surgery, such as increased ICP, the child is usually closely observed in the PICU.

12. A mother requests that her child receive the varicella vaccine at the 9-month well-child checkup. The nurse tells the mother that: 1. Children who are vaccinated will likely develop a mild case of the disease. 2. The vaccine cannot be given at that visit. 3. The vaccine will be administered after the physician examines the child. 4. A booster vaccination will be needed at 18 months of age.

2. The vaccine cannot be given at that visit. The nurse should not give the vaccine. The varicella vaccine is not usually administered prior to 1 year of age.

55. Chemotherapeutic agents such as methotrexate and cyclophosphamide are sometimes used to treat JRA. Why are these medications used? 1. Effective against cancer, like JRA. 2. Affect the immune system. 3. Are similar to NSAIDs. 4. Are absorbed into the synovial fluid.

2. These drugs affect the immune system to reduce its ability to attack itself, as in the case of JRA.

47. Which of the following situations would be considered failure to obtain informed consent? 1. Parents who speak Spanish receive information about their child's surgery from the surgeon using a telephone language-line Spanish translator. 2. Bilingual parents sign the consent form for a lumbar puncture. Later, they tell a Spanish-speaking nurse, "We do not understand why they are doing this test." 3. The physician addresses the benefits of a procedure with a child's parents but gets called away. Later, the physician returns to finish the discussion before parents sign the consent. 4. Only one parent of a child is present in the hospital to sign the surgical consent. The other parent is out of town on business.

2. This situation violates the principle of informed consent. Despite signing the written consent form, the parents state a lack of understanding of the procedure.

8. A pregnant mother is told her fetus has OI. Which of the following classifications of OI is lethal in utero and in infancy? 1. Type I. 2. Type II. 3. Type III. 4. Type IV.

2. Type II is lethal in utero and in infancy because of multiple fractures and defor mities and underdeveloped lungs.

38. The most common complication associated with myelomeningocele is: 1. Learning disability. 2. Urinary tract infection. 3. Hydrocephalus. 4. Decubitus ulcers and skin breakdown.

2. Urinary tract infections are the most common complication of myelomeningocele. Nearly all children with myelomeningocele have a neuro genic bladder that leads to incomplete emptying of the bladder and subse quent urinary tract infections. Fre quent catheterization also increases the risk of urinary tract infection.

58. A nurse is caring for a child with cystic fibrosis. Which of the following should the nurse include in teaching the parents about administrating pancreatic enzymes? 1. The enzymes may be chewed or swallowed. 2. The capsules may be opened and sprinkled over acidic food. 3. Give the same amount of the medicine with meals and snacks. 4. Store the enzymes in the refrigerator.

2. When administrating enzymes to in fants, the capsule may be opened and sprinkled over an acidic food, such as applesauce or mashed fruit.

21. The nurse is discussing a ketogenic diet with a family. The nurse knows that this diet is sometimes used with children who have had little success with anticonvulsant med ication. The diet that produces anticonvulsant effects from ketosis consists of: 1. High fat and low carbohydrates. 2. High fat and high carbohydrates. 3. Low fat and low carbohydrates. 4. Low fat and high carbohydrates.

21. 1. High fat and low carbohydrates are the components of the ketogenic diet.

21. A pediatric hospital nurse receives a telephone call from an individual who is the par ent of a child assigned to the nurse's care. Which action by the nurse is most correct? 1. Verify the privacy code assigned to the child before giving any information. 2. Decline to give out information over the telephone because no ID can be shown. 3. Update the parent on the child's condition, as no family members are in the room. 4. Take the parent's name and telephone number and give it to the other parent when visiting.

21. 1. The nurse is correct to verify the privacy code, which is given only to the child's legal guardians, before relating medical information over the telephone.

22. A nursing student records notes about a pediatric patient's condition in preparation for a clinical experience. Which of the following information is considered individu ally identifiable health-care information and cannot be attached to the notes? 1. Date of birth. 2. Medical diagnosis. 3. Nursing diagnosis. 4. Diagnostic test results.

22. 1. Date of birth is information that can be used to identify an individual and should not be attached to students' notes.

22. A child comes to clinic with a pediculosis infestation. The nurse instructs the parent to do which of the following to treat the problem? 1. Apply lindane (Kwell) to the scalp, leave it in place for 4 minutes, and then add water. 2. Apply chlorhexidine (Hibiclens) to the scalp with sterile gloves. 3. Apply terbinafine (Lamisal) as a thin layer to the scalp twice a day. 4. Apply collagenase (Santyl) to the scalp with cotton applicator.

22. 1. Lindane (Kwell) is the drug of choice because it is well absorbed by the cen tral nervous system of the parasite (lice) and results in death.

23. If an employee has medical testing at a facility where that employee works, what is the appropriate way to access the test results? 1. Complete the authorization form and receive a copy of the results. 2. Get a fellow employee who works in that department to access the results. 3. Call a friend who has access to the records and ask for a copy of the test results. 4. Check the computer system for the test results.

23. 1. Employees who seek access to their own medical records must follow the same procedures as any patient treate by the health-care facility.

25. During a home visit to an 8-month-old infant in congestive heart failure on digoxin (Lanoxin), the nurse obtains assessment information. Which assessment indicates that the nurse needs to consult the physician? 1. The infant's apical pulse is 70 at rest. 2. After crying, the infant's heart rate is 170. 3. Respirations are 40 per minute at rest. 4. Capillary refill is <3 sec.

25. 1. Bradycardia (heart rate below 90-110 beats/min in infants) is a common sign of digoxin toxicity. The provider should be notified.

25. The nurse caring for a child with osteomyelitis assesses poor appetite. Which of the following interventions are most appropriate for this child? Select all that apply. 1. Offer high-calorie liquids. 2. Offer favorite foods. 3. Do not worry about intake, as appetite loss is expected. 4. Suggest intravenous removal to encourage oral intake. 5. Decrease pain medication that might cause nausea. 6. Offer frequent small meals.

25. 1. High-calorie liquids are sometimes re ceived better when the child has a poor appetite. 2. Offering favorite foods can sometimes tempt the child to eat, even with a poor appetite. 6. Small, frequent meals might increase daily caloric intake.

26. The nurse on the pediatric floor is receiving a child with the possible diagnosis of septic arthritis of the elbow. Which of the following would the nurse expect on assessment? Select all that apply. 1. Resistance to bending the elbow. 2. Nausea and vomiting. 3. Fever. 4. Bruising of the elbow. 5. Swelling of the elbow. 6. A history of nursemaid's elbow as a toddler.

26. 1. Infection of the elbow joint can cause pain that leads to protecting the joint and resisting movement. 2. Infection of the elbow may cause generalized nausea and vomiting. 3. Infection of the elbow frequently causes fever. 5. Septic arthritis can cause swelling of the joint.

28. When caring for an infant admitted for pyloric stenosis surgery, which tasks would be appropriate for the RN to delegate to a nursing assistant? Select all that apply. 1. Physical assessment on admission. 2. Vital signs every 4 hours. 3. Discharge teaching for parents. 4. Bed, bath, and change of linens. 5. Daily weights.

28. 2, 4, 5. 1. RNs should perform all admission assessments. By law, this function cannot be delegated. 2. Determination of vital signs is within the scope of practice of the nursing assistant and is an appropriate task for the RN to delegate. 3. RNs perform all patient teaching. Therefore, it is not appropriate to delegate this task to a nursing assistant. 4. ADLs are appropriate for the RN to delegate to a nursing assistant. 5. Daily weights are appropriate for the RN to delegate to a nursing assistant.

29. The nurse is managing care of a school-age child with new-onset type I diabetes. Which tasks must be performed only by the RN and cannot be delegated to an LPN or nursing assistant? Select all that apply. 1. Teaching parents how to give subcutaneous injections of insulin. 2. Performing blood glucose monitoring before meals and bedtime. 3. Evaluating the child's response to insulin doses. 4. Determining the educational goals for the day. 5. Teaching the child signs for hypoglycemia and hyperglycemia.

29. 1, 3, 4, 5. 1. All education must be completed by the RN and cannot be delegated. This includes insulin administration. 2. Blood glucose monitoring is within the scope of practice of an LPN and nursing assistant and can be delegated by the RN. 3. Evaluation of a patient's responses to treatments and medications must be completed by the RN and cannot be delegated. 4. Determining the plan of care, includ ing educational goals, must be com pleted by the RN. This task is not within the scope of practice of the LPN or nursing assistant. 5. Teaching the child about hypoglycemia and hyperglycemia is the responsibility of the RN and cannot be delegated.

18. The parent of a 3-week-old states that the infant was recasted this morning for clubfoot and has been crying for the past hour. Which of the following interventions should the nurse suggest the parent do first? 1. Give pain medication. 2. Reposition the infant in the crib. 3. Check the neurocirculatory status of the foot. 4. Use a cool blow-dryer to blow into the cast to control itching.

3. Checking the neurocirculatory status of the foot is the highest priority.

27. A 12-year-old diagnosed with scoliosis is to wear a brace for 23 hours a day. What is the most likely reason the child will not wear it for that duration? 1. Pain from the brace. 2. Difficulty in putting the brace on. 3. Self-consciousness about appearance. 4. Not understanding what the brace is for.

3. Children this age are very conscious of their appearance and of fitting in with their peers, so they might be very resistant to wearing a brace.

40. During a clinic visit, a child's mother tells the nurse, "I'm afraid of what my husband will do." Following an ethical decision-making process, the nurse's first step is to: 1. Direct the mother to a center for abused women. 2. Provide the phone number of the domestic violence hotline. 3. Clarify what the mother means by her statement. 4. Ask the mother, "Why does your husband feel this way?"

3. Clearly identifying the problem, as sug gested in this answer, is the first step in the ethical decision-making process.

59. A child who has been diagnosed with enuresis has been started on oxybutynin (Ditropan). The nurse knows that common side effects of oxybutynin (Ditropan) are which of the following? 1. Increase in heart rate and blood pressure. 2. Sodium retention and edema. 3. Constipation and dry mouth. 4. Insomnia and hyperactivity.

3. Common side effects are constipation and dry mouth as the oxybutynin (Ditropan) has an atropine-like effect.

11. A seriously ill child is treated in the intensive care unit. Which aspect of care is the easiest for family members to evaluate and most important for consumer satisfaction? 1. The compliance with standards. 2. The efficiency of the medical equipment used in care. 3. The relationship with the nurse and other staff. 4. The accuracy of completion of medical orders.

3. Consumers always have the ability to evaluate the quality of the relationship with the person delivering the service.

28. The best method to explain a procedure to a hospitalized preschool-age child is to: 1. Show the child a pamphlet with pictures showing the procedure. 2. Have the 5-year-old next door tell the 4-year-old about the experience. 3. Demonstrate the procedure on a doll. 4. Show the child a video of the procedure

3. Demonstrate the procedure on a doll. A 4-year-old child understands in very concrete and simple terms. Therefore, medical play is an excellent method for helping to understand the procedure.

25. Ciprofloxacin (Cipro) 300 mg is ordered for a child with a urinary tract infection. The medication comes 250 mg/5 mL. The nurse has determined that the dosage prescribed is safe. How much of the medication will the nurse prepare to give to the child? 1. 1.2 mL. 2. 3 mL. 3. 6 mL. 4. 12 mL.

3. Desired over Available × Volume = amount to be given 300mg × 5 mL = 6 mL 250mg

52. Penicillin is given to a 2-year-old prior to dental work. The child weighs 22 lb. The order is for 25 mg/kg to be given 2 hours before the procedure. The penicillin comes in 250 mg/5 mL. How much of the medication will the nurse administer? 1. 2.5 mL. 2. 5 mL. 3. 10 mL. 4. 15 mL.

3. Determine the child's weight in kilo grams by dividing 44 lb by 2.2 (2.2 kg 1 lb). Then determine how many mil ligrams need to be given by multiplying the child's weight in kilograms by 25 mg. 44 lb 2.2 kg 22 kg 22 kg 25 mg 500 mg Use the formula: Desired over Available × Volume Desired dose 500 mg 5 mL 10 mL 250 mg

30. Assigning the right task to the right person is a principle of nursing delegation and assignment. Which of the following scenarios best meets this principle? 1. A 4-month-old with Down syndrome is assigned to a nurse whose own child died of heart disease due to Down syndrome 6 months ago. 2. A child with a central intravenous line that occluded on the previous shift is assigned to a new LPN. 3. A child newly diagnosed with acute leukemia is assigned to an experienced pediatric oncology nurse who floated to the general pediatric unit. 4. A child with new-onset type I diabetes is assigned to an RN who has four other complex-care patients.

3. Even though the pediatric oncology nurse has floated to the pediatric unit, this patient's care involves routine skills and knowledge used in the nurse's oncology practice. This makes this assignment the best example of "right task to right person."

41. A child with hives is prescribed diphenhydramine (Benadryl) 5 mg/kg per day in di vided doses over 6 hours. The child weighs 40 lb. How many milligrams should the nurse give with each dose? 1. 4.5 mg. 2. 11.45 mg. 3. 22.75 mg. 4. 50 mg.

3. First convert 40 pounds to kilograms by dividing the pounds by 2.2 kg (2.2 kg 1 lb) 40 2.2 18.18 kg Multiply 5 mg by the weight (18.18) gives the amount of milligrams for 24 hours 18.18 kg 5 mg 90.9 mg/day Divide the total milligrams (90.9 mg) by 4, as that is the number of doses 90.9 mg 4 22.75 mg

10. The parents of an 8-year-old come to the clinic and ask the nurse if their child should receive growth hormone to boost short stature. Which is the nurse's best response? 1. "Growth hormone only works if the child has short bones." 2. "Can your child remember to take the pills every day?" 3. "Scientific evidence is required before growth hormone can be started in children." 4. "How tall do you think your child should be?'

3. Growth hormone is approved for use only in children to treat a documented lack of growth hormone.

14. The nurse is caring for a child with diabetes. There is an order for Humalog insulin for this child. The nurse knows that the onset of Humalog insulin is which of the following? 1. 1 to 2 hours. 2. 30 minutes to 1 hour. 3. 10 to 15 minutes. 4. 2 to 4 hours.

3. Humalog insulin is rapid-acting and has an onset of 10 to 15 minutes.

15. A 10-year-old with osteomyelitis has been on intravenous antibiotics for 48 hours. The child is allergic to amoxicillin. Vital signs are: T 101.8°F (38.8°C), BP 100/60, P 96, R 24. Which of the following is the primary reason for surgical treatment? 1. Young age. 2. Drug allergies. 3. Nonresponse to intravenous antibiotics. 4. Physician preference.

3. If a patient does not respond to an appropriate antibiotic within 48 hours, surgery may be indicated. This is the correct answer.

1. The nurse is assessing a 2-week-old for signs of DDH. The nurse should expect the infant to have which of the following? 1. Excessive hip abduction. 2. Femoral lengthening of an affected leg. 3. Asymmetry of gluteal and thigh folds. 4. Pain when lying prone.

3. In DDH, asymmetrical thigh and gluteal folds are frequently present.

48. The nurse is caring for a 2-month-old male infant who is at risk for CP due to extreme low birth weight and prematurity. There is a multidisciplinary team caring for him. His parents ask why there is a speech therapist involved in his care. Select the nurse's best response. 1. "Your child is likely to have speech problems because of his early birth. Involving the speech therapist at this point will ensure vocalization at a developmentally appropriate age." 2. "The speech therapist will help with tongue and jaw movements to assist with babbling." 3. "The speech therapist will help with tongue and jaw movements to assist with feeding." 4. "It is the hospital routine to involve as many members of the health-care team in your child's care so that we will know if he has any unmet needs."

3. It is important to involve speech therapy to strengthen tongue and jaw movements to assist with feeding. The infant who is at risk for CP may have weakened and uncoordinated tongue and jaw movements.

28. A nurse is caring for an adolescent female who is going to be treated with isotretinoin (Accutane) for acne. Which of the following does the nurse know is true about the medication? 1. The adolescent needs to apply a thin layer to the skin affected with acne twice a day. 2. The use of a tanning bed will be an added benefit to help dry up the acne. 3. The adolescent needs to have a pregnancy test done prior to starting treatment. 4. The adolescent needs to keep the lips moistened to prevent inflammation.

3. It is mandatory to have a pregnancy test done before starting treatment as spontaneous abortions and/or fetal ab normalities have been associated in pregnancy with the use of isotretinoin (Accutane).

18. A nurse is giving ifosfamide as chemotherapy for a child who has leukemia. Mixed in with the ifosfamide is mesna (Mesnex). The nurse knows that mesna is given for which of the following reasons? 1. Combination chemotherapy. 2. An antiarrhythmic. 3. Prevent hemorrhagic cystitis. 4. Increase absorption of the chemotherapy.

3. Mesna (Mesnex) is a detoxifying agen used as a protectant against hemor rhagic cystitis induced by ifosfamide and cyclophosphamide (Cytoxin).

30. The nurse is caring for a 2-year-old male in the PICU with a head injury. The child is comatose and unresponsive at this time. The parents ask if he needs pain medication. Select the nurse's best response. 1. "Pain medication is not necessary as he is unresponsive and cannot feel pain." 2. "Pain medication may interfere with his ability to respond and may mask any signs of improvement." 3. "Pain medication is necessary to promote comfort." 4. "Although pain medication is necessary for comfort, we use it cautiously as it increases the demand for oxygen."

3. Pain medication promotes comfort and ultimately decreases ICP.

3. A nurse is caring for an adolescent with diabetes. The nurse gives the adolescent NPH insulin at 0730. What time would the nurse most likely see signs and symptoms of hypoglycemia? 1. 0930 to 1030. 2. 1130 to 1430. 3. 1130 to 1930. 4. 1530 to 1930.

3. Peak time for NPH insulin is 4 to 12 hours.

43. Which is the best method of distraction for an 8-year-old who is having surgery later today and is NPO? 1. Use the telephone to call friends. 2. Watch television. 3. Play a board game. 4. Read the central line pamphlet he was given.

3. Play a board game. A board game is the optimal choice be- cause school-age children enjoy being engaged in an activity with others that will require some skill and challenge.

41. The nurse is caring for a 9-year-old with myelomeningocele who has just had surgery to release a tight ligament to the lower extremity. Which of the following does the nurse include in the child's postoperative plan of care? 1. Encourage the child to resume a regular diet, beginning slowly with bland foods that are easily digested, such as bananas. 2. Encourage the child to blow balloons to increase deep breathing and avoid postoperative pneumonia. 3. Assist the child to change positions to avoid skin breakdown. 4. Provide education on dietary requirements to prevent obesity and skin breakdown.

3. Preventing skin breakdown is important in the child with myelomeningocele, as pressure points are not felt easily.

16. The nurse is caring for a child with Reye syndrome in the PICU. At noon, the nurse notes that the child is comatose with sluggish pupils. When stimulated, the child demonstrates decerebrate posturing. At 2 p.m., the nurse notes that the child remains unchanged except that the child now demonstrates decorticate posturing when stimulated. The nurse concludes that: 1. The child's condition is worsening and progressing to a more advanced stage of Reye syndrome. 2. The child's condition is worsening, and the child may likely experience cardiac and respiratory failure. 3. The child's condition is improving and progressing to a less advanced stage of Reye syndrome. 4. The child's condition remains unchanged as posturing reflexes are similar.

3. Progressing from decerebrate to decorticate posturing usually indicates an improvement in the child's condition.

51. Administration of which of the following drugs is most important in treating an infant with transposition of the great vessels? 1. Digoxin (Lanoxin). 2. Antibiotics. 3. Prostaglandin E. 4. Diuretics.

3. Prostaglandin E is necessary to maintain patency of the patent ductus arteriosus and improve systemic arterial flow in children with inade quate intracardiac mixing.

40. Which nursing action is most appropriate to gain information about how a child is feeling? 1. Actively attempt to make friends with the child before asking about her feelings. 2. Ask the child's parents what feelings she has expressed in regard to her diagnosis. 3. Provide the child with some paper to draw a picture of how she is feeling. 4. Ask the child direct questions about how she is feeling.

3. Provide the child with some paper to draw a picture of how she is feeling. Often children will include much more detail about their feelings in drawings. They will often express things in pic- tures they are unable to verbalize.

48. Which technique should the nurse suggest to the mother of an 8-year-old who does not want to complete her chores? 1. Grounding. 2. Time-out. 3. Reward system. 4. Spanking.

3. Reward system. School-age children usually respond very well to a reward system and often enjoy the rewards so much that they will continue chores without continual reminders.

16. The nurse expects the blood culture report of an 8-year-old with septic arthritis to grow which causative organism? 1. Streptococcus pneumoniae. 2. Escherichia coli. 3. Staphylococcus aureus. 4. Neisseria gonorrhoeae.

3. S. aureus is a common organism found on the skin and is frequently the cause of septic arthritis.

48. A child is diagnosed with stage IV rhabdomyosarcoma, and the parent asks what that means. The nurse provides which of the following explanations? 1. The tumor is limited to the organ site. 2. There is regional disease from the organ involved. 3. There is distant metastatic disease. 4. The disease is limited to the lymph nodes.

3. Stage IV disease means there is distant metastatic disease.

53. Which of the following would the nurse teach the adolescent is a complication of corticosteroids used in the treatment of JRA? 1. Fat loss. 2. Adrenal stimulation. 3. Immune suppression. 4. Hypoglycemia.

3. Steroids cause immune suppression, which is the reason behind its use in JRA; it reduces the body's attack on itself.

47. The nurse is explaining about rhabdomyosarcoma cancer to an adolescent. From which of the following muscles does the cancer arise? 1. Skeletal. 2. Cardiac. 3. Striated. 4. Connective.

3. Striated muscle is in many organs and sites of the body, thus leading to the multiple sites of the disease.

35. A child has been started on sulfamethoxazole (Septra) for treatment of a urinary tract infection. How should the medication be given? 1. At breakfast and dinner. 2. With a snack. 3. With water. 4. With a cola beverage.

3. Sulfamethoxazole (Septra) should be administered with a full glass of water on an empty stomach. If nausea and vomiting occur, giving the drug with food may decrease gastric distress.

47. The nurse is caring for a 13-month-old with meningitis. The child has experienced increased ICP and multiple seizures. The child's parents ask if the child is likely to develop CP. Select the nurse's best response. 1. "When your daughter is stable, she'll undergo computed tomography and magnetic resolution imaging. The physicians will be able to let you know if she has CP." 2. "Most children do not develop CP at this late age." 3. "Your child will be closely monitored after discharge, and a developmental specialist will be able to make the diagnosis." 4. "Most children who have had complications of meningitis develop some amount of CP."

3. The child will be given a chance to recover and will be monitored closely before a diagnosis is made.

26. A nurse is caring for a child with congenital heart disease who is being treated with digoxin (Lanoxin). The nurse knows that which of the following needs to be included in the family's discharge teaching? 1. Make sure the medication is taken with food. 2. Repeat the dose of the medication if the child should vomit. 3. Take the child's pulse prior to administrating the medication. 4. Weigh the child daily.

3. The child's pulse should be monitored before each dose. The dose should be withheld according to the physician's parameters.

12. A child who has been diagnosed with hypothyroidism is being started on levothyrox ine (Synthroid). Which of the following should be included in the nurse's teaching plan about this medication? 1. The child will have more energy the next day after starting the medication. 2. Optimum effectiveness of the medication may not occur for several weeks. 3. The medication should be taken once a day at any time. 4. The medication should be taken with milk.

3. The drug works best when taken on an empty stomach; the patient should se lect a time each day when the stomach is empty. In children, just prior to bed may be the best time, as most children do not eat prior to bedtime.

43. A 6-month-old male has been diagnosed with positional brachycephaly. The nurse is providing teaching about the use of a helmet for his therapy. Which of the following statements indicate that his parents understand the education? 1. "We will keep the helmet on him when he is awake and remove it only for bathing and sleeping." 2. "He will start wearing the helmet when he is closer to 12 months, as he will be more upright and mobile." 3. "He will wear the helmet 23 hours every day." 4. "Most children need to wear the helmet for 6 to 12 months."

3. The helmet is worn 23 hours every day and removed only for bathing.

20. The nurse receives a telephone call from a staff member who works on another unit. The member is inquiring about the test results of a friend's child, who is hospitalized on the nurse's unit. Which response is appropriate? 1. Summarize the test results as they are within the normal range. 2. Move to a private phone to prevent being overheard before sharing the information. 3. Decline to give out information. 4. Direct the staff member to the test results in the hospital electronic medical record.

3. The pediatric nurse cannot legally share this information, but the parent can choose to do so. This response does not violate the patient/family right to privacy.

44. If a staff member has the right to access the computer system, the member has a right to view which of the following health records? 1. Personal health records. 2. Health records of immediate family members. 3. Health records of patients assigned to their care. 4. Health records of patients' family members.

3. The right to access information is based on the need to know this infor mation to provide patient care. Staff must have access to records of assigned patients in order to provide care.

60. A 9-year-old patient on the pediatric unit is immobilized in a spica cast. When the parents are absent, the child presses the call light constantly. Which action would be most appropriate to meet the child's needs? 1. Consult with the charge nurse to obtain a 24-hour sitter for the child. 2. Speak with the parents and ask them not to leave the bedside. 3. Refer to the child life-worker for bedside play activities. 4. Obtain a social service referral to meet emotional needs.

3. The role of the child life-worker is to help hospitalized children meet normal developmental needs. Referring to them is the most appropriate action.

56. The nurse is caring for a 3-year-old with neuroblastoma. The child's parents ask the nurse what the typical signs and symptoms are at first. Select the nurse's best answer. 1. "Most children complain of abdominal fullness and difficulty urinating." 2. "Many children in the early stages of a neuroblastoma have joint pain and walk with a limp." 3. "The signs and symptoms vary depending on where the tumor is located, but typical symptoms include weight loss, abdominal distention, and fatigue." 4. "The signs and symptoms are fairly consistent regardless of the location of the tumor. They include fatigue, hunger, weight gain, and abdominal fullness."

3. The signs and symptoms vary depending on where the tumor is located, but typical symptoms include weight loss, abdominal distention, and fatigue.

14. The nurse is caring for a 1-year-old female who has just been diagnosed with viral encephalitis. The parents ask if their child will be admitted. Select the nurse's best response. 1. "Your child will likely be sent home because encephalitis is usually caused by a virus and not bacteria." 2. "Your child will likely be admitted to the pediatric floor for intravenous antibiotics and observation." 3. "Your child will likely be admitted to the PICU for close monitoring and observation." 4. "Your child will likely be sent home because she is only 1 year old. We tend to see fewer complications and a shorter disease process in the younger child."

3. The young child with encephalitis should be admitted to a PICU where close observation and monitoring are available. The child should be ob served for signs of increased ICP and for cardiac and respiratory compromise.

38. Which of the following questions represents an ethical issue in nursing practice that cannot be resolved through research? 1. How does the incidence of medication errors on the pediatric unit compare with the incidence of errors on the neonatal unit? 2. Does the use of local anesthesia during circumcision make a difference in infants' pain scores, as measured on the face, legs, activity, cry, consolability (FLACC) scale? 3. Is the emergency room nurse obligated to report suspicion of child abuse if signs of abuse are noted in the assessment? 4. Which method of irrigating central venous lines results in less line obstruction and infection?

3. This is a legal/ethical issue that is guided by state mandates and the nurse's values of altruism and justice.

59. Which situation should be referred by the pediatric nursing staff to the nurse manager? 1. Several staff members plan to get together to celebrate the end of a successful year working on the unit. 2. Staff members complain about the cafeteria food and want a broader menu. 3. Patients with occluded saline locks are repeatedly being transferred from the emergency department to the pediatric unit. 4. Nursing staff members who have joined the union are in disagreement with union policies.

3. This is a patient care issue that will take some investigation and correction on a unit-to-unit level. Pediatric unit and emergency department managers should be involved. This situation is most appropriate to refer to nurse managers.

2. The parent of a child who is being discharged from the clinic wants to know if there is a difference between Advil and ibuprofen, saying "I can buy ibuprofen over the counter at a cheaper price than the Advil that was ordered." What is the nurse's best response? 1. "Advil and ibuprofen are two different drugs with similar effects." 2. "There is no difference between the two medications, so you should use whichever one is cheaper." 3. "Similarities exist between the drugs, but you need to consult the physician about the specific order." 4. "Ibuprofen is usually cheaper, so you should use it."

3. This response answers the question and tells the parent the physician is the only one who can change a name brand to a generic drug.

36. The nurse is caring for a neonate who has just been diagnosed with a meningocele. The parents ask what to expect. Which of the following is the nurse's best response? 1. "After initial surgery to close the defect, most children experience no neurological dysfunction." 2. "Surgery to close the sac will be postponed until the infant has grown and has enough skin to form a graft." 3. "After the initial surgery to close the defect, the child will likely have motor and sensory deficits." 4. "After the initial surgery to close the defect, the child will likely have future problems with urinary and bowel continence."

36. 1. Because a meningocele does not con tain any nerve endings, most children experience no neurological problems after surgical correction.

36. An adolescent is received in the pediatric intensive care unit after scoliosis surgery. Using Maslow's hierarchy of needs as a guide, which problem takes priority? 1. Hypotension related to analgesia. 2. Fear of being left alone by parents. 3. Frustration with postoperative immobility. 4. Concern with the extensive skin incision.

36. 1. Physiological needs take priority over other needs; therefore, hypotension is the priority need.

39. Shortly before a child's elective surgery, the parent tells the nurse, "I am having sec ond thoughts about my child undergoing this surgery." The nurse respects the par ent's concern and calls the surgeon. What ethical/moral principle is represented by this situation? 1. Autonomy. 2. Equality. 3. Fidelity. 4. Justice.

39. 1. Autonomy is the right to make one's own decisions; in this case, the right of the parent to make decisions about the child's surgery. As legal guardian, the parent has the right to choose or not choose for the child to undergo an elective procedure.

39. A 6-month-old is prescribed 2.5% hydrocortisone for topical treatment of eczema. The nurse instructs the parent not to use the cream for more than a week. What is the primary reason for this instruction? 1. Adverse effects, such as skin atrophy and fragility, can occur with long-term treatment. 2. If, after a week there is no improvement, then a stronger dose is required. 3. The drug loses its efficacy after prolonged use 4. If no improvement is seen after a week, an antibiotic should be prescribed.

39. 1. Hydrocortisone cream should be used for brief periods because it can thin the skin and cause skin breakdown.

16. Which statement by the parent of a newborn male diagnosed with galactosemia demonstrates successful teaching? 1. "This is a rare disorder that usually does not affect future children." 2. "Our newborn looks normal; he may not have galactosemia." 3. "Our newborn may need to take penicillin and other medications to prevent infection." 4. "Penicillin and other drugs that contain lactose as fillers need to be avoided."

4

22. What would the most therapeutic response if the mother of a 6-month-old female tells the nurse she does not want her infant to have the DTaP vaccine because the infant had localized redness the last time she received the vaccine? 1. "I will let the physician know, and we will not administer the DTaP vaccination today." 2. "Every child has that allergic reaction, and your child will still get the DTaP today." 3. "I will let the physician know that you refuse further immunizations for your daughter." 4. "Would you mind if we discussed your concerns?"

4

27. Which is the nurse's best response when the mother of a 2-month-old who is going to get the IPV tells the nurse the older brother is immunocompromised? 1. "Your baby should not be immunized because your immunocompromised son could develop polio." 2. "Your baby should receive the oral poliovirus vaccine instead so your immunocompromised son does not get sick." 3. "You should separate your 2-month-old child from the immunocompromised son for 7 to 14 days after the IPV." 4. "Your baby can be immunized with the IPV; he will not be contagious."

4

3. The nurse is assessing a 3-week-old with suspected bacterial meningitis. Isolation and respiratory precautions have already been initiated. Which clinical assessment by the nurse would warrant immediate intervention? 1. The neonate is irritable. 2. The neonate has a rectal temperature of 100.6° F (38.1°C). 3. The neonate is quieter than usual. 4. The neonate's respiratory rate is 24 breaths per minute.

4

47. Which would be the best response to a 10-year-old girl who asks if she can take acetaminophen daily if she gets aches and pains? 1. Tell her it is better not to take medication if she gets aches and pains; she should check with her mother before taking any medication. 2. Teach her that nonprescription drugs like acetaminophen can be a poisoning hazard if too many are taken; it is best for her to check with her mother. 3. Encourage her to keep a log of when she takes acetaminophen to try to establish what is causing her aches and pains. 4. Sometimes it is okay to take acetaminophen daily, but it depends on why she has aches and pains.

4

51. Which statement most accurately describes child abuse? 1. Intentional physical abuse and neglect. 2. Intentional and unintentional physical and emotional abuse and neglect. 3. Sexual abuse of children, usually by an adult. 4. Intentional physical, emotional, and sexual abuse and neglect.

4

18. The nurse is assessing the pain level in an infant who just had surgery. The infant's parent asks which vital sign changes are expected in a child experiencing pain. The nurse's best response is: 1. "We expect to see a child's heart rate decrease and respiratory rate increase." 2. "We expect to see a child's heart rate and blood pressure decrease." 3. "We expect to see a child's heart rate and blood pressure increase." 4. "We expect to see a child's heart rate increase and blood pressure decrease."

3. "We expect to see a child's heart rate and blood pressure increase."

35. A 3-year-old admitted to the hospital with croup has the following vital signs: heart rate 90, respiratory rate 44, blood pressure 100/52, and temperature 98.8°F (37.1°C). The parents ask the nurse if these vital signs are normal. The nurse's best response is: 1. "Your son's blood pressure is elevated, but the other vital signs are within the normal range.." 2. "Your son's temperature is elevated, but the other vital signs are within the normal range.." 3. "Your son's respiratory rate is elevated, but the other vital signs are within the normal range." 4. "Your son's heart rate is elevated, but the other vital signs are within the normal range."

3. "Your son's respiratory rate is elevated, but the other vital signs are within the normal range." A normal respiratory rate for a child from 3 to 6 years is 20 to 30 breaths per minute.

17. A 12-month-old boy weighed 8 lb 2 oz at birth. Understanding developmental mile- stones, what should the nurse caring for the child expect the current weight to be? 1. 16lb4oz 2. 20lb5oz 3. 24lb6oz 4. 32lb8oz

3. 24lb6oz Children should triple their birth weight by 12 months of age.

49. The nurse evaluates teaching as successful when the parent explains that an excisional biopsy is done to do which of the following? 1. To find metastatic disease. 2. To remove all metastatic disease. 3. To confirm the type of metastatic disease. 4. To treat metastatic disease.

3. A biopsy confirms the histology of the tumor.

17. The nurse is caring for a child with Reye syndrome stage III. The child is comatose with sluggish pupils. The child is currently maintaining his own respirations, and all vital signs are within normal range. In order to treat a common manifestation, what medication would the nurse expect to have readily available? 1. Lasix. 2. Insulin. 3. Glucose. 4. Morphine.

3. A common manifestation is hypo glycemia, which is treated with the ad ministration of intravenous glucose.

42. Organizational policies on the security of computer data mandate that system users keep which of the following pieces of user information confidential? 1. Name. 2. ID. 3. Password. 4. Credentials.

3. A password uniquely identifies a user and provides access to data appropriate to the user. System users must main tain confidentiality of their password.

12. A toddler is being admitted to the hospital with a diagnosis of bacterial meningitis. Select the best room assignment for the patient. 1. A semiprivate room with a roommate who also has bacterial meningitis. 2. A semiprivate room with a roommate who has bacterial meningitis but has received intravenous antibiotics for more than 24 hours. 3. A private room that is dark and quiet with minimal stimulation. 4. A private room that is bright and colorful and has developmentally appropriate activities available.

3. A quiet private room with minimal stimulation is ideal as the child with meningitis should be in a quiet envi ronment to avoid cerebral irritation.

59. The nurse is caring for a 30-month-old female receiving radiation therapy for a brain tumor. The parents ask if their child will likely have any learning disabilities in the future. Select the nurse's best answer. 1. "All children who receive radiation have some amount of learning disability. As long as they receive extra tutoring, they usually do well in school." 2. "Because your daughter is so young, she will likely do well and have no problems in the future." 3. "Response varies with each child, but younger children who receive radiation tend to have some amount of learning disability later in life." 4. "Response varies with each child, but younger children who receive radiation tend to have fewer problems later in life than older children."

3. Although variable, younger children tend to experience more learning difficulties than older children.

36. When instructing a family about care of an orthosis, the nurse should emphasize which of the following? 1. Clean the brace with diluted bleach. 2. Dry the brace over a heater or in the sun. 3. Clean the brace weekly with mild soap and water. 4. Return the brace to the orthopedic surgeon for cleaning.

3. An orthosis should be cleaned weekly with mild soap and water.

4. At lunch, several nurses are discussing how difficult it is to care for a 16-year-old who constantly complains of pain, unrelieved by morphine via a patient-controlled analge sia pump. One nurse comments, "The teen is addicted to drugs; what do you expect!" Which of the following is the best response to this statement? 1. "The teen should be moved to an adult unit, where the teen will be told what do." 2. "We should make sure that the teen has a nursing student to give the staff some relief." 3. "Perhaps we should call a team conference to review the pain complaints and treatment." 4. "I think we should speak with the physician about changing to non-narcotic pain medications."

3. As a patient advocate, the nurse is re sponsible for recognizing when current approaches are ineffective and working with other staff members to develop ef fective care. Planning a team conference is one way to brainstorm new approaches.

36. Which action is a developmentally appropriate method for eliciting a 4-year-old's cooperation in obtaining the blood pressure? 1. Have the child's parents help put on the blood pressure cuff. 2. Tell the child that if he sits still, the blood pressure machine will go quickly. 3. Ask the child if he feels a squeezing of his arm. 4. Tell the child that measuring the blood pressure will not hurt.

3. Ask the child if he feels a squeezing of his arm.

28. The emergency room nurse is caring for a 5-year-old child who fell off his bike and sustained a closed-head injury. The child is currently awake and alert, but his mother states that he "passed out" for approximately 2 minutes. The mother appears highly anxious and is very tearful. The child was not wearing a helmet. Which of the following statements is a priority for the nurse at this time? 1. "Was anyone else injured in the accident?" 2. "Tell me more about the accident." 3. "Did he vomit, have a seizure, or display any other behavior that was unusual when he woke up?" 4. "Why was he not wearing a helmet?"

3. Asking specific questions will give the nurse the information needed to deter mine the level of care for the child.

13. What should parents understand is one of the most common causes of injury and death for a 7-month-old infant? 1. Poisoning. 2. Child abuse. 3. Aspiration. 4. Dog bites.

3. Aspiration. Aspiration is a common cause of injury and death among children of this age. These children often find small objects lying on the floor and place them in their mouths. Older siblings are often responsible for leaving small objects around.

49. The nurse is giving morning medications to a 4-year-old female who has just had a surgical procedure to release her hamstrings. The child has a history of CP. When the nurse prepares to administer baclofen, the child's parents ask what the medication is for. Select the nurse's best response. 1. "It is a medication that will help decrease the pain from her surgery." 2. "It is a medication that will prevent her from having seizures." 3. "It is a medication that will help control her spasms." 4. "It is a medication that will help with bladder control."

3. Baclofen is given to help control the spasms associated with CP.

39. A 13-year-old with osteosarcoma is going to have an amputation of the affected limb. Which of the following is most important to discuss with a teenage patient? 1. Pain. 2. Spirituality. 3. Body image. 4. Lack of coping.

3. Body image is a developmental issue for adolescents and influences their ac ceptance of themselves and by peers.

53. The nurse is caring for a 5-year-old male with CP. His weight is in the fifth percentile, and he has been hospitalized for aspiration pneumonia. His parents are anxious and state that they do not want a G-tube put in. Which of the following would be the nurse's best response? 1. "A G-tube will help your son gain weight and reduce his risk for future hospitalizations due to pneumonia." 2. "G-tubes are very easy to care for and will make feeding time easier for your family." 3. "Are you concerned that you will not be able to care for his G-tube?" 4. "Tell me your thoughts about G-tubes."

4. An open-ended question will encour age family members to share what they know and potentially clear any misconceptions.

38. Each member of the family of a child diagnosed with pinworms is prescribed a single dose of pyrantel pamoate (Antiminth). Which of the following should the nurse teach the parents regarding administration of this drug? 1. Fever and rash are common adverse effects. 2. The medication kills the eggs in about 48 hours. 3. The drug may stain the feces red. 4. The dose may be repeated in 2 weeks.

4. As the first treatment kills the adult worms, a second treatment is done in 2 weeks to treat emerging parasites.

2. A 17-year-old is seen in the emergency department and diagnosed with a bowel ob struction. Despite the nurse's best attempt to explain the reason for a nasogastric tube, the adolescent refuses to let the nurse insert the tube. The parent's approach is also in effective. Which of the following approaches would be most appropriate for the nurse to do first? 1. Obtain an order for sedation, physically restrain the patient, and insert the tube. 2. Page the physician, and document the patient's refusal to accept the nasogastric tube. 3. Explain the AMA form to the adolescent and parent. 4. Notify the hospital's patient advocate to meet with the adolescent and parent.

4. Because of the patient's age, the teen should be treated as an adult. The nurse best promotes self-determination by making additional attempts to elicit any fears and concerns that are preventing effective care. The patient advocate may be able to gain this information.

2. An infant is in a Pavlik harness for treatment of DDH. While instructing the parents on preventing skin breakdown, the nurse should stress which of the following? 1. Put socks on over the foot pieces of the harness to help stabilize the harness. 2. Use lotions or powder on skin to prevent rubbing of straps. 3. Remove harness during diaper changes for ease of cleaning diaper area. 4. Check under the straps at least two to three times daily for red areas.

4. Checking under straps frequently is suggested to prevent skin breakdown.

10. The nurse is caring for a child who is being admitted with a diagnosis of meningitis. The child's plan of care includes the following: administration of intravenous antibi otics, administration of maintenance intravenous fluids, placement of a Foley catheter, and obtaining cultures of spinal fluid and blood. Select the procedure the nurse should do first. 1. Administration of intravenous antibiotics. 2. Administration of maintenance intravenous fluids. 3. Placement of a Foley catheter. 4. Send the spinal fluid and blood cultures to the laboratory.

4. Cultures of spinal fluid and blood should be obtained, followed by admin istration of intravenous antibiotics.

9. Nurses are aware that current trends affecting health-care consumers include which of the following? 1. Generally less informed about health-care issues than previous generations. 2. Comfortable with health insurance benefits, services, and conditions. 3. More trusting and less demanding of health-care organizations and staff. 4. Expect to be more involved in decisions about health-care options.

4. Data show that current health-care consumers expect to participate more in decisions about medical treatment.

27. The nurse is caring for a child who has sustained a closed-head injury. The nurse knows that brain damage can be caused by which of the following factors? 1. Increased perfusion to the brain and increased metabolic needs of the brain. 2. Decreased perfusion to the brain and decreased metabolic needs of the brain. 3. Increased perfusion to the brain and decreased metabolic needs of the brain. 4. Decreased perfusion of the brain and increased metabolic needs of the brain.

4. Decreased perfusion of the brain and increased metabolic needs of the brain.

31. An experienced pediatric nurse relocates to a new city and state. In the new position, the nurse questions which skills and tasks can be performed by unlicensed nursing personnel. To obtain answers to this question, which resource is considered primary? 1. Hospital policies and procedures manual. 2. State nurses' association. 3. Educational program for nursing assistants. 4. State nurse practice act.

4. Each state establishes legal guidelines for health-care professionals in various roles. These guidelines are in the state's nurse practice act, which is the ultimate legal document.

42. Which statement accurately describes how the nurse should approach an 11-year-old to do a physical assessment? 1. Ask the child's parents to remain in the room during the physical exam. 2. Auscultate the heart, lungs, and abdomen first. 3. Explain that the physical exam will not hurt. 4. Explain what the nurse will be doing in basic understandable terms.

4. Explain what the nurse will be doing in basic understandable terms. School-age children are capable of un- derstanding basic functions of the body and can understand what the nurse will be doing if explained in basic terms.

15. It is a busy day on the pediatric unit, and the nurses are short-staffed. A school-age child is scheduled to undergo an invasive radiological procedure. Which staff member would be most appropriate to meet the child's support needs by accompanying the child? 1. The staff nurse with the least busy assignment. 2. The staff nurse assigned to this child. 3. A volunteer grandparent who is on the unit. 4. The child life-worker for the unit.

4. Helping children cope with intrusive or frightening procedures is part of the job description of the child life-worker. This answer has the added advantage of maintaining staff nurse numbers on the unit.

13. The staff nurse is discharging an infant with a tracheostomy and gastrostomy to be cared for by parents at home. The case manager has arranged for home health supplies and services. Whose discussion would be of most direct benefit to ensure individualized care in the home? 1. Case manager and community pediatrician. 2. Case manager and home health company supervisor. 3. Staff RN and medical supply company. 4. Hospital staff nurse and home health nurse.

4. If the goal is individualized care, the best team members to discuss this child's care are the two staff members with the most direct care-taking responsibilities—the hospital staff nurse and the home health nurse.

64. The parent of a toddler asks the nurse to define greenstick fracture. Which of the following is the nurse's best explanation? 1. It is a fracture located in the growth plate of the bone. 2. Because children's bones are not fully developed, any fracture in a young child is called a greenstick fracture. 3. It is a fracture in which a complete break occurs in the bone, and small pieces of bone are broken off. 4. It is a fracture that does not go all the way through the bone.

4. It is a fracture that does not go all the way through the bone.

52. The nurse is teaching the parent of a child newly diagnosed with JRA. The nurse would evaluate the teaching as successful when the parent is able to say that the disorder is caused by: 1. The breakdown of osteoclasts in the joint space causing bone loss. 2. The loss of cartilage in the joints. 3. The buildup of calcium crystals in joint spaces. 4. The immune-stimulated inflammatory response in the joint.

4. JRA is caused by an immune response by the body on the joint spaces.

51. The parents of a 12-month-old with CP ask the nurse if they should teach their child sign language because he has not begun any vocalization yet. The nurse bases her response on which of the following? 1. Sign language may be a very beneficial way to help children with CP communicate. 2. Sign language may cause confusion and further delay verbalization. 3. Most children with CP will have great difficulty learning sign language. 4. Sign language may be beneficial, but it would be best to wait until the child is closer to the preschool age.

51. 1. Sign language may help the child with CP communicate and ultimately de crease frustration. Children with CP may have difficulty verbalizing because of weak tongue and jaw muscles. They may be able to have sufficient motor skills to communicate with their hands.

52. The parents of a 2-year-old with CP are learning how to feed their child and avoid aspiration. When reviewing the teaching plan, the nurse should question which of the following? 1. Place the food on the tip of the tongue, as the child will be less likely to choke. 2. Place the child in an upright position during feedings. 3. Feed the child soft and blended foods. 4. Feed the child slowly.

52. 1. The food should be placed far back in the mouth to avoid tongue thrust.

53. Assault and battery is considered an intentional tort that may leave the nurse liable for malpractice. Which situation might be considered assault and battery of a child? 1. The nurse sticks a hysterical infant five times in an attempt to start a PICC line, although the parent verbally refuses to allow the procedure to continue after the third try. 2. A 2-year-old screams while being restrained for a dressing change of a complex burn wound. 3. A nurse attempts to administer an oral antibiotic to a young child who then spits the entire dose of medication out on the sheet. 4. A postoperative school-age child refuses when told to get out of bed and walk in the hall four times a day.

53. 1. The parent's refusal to allow the nurse to continue constitutes a revocation of informed consent; therefore, the nurse is acting without permission. Addition ally, the nurse should consult the agency policy and procedure manual regarding the number of attempts allowed in this procedure.

54. The nurse makes an error by giving the wrong medication to a patient. An incident report is completed per hospital policy. What information should the nurse chart in the medical record? 1. Description of the specific occurrence and treatment given. 2. Completion of the incident report. 3. Date, time, and name of person completing the incident report. 4. Nothing.

54. 1. After an error occurs, the nurse should document what occurred and what was done to solve or treat the problem.

54. Which of the following would the nurse teach a patient when NSAIDs are prescribed for treating JRA? 1. Take with food. 2. Take on an empty stomach. 3. Blood levels are required for drug dosages. 4. Good oral hygiene is needed.

54. 1. NSAIDs can cause gastric bleeding with long-term use; food helps to reduce the exposure of the drug on the stomach lining.

55. Which of the following events should be reported to a risk management committee by documentation? 1. A nurse administered a double therapeutic dose of medication based on an incorrect physician order. 2. A patient's heparin lock became clotted between intermittent medication doses. 3. A toddler dislodged the intravenous catheter, resulting in an occluded intravenous line. 4. An uncooperative child spit out an undeterminable amount of oral medication despite the nurse's best effort.

55. 1. Administration of a medication dose outside the therapeutic range is an execution error and should be reported as a critical incident. The nurse is held responsible for administering the wrong dose, just as the physician is re sponsible for ordering the wrong dose.

55. The nurse is caring for a 6-year-old female with a neuroblastoma. The girl has metastasis to the bone marrow and has been diagnosed with pancytopenia. Which of the following should be included in her care? 1. Administration of red blood cells. 2. Limit school attendance to less than 4 hours daily. 3. Administration of Coumadin. 4. Encourage a diet high in fresh fruits and vegetables.

55. 1. Red blood cells will be needed to increase the red blood cell count.

56. A nurse is giving atropine for sinus bradycardia. The nurse knows that atropine is which of the following? 1. Anticholinergic. 2. Beta-adrenergic agonist. 3. Bronchodilator. 4. Sympathomimetic.

56. 1. Atropine is an anticholinergic drug. It blocks vagal impulses to the myo cardium and stimulates the cardioin hibitory center in the medulla. It increases heart rate and cardiac output.

56. One nursing diagnosis for JRA is impaired physical mobility. Select all nursing interventions that apply. 1. Give pain medication prior to ambulation. 2. Assist with range-of-motion activities. 3. Encourage the child to eat a high-fat diet. 4. Provide oxygen as necessary. 5. Use nonpharmacological methods, such as heat.

56. 1. Providing pain medication prior to ambulation helps decrease pain during ambulation. 2. Children with JRA need to do range of-motion exercises to prevent joint stiffness. 5. Using nonpharmacological methods such as heat helps with flexibility and pain.

56. Which of the following events represents a departure from safe practice as defined by risk management? 1. A nurse double-checks an insulin dose with a nursing assistant before administration to the patient. 2. The physician writes an illegible order, then draws a line through it, initials it, and prints the same information above the line. 3. The nurse repeats a verbal order back to the physician and asks for verification of accuracy. 4. A nurse asks another RN to double-check a dose of morphine before administer ing the drug via intravenous push.

56. 1. This answer is an example of a viola tion of the policy for administering a high-risk drug such as insulin. Policy dictates that another RN (not a nursing assistant) double-check the dose before administering.

57. The nurse is working in the pediatric cancer center caring for a group of children with brain tumors. Which of the following children would have likely experienced a delay in diagnosis? 1. A 3-month-old, as signs and symptoms would not have been readily apparent. 2. A 5-month-old, as signs and symptoms would not have been readily suspected. 3. A school-age child, as signs and symptoms could have been misinterpreted. 4. An adolescent, as signs and symptoms could have been ignored and denied.

57. 1. In infants, signs and symptoms may not be readily apparent as the open fontanel allows for expansion.

57. The nurse is teaching the parent of a child diagnosed with SLE. The nurse evaluates the teaching as effective when the parent states: 1. "The cause is unknown." 2. "There is no genetic involvement." 3. "Drugs are not a trigger for the illness." 4. "Antibodies improve disease outcome."

57. 1. SLE is a complex disease; there are many triggers, but how the disease develops is not known.

6. A case manager is called to arrange for medical equipment and medications at dis charge for a child with multiple social problems. Which problem is likely to have the greatest impact on discharge planning? 1. The child and family are homeless. 2. The child is not covered by insurance. 3. The family cannot pay for medications. 4. The child does not have a primary care provider.

6. 1. Homelessness is likely to have the great est impact on discharge planning be cause all other aspects of care revolve around this issue.

6. The nurse is preparing to give preoperative teaching to the parents of an infant with hydrocephalus. The nurse knows that the most common treatment for hydrocephalus includes the surgical placement of a shunt connecting which of the following? 1. The ventricle of the brain to the peritoneum. 2. The ventricle of the brain to the right atrium of the heart. 3. The ventricle of the brain to the lower esophagus. 4. The ventricle of the brain to the small intestine.

6. 1. The ventriculoperitoneal is the most common shunt used to treat hydrocephalus.

6. The nurse tells the parent that other conditions can be associated with congenital clubfoot? Select all that apply. 1. Myelomeningocele. 2. Cerebral palsy. 3. Diastrophic dwarfism. 4. Breech position in utero. 5. Prematurity. 6. Fetal alcohol syndrome.

6. 1. There is an association between myelomeningocele and congenital clubfoot. 2. There is an association between some forms of cerebral palsy and congenital clubfoot. 3. There is an association between dia strophic dwarfism and congenital clubfoot.

61. Which of the following has the potential to alter a child's level of consciousness? Select all that apply. 1. Metabolic disorders. 2. Trauma. 3. Hypoxic episode. 4. Dehydration. 5. Endocrine disorders.

61. 1, 2, 3, 4, 5. 1. Many metabolic disorders are associ ated with hypoglycemia. The hypo glycemic child experiences a decreased level of consciousness as the brain does not have stores of glucose. 2. Trauma can lead to generalized brain swelling with resultant increased ICP. 3. Hypoxemia leads to a decreased level of consciousness as the brain is intoler ant to the lack of oxygen. 4. Dehydration can lead to inadequate perfusion to the brain, which can result in a decreased level of consciousness. 5. Endocrine disorders often result in a decreased level of consciousness as they can lead to hypoglycemia, which is poorly tolerated by the brain.

61. A young child hospitalized with asthma is ready for discharge. A home nebulizer is ordered by the physician. In order to obtain the nebulizer, a referral should be made to which staff member? 1. Case manager. 2. Nurse manager. 3. Materials management staff. 4. Child life staff.

61. 1. The role of case manager includes ordering medical equipment for dis charge; therefore, this is an appropriate referral.

62. A 6-year-old involved in a bicycle crash has a spleen injury and a right tibia/fibula fracture that has been casted. Which of the following are early signs of compartment syndrome in this child? Select all that apply. 1. Edema. 2. Numbness. 3. Severe pain. 4. Weak pulse. 5. Anular rash

62. 1. Edema, numbness or tingling, and pain are early signs of compartment syndrome. 2. Edema, numbness or tingling, and pain are early signs of compartment syndrome. 3. Edema, numbness or tingling, and pain are early signs of compartment syndrome. 4. A weak pulse is a late sign of compart ment syndrome.

63. Nursing care of a child with a fractured extremity in whom there is suspected compartment syndrome includes which of the following? Select all that apply. 1. Assess pain. 2. Assess pulses. 3. Elevate extremity above the level of the heart. 4. Monitor capillary refill. 5. Provide pain medication as needed.

63. 1. In a recent fracture, the nurse should assess pain and provide treatment. 2. Pain, pallor, and weak or absent pulses are all signs of compartment syndrome. 4. Weak or absent pulse is a sign of compartment syndrome, so monitoring capillary refill is important in assessment. 5. Pain, pallor, and weak or absent pulses are signs of compartment syndrome. Pain should be treated.

65. A nurse is caring for a 5-year-old who has a fracture of the tibia involving the growth plate. When providing information to the parents, the nurse should indicate that: 1. This is a serious injury that could cause long-term growth issues. 2. The fracture usually heals within 6 weeks without further complications. 3. The child will never be able to play contact sports. 4. Fractures involving the growth plate require pain medication.

65. 1. Fractures of the growth plate are serious, as they can disrupt the growth process.

7. When planning a rehabilitative approach for a child with osteogenesis imperfecta, the nurse should prevent which of the following? Select all that apply. 1. Positional contractures and deformities. 2. Bone infection. 3. Muscle weakness. 4. Osteoporosis.

7. 1. A long-term goal in caring for a patient with OI is to prevent contractures and deformities. 3. A long-term goal in caring for a patient with OI is to prevent muscle weakness. 4. A long-term goal in caring for a patient with OI is to prevent osteoporosis.

7. A case manager coordinates outpatient care and referrals for patients in an HMO. Which outcome would indicate effective case management for a child with moderate to severe asthma? 1. The child attends school regularly with few absences for the year. 2. The child is able to tolerate a regular diet without constipation or diarrhea. 3. The family does not fill prescriptions for prophylactic inhaled steroids. 4. The child does not utilize the peak flowmeter when cared for at home.

7. 1. Regular school attendance is a positive outcome of case management because asthma can result in frequent absences from school.

A 4-year-old boy needs to use a metered-dose inhaler to treat asthma. He cannot coordinate the breathing to use it effectively. The nurse should suggest that he use a A. spacer. B. nebulizer. C. peak expiratory flow meter. D. trial of chest physiotherapy.

A

A child has a chronic cough, no retractions but diffuse wheezing during the expiratory phase of respiration. This suggests which of the following? A. Asthma B. Pneumonia C. Croup D. Foreign body aspiration

A

An infant has been diagnosed with staphylococcal pneumonia. Nursing care of the child with pneumonia includes which of the following? A. Administration of antibiotic B. Frequent complete assessment of the infant C. Round-the-clock administration of antitussive agents

A

An infant with a congenital heart defect is receiving palivizumab (Synagis). Based on the nurse's knowledge of medication, the purpose of this medication is to A. prevent respiratory syncytial virus (RSV) infection. B. make isolation of the infant with RSV unnecessary. C. prevent secondary bacterial infection. D .decrease toxicity of antiviral agents.

A

Which of the following interventions would be an appropriate nursing intervention when caring for a child with pneumonia? A. Encourage rest B Instruct the child to avoid lying on the affected side c. Administer analgesics D. Place the child in the Trendelenburg position

A

49. When explaining the procedure to the parent of a child undergoing surgery, the provider must give the following information as part of informed consent. Select all that apply. 1. Date and time the specific procedure will be performed. 2. Alternative therapies. 3. Benefits that are likely to result from the procedure. 4. Names and titles of all staff members who will be in the operating room. 5. The patient and family may withdraw consent at any time.

49. 2, 3, 5. 1. Although this is helpful information for a patient, it is not required on the informed consent form. 2. Alternative treatments are part of the informed consent process. 3. Benefits likely to result from the pro cedure are part of the process of informed consent. 4. Only the names of the persons performing the procedure are required for informed consent. 5. The statement that the patient may withdraw consent at any time is re quired for informed consent.

5. A case manager works in an outpatient clinic that administers palivizumab (Synagis) to premature infants at high risk for respiratory syncytial virus. Which outcome is most significant as an indication of effective treatment? 1. Prevention of hospitalization. 2. Optimum weight gain. 3. Promotion of parent-infant bonding. 4. Early detection and treatment of congenital defects.

5. 1. Palivizumab is an antibody administered monthly to premature and other high risk infants during respiratory syncytial virus season. The goal is prevention of serious respiratory syncytial virus bron chiolitis requiring hospitalization.

5. The parent of a child who is being treated for Haemophilus influenzae meningitis tells the nurse that the family is being treated prophylactically with rifampin (Rifadin). Which of the following should the nurse include in teaching about this medication? 1. "The drug will change the color of the urine to orange-red, so you should protect your undergarments as it will cause staining." 2. "Adverse effects of the drug may cause urinary retention." 3. "The drug is given to treat meningitis." 4. "You will need to continue taking the drug for 7 days."

5. 1. Rifampin (Rifadin) causes an orange-red discoloration of body fluids, including urine. Knowledge of this can decrease anxiety when it occurs.

5. A parent asks the nurse to define talipes varus. The nurse tells the parent that it is which of the following? 1. An inversion or bending inward of the foot. 2. An eversion or bending outward of the foot. 3. A high arch of the foot. 4. A low arch (flatfoot) of the foot.

5. 1. Talipes varus is an inversion of the foot.

50. A preterm neonate is admitted to the hospital. The physician orders indomethacin. The nurse informs the parents that the medication is given for which of the following reasons? 1. To encourage ductal closure. 2. To prevent hypertension. 3. To promote release of surfactant. 4. To protect the immature liver.

50. 1. Preterm neonates with good renal function may receive indomethacin, a prostaglandin inhibitor, to encourage ductal closure.

51. You are caring for a child with a retroperitoneal rhabdomyosarcoma. Select all the nursing diagnoses that apply. 1. Acute pain. 2. Risk for impaired urinary elimination. 3. Impaired gas exchange. 4. Self-care deficit. 5. Risk for constipation.

51. 1. Pain occurs due to pressure on the organs in the lower abdomen. 2. A retroperitoneal tumor affects the organs of the lower abdomen, including the bowel and bladder. 5. Because this tumor is in the lower abdomen, it puts pressure on the bowel causing constipation.

The mother of an 8-year-old child tells the clinic nurse that she is concerned about the child because the child seems to be more attentive to friends than anything else. Using Erikson's psychosocial development theory, the nurse should make which response? A. "You need to be concerned." B. "You need to monitor the child's behavior closely." C. "At this age, the child is developing his own personality." D. "You need to provide more praise to the child to stop this behavior."

C. "At this age, the child is developing his own personality." Rationale: According to Erikson, during school-age years (6 to 12 years of age), the child begins to move toward peers and friends and away from the parents for support. The child also begins to develop special interests that reflect his or her own developing personality instead of the parents. Therefore options 1, 2, and 4 are incorrect responses.

The nurse at a well-baby clinic is assessing the language and communication developmental milestones of a 4-month-old infant. On the basis of the age of the infant, what should the nurse expect to note as the highest-level developmental milestone? A. Cooing sounds B. Use of gestures C. Babbling sounds D. Increased interest in sounds

C. Babbling sounds Rationale: Babbling sounds are common between the ages of 3 and 4 months. Additionally, during this age, crying becomes more differentiated. Between the ages of 1 and 3 months, the infant will produce cooing sounds. The use of gestures occurs between 9 and 12 months. An increased interest in sounds occurs between 6 and 8 months.

The clinic nurse provides information to the mother of a toddler regarding toilet training. Which statement by the mother indicates a need for further information regarding toilet training? A. "Bladder control usually is achieved before bowel control." B. "The child should not be forced to sit on the potty for long periods." C. "The ability of the child to remove clothing is a sign of physical readiness." D. "The child will not be ready to toilet train until the age of about 18 to 24 months."

A. "Bladder control usually is achieved before bowel control." Rationale: Bowel control usually is achieved before bladder control. The child should not be forced to sit for long periods . The ability to remove clothing is one of the physical signs of readiness for toilet training. The physical ability to control the anal and urethral sphincters is achieved some time after the child is walking, probably between the ages of 18 and 24 months.

A mother of a 4-year-old expresses concern because her hospitalized child has begun thumb sucking. The mother states that this behavior began 2 days after hospital admission. Which response by the nurse is most appropriate? A. "It is best to ignore the behavior." B. "Your child is acting like a baby." C. "The doctor will need to be notified." D. "A 4-year-old is too old for this type of behavior."

A. "It is best to ignore the behavior." Rationale: In the hospitalized preschooler, the best option is to accept regression if it occurs. Regression is most often a result of the stress of the hospitalization. Parents may be overly concerned about regression and should be told that their child may continue the behavior at home. When regression does occur, the best approach is to ignore it while praising existing patterns of appropriate behavior. Calling the health care provider is not necessary. Options 2 and 4 are inappropriate.

5. A nurse is conducting dietary teaching on high-fiber foods for parents of a child with constipation. Which foods should the nurse include as being high in fiber? (Select all that apply.) a. White rice b. Avocados c. Whole grain breads d. Bran pancakes e. Raw carrots

ANS: C, D, E High-fiber foods include whole grain breads, bran pancakes, and raw carrots. Unrefined (brown) rice is high in fiber, but white rice is not. Raw fruits, especially those with skins or seeds, other than ripe banana or avocado, are high in fiber. DIF: Cognitive Level: Understand REF: p. 732 TOP: Integrated Process: Teaching/Learning MSC: Area of Client Needs: Health Promotion and Maintenance

3. José is a 4-year-old child scheduled for a cardiac catheterization. What should be included in preoperative teaching? a. Directed at his parents because he is too young to understand b. Detailed in regard to the actual procedures so he will know what to expect c. Done several days before the procedure so that he will be prepared d. Adapted to his level of development so that he can understand

ANS: D d. Adapted to his level of development so that he can understand Preoperative teaching should always be directed at the child's stage of development. The caregivers also benefit from the same explanations. The parents may ask additional questions, which should be answered, but the child needs to receive the information based on developmental level. Preschoolers will not understand in-depth descriptions and should be prepared close to the time of the cardiac catheterization. DIF: Cognitive Level: Apply REF: p. 739 TOP: Integrated Process: Teaching/Learning MSC: Area of Client Needs: Health Promotion and Maintenance

2. The parents of a child with cerebral palsy ask the nurse whether any drugs can decrease their child's spasticity. The nurse's response should be based on which statement? a. Anticonvulsant medications are sometimes useful for controlling spasticity. b. Medications that would be useful in reducing spasticity are too toxic for use with children. c. Many different medications can be highly effective in controlling spasticity. d. Implantation of a pump to deliver medication into the intrathecal space to decrease spasticity has recently become available.

ANS: D Baclofen, given intrathecally, is best suited for children with severe spasticity that interferes with activities of daily living and ambulation. Anticonvulsant medications are used when seizures occur in children with cerebral palsy. The intrathecal route decreases the side effects of the drugs that reduce spasticity. Few medications are currently available for the control of spasticity. DIF: Cognitive Level: Understand REF: p. 979 TOP: Integrated Process: Teaching/Learning MSC: Area of Client Needs: Physiologic Integrity: Pharmacologic and Parenteral Therapies

A child falls on the playground and has a small laceration on the forearm. The school nurse should do which of the following to cleanse the wound? a. Slowly pour hydrogen peroxide over wound. b. Soak arm in warm water and soap for at least 30 minutes. c. Gently cleanse with sterile pad and a nonstinging povidone-iodine solution. d. Wash wound gently with mild soap and water for several minutes.

ANS: D Lacerations should be washed gently with mild soap and water or normal saline.

22. Which clinical manifestation would be the most suggestive of acute appendicitis? a. Rebound tenderness b. Bright red or dark red rectal bleeding c. Abdominal pain that is relieved by eating d. Abdominal pain that is most intense at McBurney point

ANS: D Pain is the cardinal feature. It is initially generalized, usually periumbilical. The pain localizes to the right lower quadrant at McBurney point. Rebound tenderness is not a reliable sign and is extremely painful to the child. Bright red or dark red rectal bleeding and abdominal pain that is relieved by eating are not signs of acute appendicitis. DIF: Cognitive Level: Understand REF: p. 709 TOP: Integrated Process: Nursing Process: Assessment MSC: Area of Client Needs: Physiologic Integrity: Physiologic Adaptation

The nurse is talking to the parents of a child with pediculosis capitis. Which of the following should the nurse include when explaining how to manage pediculosis capitis? a. "You will need to cut the hair shorter if infestation and nits are severe." b. "You can distinguish viable from nonviable nits, and remove all viable ones." c. "You can wash all nits out of hair with a regular shampoo." d. "You will need to remove nits with an extra-fine tooth comb or tweezers."

ANS: D Treatment consists of the application of pediculicide and manual removal of nit cases. An extra-fine tooth comb facilitates manual removal.

18. A child is unconscious after a motor vehicle accident. The watery discharge from the nose tests positive for glucose. What does this finding suggest? a. Diabetic coma b. Brainstem injury c. Upper respiratory tract infection d. Leaking of cerebrospinal fluid (CSF)

ANS: D Watery discharge from the nose that is positive for glucose suggests leaking of CSF from a skull fracture and is not associated with diabetes or respiratory tract infection. The fluid is probably CSF from a skull fracture and does not signify whether the brainstem is involved. DIF: Cognitive Level: Apply REF: p. 906 TOP: Integrated Process: Nursing Process: Assessment MSC: Area of Client Needs: Physiologic Integrity: Physiologic Adaptation

The nurse is developing a plan of care for a 4-year-old child scheduled for a renal biopsy. What developmental characteristic of this child should the nurse consider? A. Masturbation is common in this age group. B. Body image may be a concern for the child. C. Fears of mutilation may be present in the child. D. The urination pattern will cause embarrassment for the child.

C. Fears of mutilation may be present in the child. Rationale: During the preschool years, a child's fears of separation and mutilation are great, because the child is facing the developmental task of trusting others. As the child gets older, fears about virility and reproductive ability may surface. Masturbation is most common in the toddler age group as they discover their genital organs. Body image is a concern for the adolescent. Options 1, 2, and 4 are not accurate occurrences in this age group.

The mother of a 20-month-old tells the nurse that the child has a barking cough at night. The child's temperature is 37ºC (98.6ºF). The mother states the child is not having difficulty breathing. The nurse suspects croup and should recommend A. controlling the fever with acetaminophen (Tylenol) and call the primary care provider if the cough gets worse tonight. B. trying a cool-mist vaporizer at night and watching for signs of difficulty breathing. C. trying over-the-counter cough medicine and coming to the clinic tomorrow if there is no improvement. D. bringing the child to the hospital to be admitted and to be observed for impending epiglottitis.

B

When developing a care plan for the child diagnosed with cystic fibrosis, which of the following must the nurse keep in mind? A. CF is an autosomal dominant hereditary disorder B. Pulmonary secretions are abnormally thick C. Obstruction of the endocrine glands occurs D. Elevated levels of potassium are found in the sweat

B

The mother of a 16-year-old tells a nurse that she is concerned because her child sleeps about 8 hours every night and until noontime every weekend. Which nursing response is most appropriate? A. "The child should not be staying up so late at night." B. "Adolescents need that amount of sleep every night." C. "If the child eats properly, that should not be happening." D. "The child probably is anemic and should eat more foods containing iron."

B. "Adolescents need that amount of sleep every night." Rationale: An adolescent needs about 8 hours of sleep per night. During this age, with an increase in social activities, school commitments, and possibly work activities, it is important that the adolescent receive enough sleep at night. Nothing in the question indicates that the child is staying up at night. Adolescents need 8 hours of sleep each night, so diet is not a concern. Although anemia can cause fatigue, there is nothing in the question to indicate that the child has anemia, and the nurse should not attempt to diagnose a medical condition.

The clinic nurse has provided instructions regarding dental care for toddlers to the mother of a 2-year-old child. Which statement, if made by the mother, indicates a need for further instruction? A. "It is best to substitute sweets or snacks with food items such as cheese." B. "Proper dental care is not necessary for a toddler until the permanent teeth erupt." C. "My child should have the first dental exam at some point after the second birthday." D. "I do not need to be concerned if the child swallows some toothpaste while brushing the teeth."

B. "Proper dental care is not necessary for a toddler until the permanent teeth erupt." Rationale: The nurse should instruct the mother that proper dental care for a toddler is important. It is important to instruct the mother to substitute sweets with healthy food items to prevent dental caries. The first dental visit should be made after the first primary tooth erupts and no later than 30 months of age. It will not hurt the child if some of the toothpaste is swallowed.

A mother tells the nurse in a pediatrician's office that she is concerned because her children must let themselves into the house after school each day while she is at work. The nurse explores which suggestion with the mother to decrease the children's sense of isolation and fear? A. Instruct the children never to cook. B. Let the children play in neighborhood homes. C. Find community after-school programs or activities. D. Have the children call the mother at work every hour.

C. Find community after-school programs or activities.

The nurse teaches the caregiver of the infant diagnosed with nasopharyngitis to call the physician if which of the following occurs? A. Coughing B. Infant becomes irritable C. Shows signs and symptoms of an ear infection D. Low-grade fever

C

Which of the following statements by the family of a child with asthma indicates a need for additional home care teaching? A. "We need to identify what things trigger his attacks" B. "He is to use his bronchodilator inhaler before the steroid inhaler" C. "We'll make sure that he avoids exercise to prevent attacks" D. "He should increase his fluid intake regularly to thin secretions"

C

19. Which statement by the mother of an 18-month-old would lead the nurse to believe that the child should be referred for further evaluation for developmental delay? 1. "My child is able to stand but is not yet taking steps independently." 2. "My child has a vocabulary of approximately 15 words." 3. "My child is still sucking his thumb." 4. "My child seems to be quite wary of strangers."

1. "My child is able to stand but is not yet taking steps independently." The child should be walking indepen- dently by 15 to 18 months. Because this toddler is 18 months and not walking, a referral should be made for a develop- mental consult. The vocabulary of an 18-month-old should be 10 words or more. Thumb-sucking is still common for 18-month-olds and may actually be at its peak at that age. It is very common for a child of 18 months to exhibit stranger anxiety.

10. A child and family are addressed by the nurse during admission to the pediatric unit. Which statement is the best example of a proactive service orientation? 1. "Do you have any special questions or concerns that I can answer?" 2. "It is important that you keep the crib side rails up at all times you are not at the bedside." 3. "We ask that you restrict visitors to two persons at the bedside due to space limitations." 4. "Because your child is on contact precautions, your child will have to remain in the room."

10. 1. This is a proactive statement with a "ready to help" image.

19. A child has had surgery to correct bilateral clubfeet, and the cast has been removed. While instructing the parents about their child's future, the nurse should include which of the following statements? Select all that apply. 1. "Your child will need to wear a brace on the feet 23 hours a day for at least 2 months." 2. "Your child should see an orthopedic surgeon regularly until the age of 18 years." 3. "Your child will not be able to participate in sports that require a lot of running." 4. "Your child may have a recurrence of clubfoot in a year or more." 5. "Most children treated for clubfeet develop feet that appear and function normally." 6. "Most children treated for clubfeet require surgery at puberty."

19. 1. After the final casting, bracing is required for 23 to 24 hours per day for 2 months. This decreases the likelihood of a recurrence. 2. Because clubfoot can recur, it is impor tant to have regular follow-up with the orthopedic surgeon until age 18 years. 4. Even with proper bracing, there may be a recurrence. 5. Most children treated for clubfeet develop normally appearing and functioning feet.

21. What would be the priority nursing action on finding the varicella vaccine at room temperature on the shelf in the medication room? 1. Ensure the varicella vaccine's integrity is intact; if intact, follow the five rights of medication administration. 2. Do not administer this batch of vaccine. 3. Ensure the varicella vaccine's integrity is intact; if intact, give the vaccine after verifying proper physician orders. 4. Ask the mother if the child has had any prior reactions to varicella.

2

23. Which nursing intervention should take place prior to all vaccination administrations? 1. Document the vaccination to be administered on the immunization record and medical record. 2. Provide the vaccine information statement handout, and answer all questions. 3. Administer the most painful vaccination first, and then alternate injection sites. 4. Refer to the vaccination as "baby shots" so the parent understands the baby will be receiving an injection.

2

30. What would be the best plan of care for a newborn whose mother's hepatitis B antigen status is unknown? 1. Give the infant the hepatitis B vaccine within 12 hours of birth. 2. Give the infant the hepatitis B vaccine and hepatitis B immune globulin within 12 hours of birth. 3. Give the infant the hepatitis B vaccine within 24 hours of birth. 4. Give the infant the hepatitis B vaccine and hepatitis B immune globulin within 24 hours of birth.

2

34. A child with a newly applied left leg cast initially feels fine, then starts to cry and tells his mother his leg hurts. Which assessment would be the nurse's first priority? 1. Cast integrity. 2. Neurovascular integrity. 3. Musculoskeletal integrity. 4. Soft-tissue integrity.

2

36. Which assessment is most important after any injury in a child? 1. History of loss of consciousness and length of time unconscious. 2. Serial assessments of level of consciousness. 3. Initial neurological assessment. 4. Initial vital signs and oxygen saturation level.

2

37. Which would be the most appropriate nursing intervention when caring for a child newly admitted with a mild head concussion and no cervical spine injury? 1. Keep head of bed flat, side rails up, and safety measures in place. 2. Elevate head of bed, side rails up, and safety measures in place. 3. Observe for drainage from any orifice and notify physician immediately. 4. Continually stimulate the child to keep awake to check neurological status.

2

39. What would be the most appropriate advice to give to the parent of a boy with slight visual blurring after being hit in the face with a basketball? 1. "Apply ice, observe for any further eye complaints, and bring him back if he has increased pain." 2. "Take him to the emergency department to ensure that he does not have any internal eye damage." 3. "Call your pediatrician if he starts vomiting, is hard to wake up, or has worsening of eye blurring." 4. "Observe for any further eye complaints, headaches, dizziness, or vomiting, and i worsening occurs, take him to your pediatrician."

2

42. Which intervention would be most appropriate for a 3-year-old boy who had just ingested dish detergent? 1. Discuss childproofing measures in the home in a nonthreatening manner. 2. Inquire about the circumstances of the ingestion. 3. Discuss having ipecac and the Poison Control phone number in the home. 4. Tell the mother you will be giving the boy medicine to make him throw up.

2

48. What would be the nurse's best advice to a mother who says her 3-year-old son ingested his father's Visine eye solution? 1. "Initiate vomiting immediately." 2. "Call the Poison Control Center." 3. "Call the pediatrician right away." 4. "Dilute with milk 1:1 volume of suspected ingestion."

2

49. What would be the best response if a mother tells the nurse that the only way she can get her 2-year-old daughter to take medicine is to call it candy? 1. Tell her that is fine as long as the child takes all of the medicine. 2. Discuss the importance of not calling medicine candy to prevent accidental drug ingestion. 3. Discuss with the mother that the child does not have to take the medicine if she does not want it. 4. Tell the mother her child will have to go to "time out" if she does not take her medicine.

2

56. Which would be the nurse's priority intervention if a 7-year-old girl's mother tells the nurse she has noticed excessive masturbation? 1. Tell her it is normal development for children of her age. 2. Ask the mother if anyone is abusing the daughter that she knows about. 3. Talk with the child and find out why she is touching herself down there. 4. Investigate thoroughly the circumstances in which she masturbates.

2

59. Which statement is true of abused children? 1. They will tell the truth if asked about their injuries. 2. They will repeat the same story as their parents. 3. They usually are not noted to have any changes in behavior. 4. They will have outgoing personalities and be active in school activities.

2

60. The nurse is working in the emergency room caring for a 10-year-old who was in an MVA. The child is currently on a backboard with a cervical collar in place. The child is diagnosed with a cervical fracture. Which of the following would the nurse expect to find in the child's plan of care? 1. Remove the cervical collar, keep the backboard in place, and administer high dose methlyprednisolone. 2. Continue with all forms of spinal stabilization, and administer high-dose methylprednisolone and ranitidine. 3. Remove the backboard and cervical collar, and prepare for halo traction placement. 4. Remove the cervical collar and backboard, place the child on spinal precautions, and administer high-dose methylprednisolone and ranitidine.

2. All forms of spinal stabilization should be continued while methylprednisolone and Zantac are administered.

45. The nurse is caring for several children. She knows that which of the following children is at increased risk for CP? 1. An infant born at 34 weeks with an Apgar score of 6 at 5 minutes. 2. A 17-day-old infant with sepsis. 3. A 24-month-old child who has experienced a febrile seizure. 4. A 5-year-old with a closed-head injury after falling off a bike.

2. Any infection of the central nervous system increases the infant's risk of CP.

29. A nurse is caring for a child with spastic cerebral palsy. Which of the following medications should be used for the treatment of spasticity? 1. Dexamethasone (Decadron). 2. Baclofen. 3. Diclofenac (Voltaren). 4. Carbamazepine (Tegretol).

2. Baclofen is used to treat the spasticity in cerebral palsy. It is a centrally acting muscle relaxant.

12. An interdisciplinary team is assembled to review protocols for management of central intravenous lines. Which staff should be represented on the team? 1. Experienced RNs and pharmacists. 2. RNs, physicians, and pharmacists. 3. RNs, LPNs, and physicians. 4. Charge nurses and staff physicians.

2. Because all three of these disciplines are involved in care of central intra venous lines, this answer is the most complete.

24. Which of the following is most important when teaching a parent about preventing osteomyelitis? 1. Parents can stop worrying about bone infection once their child reaches school age. 2. Parents need to clean open wounds thoroughly with soap and water. 3. Children will always get a fever if they have osteomyelitis. 4. Children should wear long pants when playing outside because their legs might get scratched.

2. Because bacteria from an open wound can lead to osteomyelitis, thorough cleaning with soap and water is the best prevention.

4. Morphine sulfate 2 mg IV q2h prn for pain is ordered for a 12-year-old who has had abdominal surgery. Which of the following is the most appropriate nursing action? 1. Administer the morphine sulfate using a syringe pump over 1 hour. 2. Encourage the child to do incentive spirometer every hour during the day and when awake at night. 3. Ask the physician to change the medication to Demerol (meperidine). 4. Administer the morphine sulfate with Benadryl (diphenhydramine) to prevent itching.

2. Because morphine sulfate can depress respirations and the child has just had abdominal surgery, deep breathing should be encouraged.

27. The nurse informs a parent of a teenager that the most effective treatment of acne is with which of the following medications? 1. Salicylic acid. 2. Benzoyl peroxide. 3. Chlorhexidine (Hibiclense). 4. Collagenase (Santyl).

2. Benzoyl peroxide inhibits growth of Propionibacterium acnes (a gram positive microorganism). It is effective against inflammatory and anti inflammatory acne.

21. When a child is suspected of having osteomyelitis, the nurse can prepare the family to expect which of the following? Select all that apply. 1. Pain medication is contraindicated so that symptoms are not masked. 2. Blood cultures will be obtained. 3. Pus will be aspirated from the subperiosteum. 4. An intravenous line with antibiotics will be started. 5. Surgery will be necessary.

2. Blood cultures will be obtained. 3. Pus will be aspirated from the subperiosteum. 4. Antibiotics will be given via an intravenous line.

61. Because estrogen is a possible trigger for a SLE flare, advice for a teenager who may become sexually active includes which of the following. Select all that apply. 1. Use Ortho Tri-Cyclen. 2. Use Depo-Provera. 3. Practice abstinence. 4. Use condoms. 5. Become pregnant. 6. Use Ortho Evra.

2. Depo-Provera is progesterone, the only contraceptive that is approved for use. 3. Abstinence is always recommended to prevent pregnancy. 4. Condoms are always recommended.

43. The nurse is charting on the computer at the nursing station when a parent exits a nearby room and asks for help because the child has vomited. Which immediate action is best? 1. Immediately assist the parent. 2. Put the computer in "suspend mode" so that the screen is blank and then assist the parent. 3. Finish recording information and exit out of the computer before assisting the parent. 4. Continue charting and call another team member to assist the parent.

2. Changing the computer to suspend mode requires only a few seconds. This answer is the best choice, because confidentiality is maintained and the patient receives prompt attention.

37. The nurse is caring for an infant with a myelomeningocele. The parents ask the nurse why the nurse keeps measuring the baby's head circumference. Select the nurse's best response: 1. "We measure all babies' heads to ensure that their growth is on track." 2. "Babies with myelomeningocele are at risk for hydrocephalus, which can show up with an increase in head circumference." 3. "Because your baby has an opening on the spinal cord, your infant is at risk for meningitis, which can show up with an increase in head circumference." 4. "Many infants with myelomeningocele have microcephaly, which can show up with a decrease in head circumference."

2. Children with myelomeningocele are at increased risk for hydrocephalus, which can be manifested with an in crease in head circumference.

36. A 22-lb child is to receive treatment for Kawasaki syndrome. The physician has or dered an intravenous infusion of gamma globulin 2 g/kg over 12 hours. Which of the following doses is correct? 1. 11 g. 2. 20 g. 3. 22 g. 4. 44 g.

2. Convert pounds to kilograms: 2.2 kg = 1 lb Divide 22 pounds by 2.2 kg 22 lb. ÷ 2.2 kg = 10 kg Calculate the dose: 2 g/kg = 2 g × 10 kg = 20 g

37. A 7-year-old is diagnosed with head lice. The physician has ordered lindane (G-Well) shampoo to be used once to treat the lice. The nurse knows that the shampoo is used only once for which of the following reasons? 1. Causes hypertension. 2. Associated seizures. 3. Associated with elevated liver functions. 4. Can cause alopecia.

2. Lindane (Kwell) with topical use is associated with seizures after absorption.

21. Which foods would the nurse recommend to the mother of a 2-year-old with anemia? 1. 32 oz of whole cow's milk per day. 2. Meats, eggs, and green vegetables. 3. Fruits, whole grains, and rice. 4. 8 oz of juice, three times per day.

2. Meats, eggs, and green vegetables.

23. The nurse is providing discharge teaching to the parents of a toddler who has experienced a febrile seizure. The nurse knows that clarification is needed when the mother says: 1. "My child will likely have another seizure." 2. "My child's 7-year-old brother is also at high risk for a febrile seizure." 3. "I'll give my child acetaminophen when ill to prevent the fever from rising too high too rapidly." 4. "Most children with febrile seizures do not require seizure medicine."

2. Most children over the age of 5 years do not have febrile seizures.

38. A child with osteosarcoma is going to receive chemotherapy before surgery. Which of the following statements by the parents indicates they understand the side effect of neutropenia? 1. "My child will be more at risk for diarrhea." 2. "My child will be more at risk for infection." 3. "My child's hair will fall out." 4. "My child will need to drink more."

2. Neutropenia makes a patient more at risk for infection, because the immune system is compromised due to the chemotherapy.

24. The nurse is caring for a 5-year-old female recently diagnosed with epilepsy. She is being evaluated for anticonvulsant medication therapy. The nurse knows that the child will likely be placed on which kind of regimen? 1. Two to three oral anticonvulsant medications so that dosing can be low and side effects minimized. 2. One oral anticonvulsant medication to observe effectiveness and minimize side effects. 3. One rectal gel to be administered in the event of a seizure. 4. A combination of oral and intravenous anticonvulsant medications to ensure compliance.

2. One medication is the preferred way to achieve seizure control. The child is monitored for side effects and drug levels.

30. Which approach should the nurse use to gather information from a child brought to the ED for suspected child abuse? 1. Promise the child that her parents will not know what she tells the nurse. 2. Promise the child that she will not have to see the suspected abuser again. 3. Use correct anatomical terms to discuss body parts. 4. Tell the child that the abuse is not her fault and that she is a good person.

4. Tell the child that the abuse is not her fault and that she is a good person.

31. The nurse is caring for a 6-year-old female with a skull fracture who is unconscious and has severely increased ICP. The nurse notes the child's temperature to be 104°F (40°C). Which of the following should the nurse do first? 1. Place a cooling blanket on the child. 2. Administer Tylenol via nasogastric tube. 3. Administer Tylenol rectally. 4. Place ice packs in the child's axillary areas.

31. 1. A cooling blanket will help cool the child quickly and at a controlled temperature.

31. A 14-year-old with osteogenesis imperfecta is confined to a wheelchair. Which nursing interventions will promote normal development? Select all that apply. 1. Encourage participation in groups with teens who have disabilities or chronic illness. 2. Encourage decorating the wheelchair with stickers. 3. Encourage transfer of primary care to an adult provider at age 18 years. 4. Allow the teen to view the radiographs. 5. Help the teen set realistic goals for the future. 6. Discourage discussion of sexuality, as the child is not likely to date.

31. 1. This patient is trying to become more independent and trying to fit in with the peer group. Encouraging socializ ing with peers who face similar chal lenges alleviates feelings of isolation. 2. Decorating the wheelchair encourages the patient to assume independence in self-care. 4. Allowing the patient to view radi ographs encourages the patient to assume self-care. 5. Helping the patient set realistic goals for the future encourages independence.

32. A physician has ordered amoxicillin (Amoxil) 500 mg intravenous piggyback every 8 hours for a child with tonsillitis. Which of the following actions by the nurse is appropriate? 1. Call the physician, and question the order because the route is incorrect. 2. Give the medication as ordered. 3. Call the physician because the dosing frequency is incorrect. 4. Call the physician, and question the dose of the drug.

32. 1. Amoxicillin is given only orally, so the order should be questioned.

32. After the birth of an infant with clubfoot, the nursery nurse should do which of the following when instructing the parents? Select all that apply. 1. Speak in simple language about the defect. 2. Avoid the parents unless providing direct care so they can grieve privately. 3. Keep the infant's feet covered at all times. 4. Present the infant as precious; emphasize the well-formed parts of the body. 5. Tell the parent that defects could be much worse. 6. Be prepared to answer questions multiple times.

32. 1. The parents will likely be shocked immediately after the birth of the child. To facilitate their understanding, the nurse should speak in simple terms. 4. The baby should be shown to the parents like all newborns, emphasiz ing the well-formed parts of the body. 6. Information may need to be repeated as the family begins to absorb the information.

33. A 2-month-old infant is brought to the emergency room after experiencing a seizure. The nurse notes that the infant appears lethargic with very irregular respirations and periods of apnea. The parents report that the child is no longer interested in feeding and that, prior to the seizure, the infant rolled off the couch. What additional testing should the nurse immediately prepare for? 1. Computed tomography scan of the head and dilation of the eyes. 2. Computed tomography scan of the head and EEG. 3. Close monitoring of vital signs. 4. X-rays of all long bones.

33. 1. A computed tomography scan of the head will reveal trauma. Dilating the eyes is performed to check for retinal hemorrhages that are seen in an infant who has experienced SBS.

33. The nurse should be suspicious of what condition in the following figure?_________________________________________________________________

33. DDH. The asymmetry of the thigh folds suggests DDH.

34. Decongestant nasal drops are prescribed for a child with nasopharyngitis. The nurse should include which of the following instructions for the parents about the nasal drops? 1. "Do not use the drops for any other family member." 2. "Administer the drops as often as necessary until the nasal congestion subsides." 3. "Insert the dropper tip as far back as possible to make sure you get the medication in the nasal passage." 4. "You can save the drops for the next time your child has the same symptoms."

34. 1. The medication should not be shared because of the risk of spreading the in fection to another family member.

34. Name the harness in the following figure._________________________________

34. Pavlik harness. The Pavlik harness is used to treat DDH diagnosed in the newborn period.

1. A 16-year-old is hospitalized for treatment of gunshot wounds acquired in gang con flict. The teen often shouts at the nurses and uses vulgar language. Which activity is the best example of patient advocacy? 1. Accepting the rude behavior without comment. 2. Avoiding entering the room except for scheduled treatments. 3. Meeting all demands in order to achieve peace. 4. Asking the teen to describe any fears about current treatment.

4. All patients have the right to be in formed and participate in care decisions. Assessment of needs is the first step in providing culturally congruent care and education.

8. Which of the following is a component of family-centered care? 1. Reinforce all parenting practices. 2. Accept all cultural practices and rituals. 3. Guarantee that financial needs are met. 4. Recognize family strengths.

4. Recognizing and building on family strengths is an important component of family-centered care.

14. An 8-day-old was admitted to the hospital with vomiting and dehydration. The newborn's heart rate is 170, respiratory rate is 44, blood pressure is 85/52, and temperature is 99°F (37.2°C). What is the nurse's best response to the parents who ask if the vital signs are normal? 1. "The blood pressure is elevated, but the other vital signs are within normal limits." 2. "The temperature is elevated, but the other vital signs are within normal limits." 3. "The respiratory rate is elevated, but the other vital signs are within normal limits." 4. "The heart rate is elevated, but the other vital signs are within normal limits."

4. "The heart rate is elevated, but the other vital signs are within normal limits."

61. The mother of an adolescent complains that he has had some recent behavioral changes. He comes home from school every day, closes his door, and refrains from interaction with his family. The nurse's best response to the mother is: 1. "You should speak with your son and ask him directly what is wrong with him." 2. "You should set limits with your son and tell him that this is unacceptable behavior." 3. "Your son's behavior is abnormal, and he is going to need a psychiatric referral." 4. "Your son's behavior is normal. You should listen to him without being judgmental."

4. "Your son's behavior is normal. You should listen to him without being judgmental."

4. The nurse is teaching about congenital clubfoot in infants. The nurse evaluates the teaching as successful when the parent states that clubfoot is best treated when? 1. Immediately after diagnosis. 2. At age 4 to 6 months. 3. Prior to walking (age 9 to 12 months). 4. After walking is established (age 15 to 18 months).

4. 1. The best outcomes for clubfoot are seen if casting begins as soon as the diagnosis is made.

32. The nurse is caring for a 16-year-old female who remains unconscious 24 hours after sustaining a closed-head injury in an MVA. She responds to deep painful stimulation with decorticate posturing. The child has an intracranial monitor that shows periodic increased ICP. All other vital signs remain stable. Select the most appropriate nursing action. 1. Encourage the child's peers to visit and talk to the child about school and other pertinent events. 2. Encourage the child's parents to hold her hand and speak loudly to her in an attempt to help her regain consciousness. 3. Attempt to keep a normal day/night pattern by keeping the child in a bright lively environment during the day and dark quiet environment at night. 4. Attempt to keep the environment dark and quiet, and encourage minimal stimulation.

4. A dark, quiet environment and minimal stimulation will decrease oxygen con sumption and ICP.

6. A 2-year-old child has been prescribed amoxicillin (Amoxil) three times a day for treatment of pharyngitis. Which of the following statements by the parent indicates the parent knows how to give the medication? 1. "If I miss giving my child a dose at lunch, I will double up on the dose at night." 2. "I will give the medication at breakfast, lunch, and dinner." 3. "I know that amoxicillin (Amoxil) is a chewable tablet, but sometimes my child likes to swallow it whole." 4. "I will continue giving the amoxicillin (Amoxil) for 10 days even if my child's cough gets better."

4. A full course of the antibiotic must be taken to decrease the risk of resistance to the antibiotic or recurrence of the infection.

40. The parents of a 12-month-old female with a neurogenic bladder ask the nurse if their child will always have to be catheterized. Select the nurse's best response. 1. "Your child will never feel when her bladder is full, so she will always have to be catheterized. Because she is female, she will always need assistance." 2. "As your child ages, she will likely be able to sense when her bladder is full and will be able to empty it on her own." 3. "Although your child will not be able to feel when her bladder is full, she can learn to urinate every 4 to 6 hours and therefore not require catheterizations." 4. "Your child will never be able to completely empty her bladder spontaneously, but there are other options to traditional catheterization. An opening can be made surgically through the abdomen, thus allowing the parents and child to be able to place a catheter into the opening."

4. A vesicostomy is an example of an op tion for children with myelomeningo celes where alternatives to traditional catheterizations are created.

37. When teaching parents about osteosarcoma, the nurse knows instruction has been successful when a parent says that this type of cancer is common in which age group? 1. Infancy. 2. Toddlers. 3. School-age children. 4. Adolescents.

4. Osteosarcoma is a common cancer of adolescents.

4. The nurse is caring for a 9-year-old female who is unconscious in the PICU. The child's mother has been calling her name repeatedly and gently shaking her shoulders in an attempt to wake her up. The nurse notes that the child is flexing her arms and wrists while bringing her arms closer to the midline of her body. The child's mother asks, "What is going on?" Select the nurse's best response. 1. "I think your daughter hears you, and she is attempting to reach out to you." 2. "Your child is responding to you; please continue trying to stimulate her." 3. "It appears that your child is having a seizure." 4. "Your child is demonstrating a reflex that indicates she is overwhelmed with the stimulation she is receiving."

4. Posturing is a reflex that often indicates that the child is receiving too much stimulation.

60. Renal involvement is a side effect of latent SLE. Which of the following is an impor tant nursing intervention to monitor in a child with renal involvement? 1. Push fluids or start IVF. 2. Check for uric salts in urine. 3. Watch for hypotension. 4. Check for protein in urine.

4. Protein in urine is a sign of renal impairment, even in nephrotic syndrome, in which the kidneys are losing protein.

11. Which of the following should be performed to make a diagnosis of SCFE? 1. A history of hip trauma. 2. A physical examination of hip, thigh, and knees. 3. A complete blood count. 4. A radiographic examination of the hip.

4. Radiographic examination is the only definitive diagnostic tool for SCFE.

60. Ribavirin (Virazole) is prescribed for a hospitalized child with respiratory syncytial virus. The nurse prepares to administer the medication by which of the following routes? 1. Oral. 2. Subcutaneous. 3. Intramuscular. 4. Oxygen tent.

4. Ribavirin (Virazole) is an antiviral res piratory medication used in the hospi tal for children with severe respiratory syncytial virus. Administration is via hood, face mask, or oxygen tent.

55. The treatment for a child with sinus bradycardia is atropine 0.02 mg/kg/dose. How much should the nurse give a child who weighs 20 kg? 1. 0.02 mg. 2. 0.04 mg. 3. 0.2 mg. 4. 0.4 mg.

4. Take the dose of atropine (0.02 mg/kg) times the child's weight (20 kg) 0.02 mg/kg × 20 kg = 0.4 mg

39. The nurse is caring for a newborn infant who has just been diagnosed with a myelomeningocele. Which of the following is included in the child's plan of care? 1. Place the child in the prone position with a sterile dry dressing over the defect. Slowly begin oral gastric feeds to prevent the development of necrotizing enterocolitis. 2. Place the child in the prone position with a sterile dry dressing over the defect. Begin intravenous fluids to prevent dehydration. 3. Place the child in the prone position with a sterile moist dressing over the defect. Slowly begin oral gastric feeds to prevent the development of necrotizing enterocolitis. 4. Place the child in the prone position with a sterile moist dressing over the defect. Begin intravenous fluids to prevent dehydration.

4. The child is placed in the prone posi tion to avoid any pressure on the de fect. A sterile moist dressing is placed over the defect to keep it as clean as possible. Intravenous fluids are begun after the surgery.

24. According to developmental theories, which important event is essential to the development of the toddler? 1. The child learns to feed self. 2. The child develops friendships. 3. The child learns to walk. 4. The child participates in being potty-trained.

4. The child participates in being potty-trained. Developmental theorists like Erickson and Freud believe that toilet training is the essential event that must be mastered by the toddler.

31. The nurse is to administer eardrops to a 5-year-old who has a draining ear. Which of the following is the correct method of instillation? 1. Pull the pinna of the ear downward and back for instillation. 2. Place cotton tightly in the ear after instillation. 3. Have the child remain upright after instillation. 4. Pull the pinna of the ear upward and back for instillation.

4. The correct way to administer eardrops in a child older than 3 years of age is to pull the pinna up and back, the same as for an adult.

20. Filgrastim (Neupogen) is given to a child who has received chemotherapy. The nurse knows that this drug is given for which of the following reasons? 1. Reduce fatigue level. 2. Prevent infection. 3. Reduce nausea and vomiting. 4. Increase mobilization of stem cells.

4. The drug mobilizes stem cells to produce neutrophils.

7. A nurse is caring for a child who is receiving amphotericin B intravenously daily for a fungal infection. Prior to starting the therapy, the nurse should review which of the following? 1. Aspartate aminotransferase and alanine aminotransferase levels. 2. Serum amphotericin level. 3. Serum protein and sodium levels. 4. Blood, urea, and nitrogen and creatinine levels.

4. The drug tends to be nephrotoxic. Ele vation of blood, urea, and nitrogen and creatinine levels indicates renal damage. If elevated, the physician must be noti fied to determine if the drug must be withheld for the day.

52. The legal basis for nursing activities may be at the local, state, or national level. The nurse should check the agency's policy and procedure manual as the primary legal authority for which of the following? 1. Whether an RN can initiate a blood transfusion. 2. Sharing of patient-sensitive information. 3. Legal protection when providing CPR to a non-patient. 4. Procedure for flushing a central venous line.

4. The local agency's policy manual would describe acceptable procedures for performing specific skills, such as flushing a central venous line. These procedures serve as the legal standard of care.

46. An adolescent is about to have a tuberculin skin test. Which is the best area for this intradermal test? 1. Upper thigh. 2. Scapular area. 3. Back. 4. Ventral forearm.

4. The ventral forearm is the preferred site for the tuberculin skin test.

7. The nurse receives a phone call from the parents of a 9-year-old female who is com plaining of a headache and blurry vision. The child has been healthy but has a history of hydrocephalus and received a ventriculoperitoneal shunt at the age of 1 month. The parents also state that she is not acting like herself, is irritable, and sleeps more than she used to. They ask the nurse what they should do. Select the nurse's best response. 1. "Give her some acetaminophen, and see if her symptoms improve. If they do not improve, bring her to the pediatrician's office." 2. "It is common for girls to have these symptoms, especially prior to beginning their menstrual cycle. Give her a few days, and see if she improves." 3. "You are probably worried that she is having a problem with her shunt. This is very unlikely as it has been working well for 9 years." 4. "You should immediately bring her to the emergency room as these may be symptoms of a shunt malfunction."

4. These are symptoms of a shunt malfunc tion and should be evaluated immediately.

27. Which pediatric patient can benefit most by a primary nursing approach in which one nurse cares for the child whenever on duty and coordinates other staff care? 1. A patient admitted for a diagnostic workup of mononucleosis. 2. A patient who had surgery 1 day ago for appendicitis. 3. An infant treated with phototherapy for hyperbilirubinemia. 4. A toddler with leukemia, admitted for chemotherapy.

4. This child is likely to be admitted for repeated hospitalizations and over a longer period of time than other pa tients. Having a close relationship with one or a few nurses can lead to increased satisfaction for all.

24. For a school-age child who has Kawasaki disease and is taking aspirin, which laboratory value should be reported to the physician? 1. Blood, urea, nitrogen 18 mg/dL. 2. Hematocrit 42%. 3. Potassium 3.8 mEq/L. 4. PT 14.6 sec.

4. This value indicates an increase in PT and should be reported to the physi cian. Normal range is 11-12.5 seconds. Prolonged bleeding time can be a side effect of aspirin.

58. A charge nurse overhears a staff nurse make an erroneous statement to a parent in the hallway. Which initial approach by the charge nurse should follow the observation? 1. Discuss the correct information with the parent. 2. Discuss the observation with the manager of the unit. 3. Write a description of the incident and share it with the staff nurse. 4. Ask the staff nurse to describe the situation in private.

4. Validation of information that was actu ally communicated to the parent should be the next step. This should come from the staff nurse in a private discussion.

41. Use the following labels to n 1. Epiphysis. 2. Diaphysis. 3. Epiphyseal plate. 4. Medullary cavity. 5. Calcaneus bone. 6. Compact bone.

41. Calcaneus bone Epiphysis Diaphysis Epiphyseal plate Medullary cavity Compact bone

41. Which situation would be appropriate to refer to the hospital ethics committee? 1. The physician recommends that a young child in the end stages of terminal cancer be taken off the ventilator. The parents, who are divorced and have joint custody of the child, have different views about whether to discontinue the ventilator. 2. A child in end-stage renal failure is placed on the renal transplant list. The parents are asked to sign permission for surgery after a cadaver kidney is located. One parent is out of town and gives telephone consent. 3. After initial therapies have failed, a child with leukemia is evaluated for a new cancer protocol. The child, age 8 years, gives assent for the new treatment, and his parents give consent as well. 4. Parents are shocked when their child is diagnosed with a malignant bone tumor. The orthopedic surgeon discusses the options of limb amputation and a limb salvage procedure. The parents are asked to consider each option.

41. 1. This situation involves differences of opinion among persons who have a le gal responsibility to make decisions for the child. Consultation by the multidis ciplinary members of the ethics com mittee may help caregivers reach an agreement.

42. The nurse is teaching an adolescent about Ewing sarcoma and indicates which of the following as a common site? 1. Shaft. 2. Growth plate. 3. Ball of the femur. 4. Bone marrow.

42. 1. Ewing sarcoma is a bone tumor that affects the shafts of long bones.

42. A child is receiving furosemide (Lasix) 20 mg daily. The parent asks the nurse what is the best time of day for giving the medication. The nurse indicates which of the fol lowing times? 1. 8:00 a.m. 2. 12 noon. 3. 6:00 p.m. 4. Bedtime.

42. 1. The onset of Lasix is 20 to 60 minutes. It peaks at 60 to 70 minutes, with a du ration of 2 hours. By 24 hours, 50% is eliminated. Because the child may not respond as expected, the mother should be instructed to give the Lasix at 8:00 a.m. to avoid interruption of sleep with frequent urination.

43. Which of the following nursing diagnoses is most important for a child with Ewing sarcoma who will be undergoing chemotherapy? 1. Risk for fluid volume deficit. 2. Potential for chronic pain. 3. Risk for skin impairment. 4. Ineffective airway clearance.

43. 1. Chemotherapy can cause nausea, vomiting, and possibly diarrhea, which contribute to fluid volume deficit.

43. A school nurse administers albuterol (Proventil) to a 10-year-old who is having an acute asthma attack. Which of the following assessment findings should the nurse observe? 1. Decrease in wheezing. 2. Decrease in respiratory rate from 34 to 22. 3. Decrease in dyspnea. 4. Decrease in heart rate.

43. 1. The symptoms of an acute asthma at tack are related to constriction of the airway, which leads to dyspnea and an increased respiratory rate. The albuterol (Proventil) is a beta-adrenergic agent that relaxes the smooth muscles of the bronchial tree, which will decrease the wheezing.

44. A child is going to receive radiation for Ewing sarcoma. Which of the following is the best nursing intervention to prevent skin breakdown during therapy? 1. Advise the child to wear loose-fitting clothes to minimize irritation. 2. Advise the child to use emollients to prevent dry skin. 3. Apply cold packs nightly to reduce the warmth caused by the treatments. 4. Apply hydrocortisone to soothe itching from dry skin.

44. 1. Loose clothing helps reduce irritation on the sensitive irradiated skin.

46. A child with Ewing sarcoma is receiving chemotherapy and is experiencing severe nausea. The nurse has to administer Ativan at 0.04 mg/kg, and the child weighs 65 lb. What dose should the nurse administer? __________________

46. 1.1 mg. Change pounds to kilograms (2.2 lb 1 kg: 65/2.2 29.5 kg). Then multiply kilograms by the dose of 0.04 mg/kg: 29.5 0.04 1.1 mg.

47. A hospitalized child is to receive 75 mg of acetaminophen (Tylenol) for fever of 101°F (38.3°C). If the acetaminophen (Tylenol) is 80 mg per 0.8 mL, how much will the nurse administer? 1. 0.75 mL. 2. 1.5 mL. 3. 2.5 mL. 4. 3 mL.

47. 1. Desired over Available × Volume = Desired dose 75 mg × 0.8 mL = 0.75 mL 80 mg

48. Staff members working with school-age children believe it is important for each child to understand and agree to medical treatment, especially when treatment is part of research protocols. The term for this process is: 1. Assent. 2. Informed consent. 3. Confidentiality. 4. Emancipation.

48. 1. Assent is the process by which children give their consent for medical treat ment. Assent does not have the same legal implications as informed consent, which is given by parents or legal representatives.

The 2-year-old child is treated in the emergency department for a burn to the chest and abdomen. The child sustained the burn by grabbing a cup of hot coffee that was left on the kitchen counter. The nurse reviews safety principles with the parents before discharge. Which statement by the parents indicates an understanding of measures to provide safety in the home? A. "We will be sure not to leave hot liquids unattended." B. "I guess my children need to understand what the word hot means." C. "We will be sure that the children stay in their rooms when we work in the kitchen." D. "We will install a safety gate as soon as we get home so the children cannot get into the kitchen."

A. "We will be sure not to leave hot liquids unattended." Rationale: Toddlers, with their increased mobility and development of motor skills, can reach hot water or hot objects placed on counters and stoves and can reach open fires or stove burners above their eye level. The nurse should encourage parents to remain in the kitchen when preparing a meal, use the back burners on the stove, and turn pot handles inward and toward the middle of the stove. Hot liquids should never be left unattended or within the child's reach, and the toddler should always be supervised. The statements in options 2, 3, and 4 do not indicate an understanding of the principles of safety.

A 7-year-old child is hospitalized with a fracture of the femur and is placed in traction. In meeting the psychosocial needs of the child, the nurse most appropriately selects which play activity for the child? A. A board game B. A large puzzle C. A finger-painting set D. A coloring book with crayons

A. A board game Rationale: The school-age child becomes organized with more direction with play activities. Such activities include collections, drawing, construction, dolls, pets, guessing games, board and computer games, riddles, hobbies, competitive games, and listening to the radio or television. Options 3 and 4 are appropriate for a preschooler. Option 2 is appropriate for a toddler.

The maternity nurse is providing instructions to a new mother regarding the psychosocial development of the newborn infant. Using Erikson's psychosocial development theory, the nurse instructs the mother to take which measure? A. Allow the newborn infant to signal a need. B. Anticipate all the needs of the newborn infant. C. Attend to the newborn infant immediately when crying. D. Avoid the newborn infant during the first 10 minutes of crying.

A. Allow the newborn infant to signal a need. Rationale: According to Erikson, the caregiver should not try to anticipate the newborn infant's needs at all times but must allow the newborn infant to signal needs. If a newborn infant is not allowed to signal a need, the newborn will not learn how to control the environment. Erikson believed that a delayed or prolonged response to a newborn infant's signal would inhibit the development of trust and lead to mistrust of others.

The mother of a 5-year-old child tells the nurse that the child scolds the floor or a table if she hurts herself on the object. According to Piaget's Theory of Cognitive Development, what term or phrase best describes this behavior? A. Animism B. Egocentric speech C. Object permanence D. Global organization

A. Animism

The nurse is caring for a 14-year-old girl who is hospitalized and has been placed in traction using Crutchfield tongs. The child is having difficulty adjusting to the prolonged hospital confinement. Which nursing action would be most appropriate to meet the child's needs? A. Let the child wear her own clothing when friends visit. B. Allow the girl to have her hair dyed if the parent agrees. C. Allow the child to play loud music in the hospital room. D. Allow the child to keep the shades closed and the room darkened at all times.

A. Let the child wear her own clothing when friends visit. Rationale: Adolescents need to identify with their peers and have a strong need to belong to a group. They prefer to dress like the group and wear similar hairstyles, which are different from their parents'. The child should be allowed to wear her own clothes to feel a sense of belonging to the group. Because Crutchfield tongs require the use of skeletal pins, hair dye is not appropriate. Loud music may disturb others in the hospital. The child's request for a darkened room may indicate a problem with depression that may need further evaluation and intervention.

Which interventions are appropriate for the care of an infant? SELECT ALL THAT APPLY A. Provide swaddling. B. Talk in a loud voice. C. Provide the infant with a bottle of juice at nap time. D. Hang mobiles with black and white contrast designs. E. Caress the infant while bathing or during diaper changes. F. Allow the infant to cry for at least 10 minutes before responding.

A. Provide swaddling. D. Hang mobiles with black and white contrast designs. E. Caress the infant while bathing or during diaper changes.

Which of the following is usually the only symptom of pediculosis capitis (head lice)? a. Itching b. Vesicles c. Scalp rash d. Localized inflammatory response

ANS: A Itching is generally the only manifestation of pediculosis capitis (head lice). Diagnosis is made by observation of the white eggs (nits) on the hair shaft

A parent of a 3-year-old tells a clinic nurse that the child is rebelling constantly and having temper tantrums. Using Erikson's psychosocial development theory, which instructions should the nurse provide to the parent? SELECT ALL THAT APPLY A. Set limits on the child's behavior. B. Ignore the child when this behavior occurs. C. Allow the behavior, because this is normal at this age period. D. Provide a simple explanation of why the behavior is unacceptable. E. Punish the child every time the child says "no" to change the behavior.

A. Set limits on the child's behavior. D. Provide a simple explanation of why the behavior is unacceptable. Rationale: According to Erikson, the child focuses on gaining some basic control over self and the environment and independence between ages 1 and 3 years. Gaining independence often means that the child has to rebel against the parents' wishes. Saying things like "no" or "mine" and having temper tantrums are common during this period of development. Being consistent and setting limits on the child's behavior are necessary elements. Providing a simple explanation of why certain behaviors are unacceptable is an appropriate action. Options 2 and 3 do not address the child's behavior. Option 5 is likely to produce a negative response during this normal developmental pattern.

A 1-year-old child with hypospadias is scheduled for surgery to correct this condition. While preparing the nursing care plan for this child, which factor should the nurse take into consideration? A. This surgery is taking place at a time when fears of separation are great. B. This surgery is taking place at a time when sibling rivalry will cause regression to occur. C. This surgery is taking place at a time when concern over size and function of the penis is present. D. This surgery is taking place at a time when embarrassment about voiding irregularities is common.

A. This surgery is taking place at a time when fears of separation are great. Rationale: At the age of 1 year, a child's fears of separation are great because the child is facing the developmental task of trusting others. No data in the question allow one to determine that siblings exist. The options "concern over size and function of the penis is present" and "embarrassment about voiding irregularities is common" might be issues if the child were older.

The nurse is caring for a 4-year-old child. When experiencing pain, the nurse anticipates which about the child? Select all that apply. A. Views pain as a punishment B. Verbalizes the reason for the pain C. Blames someone else for the pain D. Believes pain will disappear magically E. Fears losing control during the painful episode F. Will be able to explain the sequence of events leading to the pain

A. Views pain as a punishment C. Blames someone else for the pain D. Believes pain will disappear magically

48. A nurse is receiving report on a newborn admitted yesterday after a gastroschisis repair. In the report, the nurse is told the newborn has a physician's prescription for an NG tube to low intermittent suction. The reporting nurse confirms that the NG tube is to low intermittent suction and draining light green stomach contents. Upon initial assessment, the nurse notes that the newborn has pulled the NG tube out. Which is the priority action the nurse should take? a. Replace the NG tube and continue the low intermittent suction. b. Leave the NG tube out and notify the physician at the end of the shift. c. Leave the NG tube out and monitor for bowel sounds. d. Replace the NG tube, but leave to gravity drainage instead of low wall suction.

ANS: A A newborn with a gastroschisis performed the day before will require bowel decompression with an NG tube to low wall intermittent suction. The nurse's priority action is to replace the NG tube and continue with the low wall intermittent suctioning. The NG tube cannot be left out this soon after surgery. The physician's prescription was to have the NG tube to low wall intermittent suction, so the tube cannot be placed to gravity drainage. DIF: Cognitive Level: Apply REF: p. 729 TOP: Integrated Process: Nursing Process: Planning MSC: Area of Client Needs: Physiologic Integrity: Reduction of Risk Potential

4. The nurse is conducting a staff in-service on common problems associated with myelomeningocele. Which common problem is associated with this defect? a. Hydrocephalus b. Craniostenosis c. Biliary atresia d. Esophageal atresia

ANS: A Hydrocephalus is a frequently associated anomaly in 80% to 90% of children. Craniostenosis is the preterm closing of the cranial sutures and is not associated with myelomeningocele. Biliary and esophageal atresia is not associated with myelomeningocele. DIF: Cognitive Level: Understand REF: p. 984 TOP: Integrated Process: Teaching/Learning MSC: Area of Client Needs: Physiologic Integrity: Physiologic Adaptation

1. The nurse is planning a staff in-service on childhood spastic cerebral palsy. What characterizes spastic cerebral palsy? a. Hypertonicity and poor control of posture, balance, and coordinated motion b. Athetosis and dystonic movements c. Wide-based gait and poor performance of rapid, repetitive movements d. Tremors and lack of active movement

ANS: A Hypertonicity and poor control of posture, balance, and coordinated motion are part of the classification of spastic cerebral palsy. Athetosis and dystonic movements are part of the classification of dyskinetic (athetoid) cerebral palsy. Wide-based gait and poor performance of rapid, repetitive movements are part of the classification of ataxic cerebral palsy. Tremors and lack of active movement may indicate other neurologic disorders. DIF: Cognitive Level: Understand REF: p. 978 TOP: Integrated Process: Teaching/Learning MSC: Area of Client Needs: Physiologic Integrity: Physiologic Adaptation

20. The nurse is teaching nursing students about childhood nervous system tumors. Which best describes a neuroblastoma? a. Diagnosis is usually made after metastasis occurs. b. Early diagnosis is usually possible because of the obvious clinical manifestations. c. It is the most common brain tumor in young children. d. It is the most common benign tumor in young children.

ANS: A Neuroblastoma is a silent tumor with few symptoms. In more than 70% of cases, diagnosis is made after metastasis occurs, with the first signs caused by involvement in the nonprimary site. In only 30% of cases is diagnosis made before metastasis. Neuroblastomas are the most common malignant extracranial solid tumors in children. The majority of tumors develop in the adrenal glands or the retroperitoneal sympathetic chain. They are not benign but metastasize. DIF: Cognitive Level: Apply REF: p. 835 TOP: Integrated Process: Teaching/Learning MSC: Area of Client Needs: Physiologic Integrity: Physiologic Adaptation

The nurse is teaching a class on preventing diaper rash in newborns to a group of new parents. Which of the following statements made by a parent indicates a correct understanding of the teaching? a. "I should wash my infant's buttocks with soap and water every time I change the diaper." b. "I will wash with a mild soap and water and dry thoroughly whenever my infant has a bowel movement." c. "I should wash my infant's buttocks with soap before applying a thin layer of oil." d. "I will apply baby oil and powder to the creases in my infant's buttocks."

ANS: A Overwashing the skin should be avoided, especially with perfumed soaps or commercial wipes, which may be irritating.

10. When both parents have sickle cell trait, which is the chance their children will have sickle cell anemia? a. 25% b. 50% c. 75% d. 100%

ANS: A Sickle cell anemia is inherited in an autosomal recessive pattern. If both parents have sickle cell trait (one copy of the sickle cell gene), then for each pregnancy, a 25% chance exists that their child will be affected with sickle cell disease. With each pregnancy, a 50% chance exists that the child will have sickle cell trait. Percentages of 75% and 100% are too high for the children of parents who have sickle cell trait. DIF: Cognitive Level: Analyze REF: p. 791 TOP: Integrated Process: Nursing Process: Assessment MSC: Area of Client Needs: Physiologic Integrity: Reduction of Risk Potential

9. The nurse is conducting a staff in-service on congenital heart defects. Which structural defect constitutes tetralogy of Fallot? a. Pulmonic stenosis, ventricular septal defect, overriding aorta, right ventricular hypertrophy b. Aortic stenosis, ventricular septal defect, overriding aorta, right ventricular hypertrophy c. Aortic stenosis, atrial septal defect, overriding aorta, left ventricular hypertrophy d. Pulmonic stenosis, ventricular septal defect, aortic hypertrophy, left ventricular hypertrophy

ANS: A Tetralogy of Fallot has these four characteristics: pulmonic stenosis, ventricular septal defect, overriding aorta, and right ventricular hypertrophy. There is pulmonic stenosis but not atrial stenosis in tetralogy of Fallot. Right ventricular hypertrophy, not left ventricular hypertrophy, is present in tetralogy of Fallot. Tetralogy of Fallot has right ventricular hypertrophy, not left ventricular hypertrophy, and an atrial septal defect, not aortic hypertrophy. DIF: Cognitive Level: Understand REF: p. 743 TOP: Integrated Process: Nursing Process: Assessment MSC: Area of Client Needs: Physiologic Integrity: Physiologic Adaptation

23. Parents of a 3-year-old child with congenital heart disease are afraid to let their child play with other children because of possible overexertion. The nurse's reply should be based on which statement? a. The child needs opportunities to play with peers. b. The child needs to understand that peers' activities are too strenuous. c. Parents can meet all of the child's needs. d. Constant parental supervision is needed to avoid overexertion.

ANS: A The child needs opportunities for social development. Children usually limit their activities if allowed to set their own pace. The child will limit activities as necessary. Parents must be encouraged to seek appropriate social activities for the child, especially before kindergarten. The child needs to have activities that foster independence. The child will be able to regulate activities. DIF: Cognitive Level: Analyze REF: p. 760 TOP: Integrated Process: Teaching/Learning MSC: Area of Client Needs: Psychosocial Integrity

Therapeutic management of the child with ringworm infection would include which of the following? a. Administer oral griseofulvin. b. Administer topical or oral antibiotics. c. Apply topical sulfonamides. d. Apply Burow solution compresses to affected area.

ANS: A Treatment with the antifungal agent griseofulvin is part of the treatment for the fungal disease, ringworm. Oral griseofulvin therapy frequently continues for weeks or months.

3. The nurse is conducting discharge teaching to parents of a preschool child with myelomeningocele, repaired at birth, being discharged from the hospital after a urinary tract infection (UTI). Which should the nurse include in the discharge instructions related to management of the child's genitourinary function? (Select all that apply.) a. Continue to perform the clean intermittent catheterizations (CIC) at home. b. Administer the oxybutynin chloride (Ditropan) as prescribed. c. Reduce fluid intake in the afternoon and evening hours. d. Monitor for signs of a recurrent urinary tract infection. e. Administer furosemide (Lasix) as prescribed.

ANS: A, B, D Discharge teaching to prevent renal complications in a child with myelomeningocele include: (1) regular urologic care with prompt and vigorous treatment of infections; (2) a method of regular emptying of the bladder, such as CIC taught to and performed by parents and self-catheterization taught to children; (3) medications to improve bladder storage and continence, such as oxybutynin chloride (Ditropan) and tolterodine (Detrol). Fluids should not be limited and Lasix is not used to improve renal function for children with myelomeningocele. DIF: Cognitive Level: Apply REF: p. 984 TOP: Integrated Process: Teaching/Learning MSC: Area of Client Needs: Physiologic Integrity: Reduction of Risk Potential

20. A child steps on a nail and sustains a puncture wound of the foot. Which is the most appropriate method for cleansing this wound? a. Wash wound thoroughly with chlorhexidine. b. Wash wound thoroughly with povidone-iodine. c. Soak foot in warm water and soap. d. Soak foot in solution of 50% hydrogen peroxide and 50% water.

ANS: C Puncture wounds should be cleansed by soaking the foot in warm water and soap. Chlorhexidine, hydrogen peroxide, and povidone-iodine should not be used because they have a cytotoxic effect on healthy cells and minimal effect on controlling infection. DIF: Cognitive Level: Apply REF: p. 998 TOP: Integrated Process: Nursing Process: Implementation MSC: Area of Client Needs: Physiologic Integrity: Reduction of Risk Potential

1. A 14-year-old girl is in the intensive care unit after a spinal cord injury 2 days ago. Nursing care for this child includes which action(s)? (Select all that apply.) a. Monitoring and maintaining systemic blood pressure b. Administering corticosteroids c. Minimizing environmental stimuli d. Discussing long-term care issues with the family e. Monitoring for respiratory complications

ANS: A, B, E Spinal cord injury patients are physiologically labile, and close monitoring is required. They may be unstable for the first few weeks after the injury. Corticosteroids are administered to minimize the inflammation present with the injury. It is not necessary to minimize environmental stimuli for this type of injury. Discussing long-term care issues with the family is inappropriate. The family is focusing on the recovery of their child. It will not be known until the rehabilitation period how much function the child may recover. DIF: Cognitive Level: Apply REF: p. 1000 TOP: Integrated Process: Nursing Process: Implementation MSC: Area of Client Needs: Physiologic Integrity: Physiologic Adaptation

3. A nurse is planning preoperative care for a newborn with tracheoesophageal fistula (TEF). Which interventions should the nurse plan to implement? (Select all that apply.) a. Positioning with head elevated on a 30-degree plane b. Feedings through a gastrostomy tube c. Nasogastric tube to continuous low wall suction d. Suctioning with a Replogle tube passed orally to the end of the pouch e. Gastrostomy tube to gravity drainage

ANS: A, D, E The most desirable position for a newborn who has TEF is supine (or sometimes prone) with the head elevated on an inclined plane of at least 30 degrees. This positioning minimizes the reflux of gastric secretions at the distal esophagus into the trachea and bronchi, especially when intra-abdominal pressure is elevated. It is imperative to immediately remove any secretions that can be aspirated. Until surgery, the blind pouch is kept empty by intermittent or continuous suction through an indwelling double-lumen or Replogle catheter passed orally or nasally to the end of the pouch. In some cases, a percutaneous gastrostomy tube is inserted and left open so that any air entering the stomach through the fistula can escape, thus minimizing the danger of gastric contents being regurgitated into the trachea. The gastrostomy tube is emptied by gravity drainage. Feedings through the gastrostomy tube and irrigations with fluid are contraindicated before surgery in an infant with a distal TEF. A nasogastric tube to low intermittent suctioning could not be accomplished because the esophagus ends in a blind pouch in TEF. DIF: Cognitive Level: Apply REF: p. 725 TOP: Integrated Process: Nursing Process: Planning MSC: Area of Client Needs: Physiologic Integrity: Reduction of Risk Potential

17. Chelation therapy is begun on a child with b-thalassemia major. What is the purpose of this therapy? a. Treat the disease b. Eliminate excess iron c. Decrease risk of hypoxia d. Manage nausea and vomiting

ANS: B A complication of the frequent blood transfusions in thalassemia is iron overload. Chelation therapy with deferoxamine (an iron-chelating agent) is given with oral supplements of vitamin C to increase iron excretion. Chelation therapy treats the side effect of the disease management. Decreasing the risk of hypoxia and managing nausea and vomiting are not the purposes of chelation therapy. DIF: Cognitive Level: Understand REF: p. 799 TOP: Integrated Process: Nursing Process: Implementation MSC: Area of Client Needs: Physiologic Integrity: Pharmacologic and Parenteral Therapies

7. A young boy will receive a bone marrow transplant (BMT). This is possible because one of his older siblings is a histocompatible donor. Which is this type of BMT called? a. Syngeneic b. Allogeneic c. Monoclonal d. Autologous

ANS: B Allogeneic transplants are from another individual. Because he and his sibling are histocompatible, the BMT can be done. Syngeneic marrow is from an identical twin. There is no such thing as a monoclonal BMT. Autologous refers to the individual's own marrow. DIF: Cognitive Level: Understand REF: p. 824 TOP: Integrated Process: Nursing Process: Planning MSC: Area of Client Needs: Physiologic Integrity: Physiologic Adaptation

The nurse is examining 12-month-old Amy, who was brought to the clinic for persistent diaper rash. The nurse finds perianal inflammation with satellite lesions that cross the inguinal folds. This is most likely caused by which of the following? a. Impetigo b. Candida albicans c. Urine and feces d. Infrequent diapering

ANS: B C. albicans infection produces perianal inflammation and a maculopapular rash with satellite lesions that may cross the inguinal folds.

17. A neonate born with ambiguous genitalia is diagnosed with congenital adrenogenital hyperplasia. What does therapeutic management include? a. Administration of vitamin D b. Administration of cortisone c. Administration of stool softeners d. Administration of calcium carbonate

ANS: B Cortisone is administered to suppress the abnormally high secretions of adrenocorticotropic hormone (ACTH). This in turn inhibits the secretion of adrenocorticosteroid, which stems the progressive virilization. Vitamin D, stool softeners, and calcium carbonate have no role in the therapy of adrenogenital hyperplasia. DIF: Cognitive Level: Understand REF: p. 923 TOP: Integrated Process: Nursing Process: Implementation MSC: Area of Client Needs: Physiologic Integrity: Pharmacologic and Parenteral Therapies

18. What is the most common clinical manifestation(s) of brain tumors in children? a. Irritability b. Seizures c. Headaches and vomiting d. Fever and poor fine motor control

ANS: C Headaches, especially on awakening, and vomiting that is not related to feeding are the most common clinical manifestation(s) of brain tumors in children. Irritability, seizures, and fever and poor fine motor control are clinical manifestations of brain tumors, but headaches and vomiting are the most common. DIF: Cognitive Level: Understand REF: p. 831 TOP: Integrated Process: Nursing Process: Assessment MSC: Area of Client Needs: Physiologic Integrity: Physiologic Adaptation

The nurse should know that Lyme disease is which of the following? a. Difficult to prevent b. Easily treated with oral antibiotics in stages 1, 2, and 3 c. Caused by a spirochete that enters the skin through a tick bite d. Common in geographic areas where the soil contains the mycotic spores that cause the disease

ANS: C Lyme disease is caused by Borrelia burgdorferi, a spirochete spread by ticks. The early characteristic rash is erythema migrans.

Which of the following is the primary clinical manifestation of scabies? a. Edema b. Redness c. Pruritus d. Maceration

ANS: C Scabies is caused by the scabies mite. The inflammatory response and intense itching occur after the host has become sensitized to the mite. This occurs approximately 30 to 60 days after initial contact.

A 2-year-old child has been admitted to the hospital for management of pneumonia. The child is placed in an oxygen tent. Taking into consideration the child's age and developmental level and the treatment being administered, which statement is appropriate for the nurse to make to the parents? A. "He can play in the tent with his blocks and plush stuffed animals." B. "You can sit next to him and hold his hand through the tent, but he needs to remain inside of it." C. "At his age, separation anxiety is high, so bringing in the wool blanket that he usually sleeps with is a good idea." D. "Before you leave for the night, it is a good idea to rock him to sleep. He can be out of the tent for up to 30 minutes without any consequences."

B. "You can sit next to him and hold his hand through the tent, but he needs to remain inside of it." Rationale: Oxygen therapy is an important component of management of pneumonia and is effective only if it is used appropriately. It is important to maintain the toddler in the oxygen environment at all times. With the addition of oxygen therapy, the hospitalized toddler is at risk for increased anxiety. Attachment is critical to optimal growth and development of children, particularly in the infant and toddler years. Therefore sitting with the child and holding the child's hand is important. Wool blankets, stuffed toys, and many toy cars can produce sparks, which could lead to an oxygen tent's catching fire. It is important to educate parents and family members not to bring these types of objects to the hospital.

The mother of a 3-year-old is concerned because her child still is insisting on a bottle at nap time and at bedtime. Which is the most appropriate suggestion to the mother? A. Allow the bottle if it contains juice. B. Allow the bottle if it contains water. C. Do not allow the child to have the bottle. D. Allow the bottle during naps but not at bedtime.

B. Allow the bottle if it contains water Rationale: A toddler should never be allowed to fall asleep with a bottle containing milk, juice, soda pop, sweetened water, or any other sweet liquid because of the risk of nursing (bottle-mouth) caries. If a bottle is allowed at nap time or bedtime, it should contain only water.

Which car safety device should be used for a child who is 8 years old and is 4 feet tall? A. Seat belt B. Booster seat C. Rear-facing convertible seat D. Front-facing convertible seat

B. Booster Seat Rationale: All children whose weight or height is above the forward-facing limit for their car safety seat should use a belt-positioning booster seat until the vehicle seat belt fits properly, typically when they have reached 4 feet 9 inches in height and are between 8 and 12 years of age. Infants should ride in a car in a semireclined, rear-facing position in an infant-only seat or a convertible seat until they weigh at least 20 pounds and are at least 1 year of age. The transition point for switching to the forward-facing position is defined by the manufacturer of the convertible car safety seat but is generally at a body weight of 9 kg (20 pounds) and 1 year of age.

The nurse is monitoring a 3-month-old infant for signs of increased intracranial pressure. On palpation of the fontanels, the nurse notes that the anterior fontanel is soft and flat. On the basis of this finding, which nursing action is most appropriate? A. Increase oral fluids. B. Document the finding. C. Notify the health care provider (HCP). D. Elevate the head of the bed to 90 degrees.

B. Document the finding. Rationale: The anterior fontanel is diamond-shaped and located on the top of the head. The fontanel should be soft and flat in a normal infant, and it normally closes by 12 to 18 months of age. The nurse would document the finding because it is normal. There is no useful reason to increase oral fluids, notify the HCP, or elevate the head of the bed to 90 degrees.

The nurse assesses the vital signs of a 12-month-old infant with a respiratory infection and notes that the respiratory rate is 35 breaths/minute. On the basis of this finding, which action is most appropriate? A. Administer oxygen. B. Document the findings. C. Notify the health care provider. D. Reassess the respiratory rate in 15 minutes.

B. Document the findings Rationale: The normal respiratory rate in a 12-month-old infant is 20 to 40 breaths/minute. The normal apical heart rate is 90 to 130 beats/minute, and the average blood pressure is 90/56 mm Hg. The nurse would document the findings.

The nurse at a well-baby clinic is providing nutrition instructions to the mother of a 1-month-old infant. What instruction should the nurse give to the mother? A. To introduce strained fruits one at a time B. That breast milk or formula is the main food C. To introduce strained vegetables one at a time D. To offer rice cereal mixed with breast milk or formula

B. That breast milk or formula is the main food Rationale: Breast milk or formula is the main food throughout infancy. Rice cereal mixed with breast milk or formula is introduced at 4 months of age. Strained vegetables, fruits, and meats are introduced one at a time and can begin at 6 months of age.

The nurse is evaluating the developmental level of a 2-year-old. Which does the nurse expect to observe in this child? A. Uses a fork to eat B. Uses a cup to drink C. Pours own milk into a cup D. Uses a knife for cutting food

B. Uses a cup to drink Rationale: By age 2 years, the child can use a cup and spoon correctly but with some spilling. By age 3 to 4, the child begins to use a fork. By the end of the preschool period, the child should be able to pour milk into a cup and begin to use a knife for cutting.

The parents of a 2-year-old arrive at a hospital to visit their child. The child is in the playroom when the parents arrive. When the parents enter the playroom, the child does not readily approach the parents. Which is the correct interpretation of the behavior? A. The child is withdrawn. B. The child is self-centered. C. The child exhibits detachment. D. The child has adjusted to the hospital setting.

C. The child exhibits detachment. Rationale: The phases through which young children progress when separated from their parents include protest, despair, and denial or detachment. In detachment, when the parents return, the child becomes more interested in the environment and new persons (seemingly unaware of the lost parents), friendly with the staff, and interested in developing superficial relationships. In the stage of protest, the child may cry, scream, and search for a parent. In the stage of despair, the child may be withdrawn and uninterested in the environment. That the child is withdrawn, self-centered, or that the child has adjusted to the hospital setting are incorrect interpretations of the child's behavior.

The clinic nurse assesses the communication patterns of a 5-month-old infant. Which assessment finding should lead the nurse to determine that the infant is demonstrating the highest level of developmental achievement expected? A. Coos when comforted B. Links syllables together C. Uses monosyllabic babbling D. Uses simple words such as mama

C. Uses monosyllabic babbling Rationale: Using monosyllabic babbling occurs between 3 and 6 months of age. Cooing begins at birth and continues until 2 months of age. Linking syllables together when communicating occurs between 6 and 9 months of age. Using simple words such as mama occurs between 9 and 12 months of age.

A 15-year-old is injured and sustains a fractured jaw. The fractured jaw has been surgically wired, and the health care provider has prescribed a full liquid diet. Which nursing action would best promote compliance and provide an adequate nutrient value with the full liquid diet for this teenager? A. Offer chocolate milkshakes between meals. B. Explain the importance of good nutrition to the teenager. C. Offer commercial nutritional supplements four to six times per day. D. Ask the teenager for food preferences and blenderize these foods into liquids.

D. Ask the teenager for food preferences and blenderize these foods into liquids. Rationale: A 15-year-old may have difficulty maintaining compliance with a diet that is only liquids. To encourage compliance it is important to have the teenager participate in as much decision making in the diet as possible. Although blenderized foods may be unappealing under many circumstances, the nutrient value is unchanged. The teenager will have an opportunity to "eat" the same foods that he or she was eating before the jaw fracture. Option 1 may be beneficial but does not offer the teenager any choices. Teenagers may or may not respond to reasoning and explanations as with option 2. Commercial supplements also are beneficial nutritional sources but will not be effective unless the client is willing to drink them.

The nurse in the pediatric unit is admitting a 2½-year-old child. The nurse plans care, knowing that the child is in which stage of Erikson's psychosocial stages of development? A. Trust versus Mistrust B. Initiative versus Guilt C. Industry versus Inferiority D. Autonomy versus Shame and Doubt

D. Autonomy versus Shame and Doubt Rationale: A 2½-year-old child, a toddler, is in the Autonomy versus Shame and Doubt stage. In this stage the toddler develops a sense of control over the self and bodily functions. Trust versus Mistrust characterizes the stage of infancy. Initiative versus Guilt characterizes the preschool age. Industry versus Inferiority characterizes the school-aged child.

The nurse is caring for a 4-year-old child with human immunodeficiency virus (HIV) infection. The nurse should plan care with the understanding that which childhood psychosocial need occurs at this age? A. Expressing fear, withdrawal, and denial B. Beginning to understand that something is wrong C. Unable to grasp the concept of illness and death D. Beginning to conceptualize the death process as involving physical harm

D. Beginning to conceptualize the death process as involving physical harm Rationale: A preschool child begins to conceptualize the death process as involving physical harm. A child from birth to 2 years of age is unable to grasp the concept of illness and death. A school-age child begins to understand that something is wrong. An adolescent expresses fear, withdrawal, and denial.

A mother arrives at a clinic with her toddler and tells the nurse that she has a difficult time getting the child to go to bed at night. What measure is most appropriate for the nurse to suggest to the mother? A. Allow the child to set bedtime limits. B. Allow the child to have temper tantrums. C. Avoid letting the child nap during the day. D. Inform the child of bedtime a few minutes before it is time for bed.

D. Inform the child of bedtime a few minutes before it is time for bed. Rationale: Toddlers often resist going to bed. Bedtime protests may be reduced by establishing a consistent before-bedtime routine and enforcing consistent limits regarding the child's bedtime behavior. Informing the child of bedtime a few minutes before it is time for bed is the most appropriate option. Most toddlers take an afternoon nap and, until their second birthday, also may require a morning nap. Firm, consistent limits are needed for temper tantrums or when toddlers try stalling tactics.

The nurse is admitting a 10-month-old infant who is being hospitalized for a respiratory infection. The nurse develops a plan of care for the infant and includes which most appropriate intervention? A. Keeping the infant as quiet as possible B. Restraining the infant to prevent dislodging of tubes C. Placing small toys in the crib to provide stimulation for the infant D. Providing a consistent routine with touching, rocking, and cuddling throughout hospitalization

D. Providing a consistent routine with touching, rocking, and cuddling throughout hospitalization Rationale: A 10-month-old is in the Trust versus Mistrust stage of psychosocial development according to Erikson. The infant is developing a sense of self, and the nurse should appropriately provide a consistent routine for the child. Hospitalization may have an adverse effect, and the nurse should touch, rock, and cuddle the infant to promote a sense of trust and provide sensory stimulation. Keeping the infant as quiet as possible will not provide sensory stimulation. The infant should not be restrained. Placing small toys in the crib is an unsafe action

The nurse at a well-baby clinic is assessing the motor development of a 24-month-old child. On the basis of the age of the child, the nurse expects to note what as the highest-level developmental milestone? A. The child snaps large snaps. B. The child builds a tower of two blocks. C. The child puts on simple clothes independently. D. The child opens a door by turning the doorknob.

D. The child opens a door by turning the doorknob. Rationale: A 24-month-old child should be able to open a door using the doorknob. At age 15 months, the nurse should expect that the child could build a tower of two blocks. At age 30 months, the child should be able to snap large snaps and put on simple clothes independently.

26. A nurse is admitting a child with Crohn disease. Parents ask the nurse, "How is this disease different from ulcerative colitis?" Which statement should the nurse make when answering this question? a. "With Crohn disease the inflammatory process involves the whole GI tract." b. "There is no difference between the two diseases." c. "The inflammation with Crohn disease is limited to the colon and rectum." d. "Ulcerative colitis is characterized by skip lesions."

a. "With Crohn disease the inflammatory process involves the whole GI tract." ANS: A The chronic inflammatory process of Crohn disease involves any part of the GI tract from the mouth to the anus but most often affects the terminal ileum. Crohn disease involves all layers of the bowel wall in a discontinuous fashion, meaning that between areas of intact mucosa, there are areas of affected mucosa (skip lesions). The inflammation found with ulcerative colitis is limited to the colon and rectum, with the distal colon and rectum the most severely affected. Inflammation affects the mucosa and submucosa and involves continuous segments along the length of the bowel with varying degrees of ulceration, bleeding, and edema. DIF: Cognitive Level: Apply REF: p. 713 TOP: Integrated Process: Teaching/Learning MSC: Area of Client Needs: Physiologic Integrity: Physiologic Adaptation

20. The nurse is admitting a child with a Wilms tumor. Which is the initial assessment finding associated with this tumor? a. Abdominal swelling b. Weight gain c. Hypotension d. Increased urinary output

a. Abdominal swelling ANS: A The initial assessment finding with a Wilms (kidney) tumor is abdominal swelling. Weight loss, not weight gain, may be a finding. Hypertension occasionally occurs with a Wilms tumor. Urinary output is not increased, but hematuria may be noted. PTS: 1 DIF: Cognitive Level: Understand REF: 917 TOP: Integrated Process: Nursing Process: Assessment MSC: Area of Client Needs: Physiologic Integrity: Physiologic Adaptation

2. A school-age child is admitted to the hospital with acute glomerulonephritis and oliguria. Which dietary menu items should be allowed for this child? (Select all that apply.) a. Apples b. Bananas c. Cheese d. Carrot sticks e. Strawberries

a. Apples d. Carrot sticks e. Strawberries ANS: A, D, E Moderate sodium restriction and even fluid restriction may be instituted for children with acute glomerulonephritis. Foods with substantial amounts of potassium are generally restricted during the period of oliguria. Apples, carrot sticks, and strawberries would be items low in sodium and allowed. Bananas are high in potassium and cheese is high in sodium. Those items would be restricted. PTS: 1 DIF: Cognitive Level: Apply REF: 916 TOP: Integrated Process: Nursing Process: Planning MSC: Area of Client Needs: Physiologic Integrity: Reduction of Risk Potential

MULTIPLE RESPONSE 1. A child who has just had definitive repair of a high rectal malformation is to be discharged. Which should the nurse address in the discharge preparation of this family? (Select all that apply.) a. Perineal and wound care b. Necessity of firm stools to keep suture line clean c. Bowel training beginning as soon as child returns home d. Reporting any changes in stooling patterns to practitioner e. Use of diet modification to prevent constipation

a. Perineal and wound care d. Reporting any changes in stooling patterns to practitioner e. Use of diet modification to prevent constipation ANS: A, D, E Wound care instruction is necessary in a child who is being discharged after surgery. The parents are taught to notify the practitioner if any signs of an anal stricture or other complications develop. Constipation is avoided, since a firm stool will place strain on the suture line. Fiber and stool softeners are often given to keep stools soft and avoid tension on the suture line. The child needs to recover from the surgical procedure. Then bowel training may begin, depending on the child's developmental and physiologic readiness. DIF: Cognitive Level: Apply REF: p. 732 TOP: Integrated Process: Teaching/Learning MSC: Area of Client Needs: Physiologic Integrity: Reduction of Risk Potential

12. A hospitalized child with minimal change nephrotic syndrome is receiving high doses of prednisone. Which is an appropriate nursing goal related to this? a. Prevent infection. b. Stimulate appetite. c. Detect evidence of edema. d. Ensure compliance with prophylactic antibiotic therapy.

a. Prevent infection. ANS: A High-dose steroid therapy has an immunosuppressant effect. These children are particularly vulnerable to upper respiratory tract infections. A priority nursing goal is to minimize the risk of infection by protecting the child from contact with infectious individuals. Appetite is increased with prednisone therapy. The amount of edema should be monitored as part of the disease process, not necessarily related to the administration of prednisone. Antibiotics would not be used as prophylaxis. pg 860

15. Which is a high-fiber food that the nurse should recommend for a child with chronic constipation? a. Raisins b. Pancakes c. Muffins d. Ripe bananas

a. Raisins ANS: A Raisins are a high-fiber food. Pancakes and muffins do not have significant fiber unless made with fruit or bran. Raw fruits, especially those with skins and seeds, other than ripe bananas, have high fiber. DIF: Cognitive Level: Apply REF: p. 703 TOP: Integrated Process: Teaching/Learning MSC: Area of Client Needs: Physiologic Integrity: Basic Care and Comfort

46. Which is an important nursing consideration in the care of a child with celiac disease? a. Refer to a nutritionist for detailed dietary instructions and education. b. Help child and family understand that diet restrictions are usually only temporary. c. Teach proper hand washing and standard precautions to prevent disease transmission. d. Suggest ways to cope more effectively with stress to minimize symptoms.

a. Refer to a nutritionist for detailed dietary instructions and education. ANS: A The main consideration is helping the child adhere to dietary management. Considerable time is spent explaining to the child and parents about the disease process, the specific role of gluten in aggravating the condition, and foods that must be restricted. Referral to a nutritionist would help in this process. The most severe symptoms usually occur in early childhood and adult life. Dietary avoidance of gluten should be lifelong. Celiac disease is not transmissible or stress related. DIF: Cognitive Level: Apply REF: p. 732 TOP: Integrated Process: Teaching/Learning MSC: Area of Client Needs: Physiologic Integrity: Basic Care and Comfort

50. A child has recurrent abdominal pain (RAP) and a dairy-free diet has been prescribed for 2 weeks. Which explanation is the reason for prescribing a dairy-free diet? a. To rule out lactose intolerance b. To rule out celiac disease c. To rule out sensitivity to high sugar content d. To rule out peptic ulcer disease

a. To rule out lactose intolerance ANS: A Treatment for RAP involves providing reassurance and reducing or eliminating symptoms. Dietary modifications may include removal of dairy products to rule out lactose intolerance. Fructose is eliminated to rule out sensitivity to high sugar content, and gluten is removed to rule out celiac disease. A dairy-free diet would not rule out peptic ulcer disease. DIF: Cognitive Level: Understand REF: p. 734 TOP: Integrated Process: Nursing Process: Assessment MSC: Area of Client Needs: Physiologic Integrity: Physiologic Adaptation

5. Which should the nurse recommend to prevent urinary tract infections in young girls? a. Wear cotton underpants. b. Limit bathing as much as possible. c. Increase fluids; decrease salt intake. d. Cleanse perineum with water after voiding.

a. Wear cotton underpants. ANS: A Cotton underpants are preferable to nylon underpants. No evidence exists that limiting bathing, increasing fluids/decreasing salt intake, or cleansing the perineum with water after voiding decrease urinary tract infections in young girls. PTS: 1 DIF: Cognitive Level: Apply REF: 910 TOP: Integrated Process: Teaching/Learning MSC: Area of Client Needs: Physiologic Integrity: Reduction of Risk Potential

24. When a child has chronic renal failure, the progressive deterioration produces a variety of clinical and biochemical disturbances that eventually are manifested in the clinical syndrome known as: a. uremia. b. oliguria. c. proteinuria. d. pyelonephritis.

a. uremia. ANS: A Uremia is the retention of nitrogenous products, producing toxic symptoms. Oliguria is diminished urinary output. Proteinuria is the presence of protein, usually albumin, in the urine. Pyelonephritis is an inflammation of the kidney and renal pelvis. PTS: 1 DIF: Cognitive Level: Remember REF: 921 TOP: Integrated Process: Nursing Process: Evaluation MSC: Area of Client Needs: Physiologic Integrity: Physiologic Adaptation

19. The nurse is talking to a parent of an infant with heart failure about feeding the infant. Which statement about feeding the child is correct? a. "You may need to increase the caloric density of your infant's formula." b. "You should feed your baby every 2 hours." c. "You may need to increase the amount of formula your infant eats with each feeding." d. "You should place a nasal oxygen cannula on your infant during and after each feeding."

a. "You may need to increase the caloric density of your infant's formula." ANS: A The metabolic rate of infants with heart failure is greater because of poor cardiac function and increased heart and respiratory rates. Their caloric needs are greater than those of the average infants, yet their ability to take in the calories is diminished by their fatigue. Infants with heart failure should be fed every 3 hours; a 2-hour schedule does not allow for enough rest, and a 4-hour schedule is too long. Fluids must be carefully monitored because of the heart failure. Infants do not require supplemental oxygen with feedings. DIF: Cognitive Level: Apply REF: p. 754 TOP: Integrated Process: Nursing Process: Planning MSC: Area of Client Needs: Physiologic Integrity: Physiologic Adaptation

25. An adolescent with osteosarcoma is scheduled for a leg amputation in 2 days. The nurse's approach should include which action? a. Answering questions with straightforward honesty b. Avoiding discussing the seriousness of the condition c. Explaining that, although the amputation is difficult, it will cure the cancer d. Assisting the adolescent in accepting the amputation as better than a long course of chemotherapy

a. Answering questions with straightforward honesty ANS: A Honesty is essential to gain the child's cooperation and trust. The diagnosis of cancer should not be disguised with falsehoods. The adolescent should be prepared for the surgery so he or she has time to reflect on the diagnosis and subsequent treatment. This allows questions to be answered. To accept the need for radical surgery, the child must be aware of the lack of alternatives for treatment. Amputation is necessary, but it will not guarantee a cure. Chemotherapy is an integral part of the therapy with surgery. The child should be informed of the need for chemotherapy and its side effects before surgery. DIF: Cognitive Level: Apply REF: p. 836 TOP: Integrated Process: Teaching/Learning MSC: Area of Client Needs: Health Promotion and Maintenance

18. In which of the conditions are all the formed elements of the blood simultaneously depressed? a. Aplastic anemia b. Sickle cell anemia c. Thalassemia major d. Iron-deficiency anemia

a. Aplastic anemia' ANS: A Aplastic anemia refers to a bone marrow-failure condition in which the formed elements of the blood are simultaneously depressed. Sickle cell anemia is a hemoglobinopathy in which normal adult hemoglobin is partly or completely replaced by abnormal sickle hemoglobin. Thalassemia major is a group of blood disorders characterized by deficiency in the production rate of specific hemoglobin globin chains. Iron- deficiency anemia results in a decreased amount of circulating red cells. DIF: Cognitive Level: Understand REF: p. 800 TOP: Integrated Process: Nursing Process: Assessment MSC: Area of Client Needs: Physiologic Integrity: Physiologic Adaptation

3. A child with growth hormone (GH) deficiency is receiving GH therapy. When is the best time for the GH to be administered? a. At bedtime b. After meals c. Before meals d. On arising in the morning

a. At bedtime ANS: A Injections are best given at bedtime to more closely approximate the physiologic release of GH. After or before meals and on arising in the morning do not mimic the physiologic release of the hormone. DIF: Cognitive Level: Apply REF: p. 911 TOP: Integrated Process: Nursing Process: Implementation MSC: Area of Client Needs: Physiologic Integrity: Pharmacologic and Parenteral Therapies

10. Latex allergy is suspected in a child with spina bifida. Appropriate nursing interventions include which action? a. Avoid using any latex product. b. Use only nonallergenic latex products. c. Administer medication for long-term desensitization. d. Teach family about long-term management of asthma.

a. Avoid using any latex product. ANS: A Care must be taken that individuals who are at high risk for latex allergies do not come in direct or secondary contact with products or equipment containing latex at any time during medical treatment. There are no nonallergenic latex products. At this time, desensitization is not an option. The child does not have asthma. The parents must be taught about allergy and the risk of anaphylaxis. DIF: Cognitive Level: Apply REF: p. 990 TOP: Integrated Process: Nursing Process: Implementation MSC: Area of Client Needs: Physiologic Integrity: Reduction of Risk Potential

31. The nurse is implementing care for a school-age child admitted to the pediatric intensive care in diabetic ketoacidosis (DKA). Which prescribed intervention should the nurse implement first? a. Begin 0.9% saline solution intravenously as prescribed. b. Administer regular insulin intravenously as prescribed. c. Place child on a cardiac monitor. d. Place child on a pulse oximetry monitor.

a. Begin 0.9% saline solution intravenously as prescribed. ANS: A All patients with DKA experience dehydration (10% of total body weight in severe ketoacidosis) because of the osmotic diuresis, accompanied by depletion of electrolytes, sodium, potassium, chloride, phosphate, and magnesium. The initial hydrating solution is 0.9% saline solution. Insulin therapy should be started after the initial rehydration bolus because serum glucose levels fall rapidly after volume expansion. The child should be placed on the cardiac and pulse oximetry monitor after the rehydrating solution has been initiated. DIF: Cognitive Level: Analyze REF: p. 933 TOP: Integrated Process: Nursing Process: Implementation MSC: Area of Client Needs: Physiologic Integrity: Physiologic Adaptation

17. An adolescent boy is brought to the emergency department after a motorcycle accident. His respirations are deep, periodic, and gasping. There are extreme fluctuations in blood pressure. Pupils are dilated and fixed. The nurse should suspect which type of head injury? a. Brainstem b. Skull fracture c. Subdural hemorrhage d. Epidural hemorrhage

a. Brainstem ANS: A Signs of brainstem injury include deep, rapid, periodic or intermittent, and gasping respirations. Wide fluctuations or noticeable slowing of the pulse, widening pulse pressure, or extreme fluctuations in blood pressure are consistent with a brainstem injury. Skull fracture, subdural hemorrhage, and epidural hemorrhage are not consistent with brainstem injuries. DIF: Cognitive Level: Understand REF: p. 887 TOP: Integrated Process: Nursing Process: Assessment MSC: Area of Client Needs: Physiologic Integrity: Physiologic Adaptation

38. Which clinical manifestations would suggest hydrocephalus in a neonate? a. Bulging fontanel and dilated scalp veins b. Closed fontanel and high-pitched cry c. Constant low-pitched cry and restlessness d. Depressed fontanel and decreased blood pressure

a. Bulging fontanel and dilated scalp veins ANS: A Bulging fontanels, dilated scalp veins, and separated sutures are clinical manifestations of hydrocephalus in neonates. Closed fontanel and high-pitched cry, constant low-pitched cry and restlessness, and depressed fontanel and decreased blood pressure are not clinical manifestations of hydrocephalus, but all should be referred for evaluation. DIF: Cognitive Level: Analyze REF: p. 906 TOP: Integrated Process: Nursing Process: Assessment MSC: Area of Client Needs: Physiologic Integrity: Physiologic Adaptation

14. A nurse is preparing to administer an angiotensin-converting enzyme (ACE) inhibitor. Which drug should the nurse administer? a. Captopril (Capoten) b. Furosemide (Lasix) c. Spironolactone (Aldactone) d. Chlorothiazide (Diuril)

a. Captopril (Capoten) ANS: A Captopril is an ACE inhibitor. Furosemide is a loop diuretic. Spironolactone blocks the action of aldosterone. Chlorothiazide works on the distal tubules. DIF: Cognitive Level: Remember REF: p. 752 TOP: Integrated Process: Nursing Process: Implementation MSC: Area of Client Needs: Physiologic Integrity: Pharmacologic and Parenteral Therapies

1. The nurse is preparing to give oral care to a school-age child with mucositis secondary to chemotherapy administered to treat leukemia. Which preparations should the nurse use for oral care on this child? (Select all that apply.) a. Chlorhexidine gluconate (Peridex) b. Lemon glycerin swabs c. Antifungal troches (lozenges) d. Lip balm (Aquaphor) e. Hydrogen peroxide

a. Chlorhexidine gluconate (Peridex) c. Antifungal troches (lozenges) d. Lip balm (Aquaphor) ANS: A, C, D Preparations that may be used to prevent or treat mucositis include chlorhexidine gluconate (Peridex) because of its dual effectiveness against candidal and bacterial infections, antifungal troches (lozenges) or mouthwash, and lip balm (e.g., Aquaphor) to keep the lips moist. Agents that should not be used include lemon glycerin swabs (irritate eroded tissue and can decay teeth), hydrogen peroxide (delays healing by breaking down protein), and milk of magnesia (dries mucosa). DIF: Cognitive Level: Apply REF: p. 819 TOP: Integrated Process: Nursing Process: Implementation MSC: Area of Client Needs: Physiologic Integrity: Physiologic Adaptation

34. What is an important nursing intervention when caring for a child who is experiencing a seizure? a. Describe and record the seizure activity observed. b. Restrain the child when seizure occurs to prevent bodily harm. c. Place a tongue blade between the teeth if they become clenched. d. Suction the child during a seizure to prevent aspiration.

a. Describe and record the seizure activity observed. ANS: A When a child is having a seizure, the priority nursing care is observation of the child and seizure. The nurse then describes and records the seizure activity. The child should not be restrained, and nothing should be placed in the child's mouth. This may cause injury. To prevent aspiration, if possible, the child should be placed on the side, facilitating drainage. DIF: Cognitive Level: Apply REF: p. 896 TOP: Integrated Process: Nursing Process: Implementation MSC: Area of Client Needs: Physiologic Integrity: Physiologic Adaptation

19. What is a possible cause of acquired aplastic anemia in children? a. Drugs b. Injury c. Deficient diet d. Congenital defect

a. Drugs ANS: A Drugs, such as chemotherapeutic agents and several antibiotics (e.g., chloramphenicol), can cause aplastic anemia. Injury, deficient diet, and congenital defect are not causative agents in acquired aplastic anemia. DIF: Cognitive Level: Understand REF: p. 800 TOP: Integrated Process: Nursing Process: Assessment MSC: Area of Client Needs: Physiologic Integrity: Pharmacologic and Parenteral Therapies

9. Which is the priority nursing intervention for an unconscious child after a fall? a. Establish adequate airway. b. Perform neurologic assessment. c. Monitor intracranial pressure. d. Determine whether a neck injury is present.

a. Establish adequate airway. ANS: A Respiratory effectiveness is the primary concern in the care of the unconscious child. Establishment of an adequate airway is always the first priority. A neurologic assessment and determination of whether a neck injury is present will be performed after breathing and circulation are stabilized. Intracranial, not intercranial, pressure is monitored if indicated after airway, breathing, and circulation are maintained. DIF: Cognitive Level: Apply REF: p. 879 TOP: Integrated Process: Nursing Process: Implementation MSC: Area of Client Needs: Physiologic Integrity: Physiologic Adaptation

23. A nurse is conducting a staff in-service on childhood cancers. Which is the primary site of osteosarcoma? a. Femur b. Humerus c. Pelvis d. Tibia

a. Femur ANS: A Osteosarcoma is the most frequently encountered malignant bone cancer in children. The peak incidence is between ages 10 and 25 years. More than half occur in the femur. After the femur, most of the remaining sites are the humerus, tibia, pelvis, jaw, and phalanges. DIF: Cognitive Level: Understand REF: p. 836 TOP: Integrated Process: Teaching/Learning MSC: Area of Client Needs: Physiologic Integrity: Reduction of Risk Potential

A nurse enters the room of an 8-year-old child newly admitted and diagnosed with type I diabetes. His mother is sitting in a chair at his bedside. What should the nurse do first? a. Go the bedside and meet the child. b. Stand by the door, and say "I am the assigned nurse today." c. Go over to the mother and ask what brought the child into the hospital today. d. Explain the use of the call light to child.

a. Go the bedside and meet the child.

13. The nurse is preparing to administer a dose of digoxin (Lanoxin) to a child in heart failure (HF). Which is a beneficial effect of administering digoxin (Lanoxin)? a. It decreases edema. b. It decreases cardiac output. c. It increases heart size. d. It increases venous pressure.

a. It decreases edema ANS: A Digoxin has a rapid onset and is useful for increasing cardiac output, decreasing venous pressure, and, as a result, decreasing edema. Cardiac output is increased by digoxin. Heart size and venous pressure are decreased by digoxin. DIF: Cognitive Level: Understand REF: p. 752 TOP: Integrated Process: Nursing Process: Implementation MSC: Area of Client Needs: Physiologic Integrity: Pharmacologic and Parenteral Therapies.

10. Which drug should the nurse expect to administer to a preschool child who has increased intracranial pressure (ICP) resulting from cerebral edema? a. Mannitol (Osmitrol) b. Epinephrine hydrochloride (Adrenalin) c. Atropine sulfate (Atropine) d. Sodium bicarbonate (Sodium bicarbonate)

a. Mannitol (Osmitrol) ANS: A For increased ICP, mannitol, an osmotic diuretic, administered intravenously, is the drug used most frequently for rapid reduction. Epinephrine hydrochloride, atropine sulfate, and sodium bicarbonate are not used to decrease ICP. DIF: Cognitive Level: Apply REF: p. 879 TOP: Integrated Process: Nursing Process: Implementation

4. The nurse is teaching parents about the importance of iron in a toddler's diet. Which explains why iron-deficiency anemia is common during toddlerhood? a. Milk is a poor source of iron. b. Iron cannot be stored during fetal development. c. Fetal iron stores are depleted by age 1 month. d. Dietary iron cannot be started until age 12 months.

a. Milk is a poor source of iron. ANS: A Children between the ages of 12 and 36 months are at risk for anemia because cow's milk is a major component of their diet and it is a poor source of iron. Iron is stored during fetal development, but the amount stored depends on maternal iron stores. Fetal iron stores are usually depleted by age 5 to 6 months. Dietary iron can be introduced by breastfeeding, iron-fortified formula, and cereals during the first 12 months of life. DIF: Cognitive Level: Understand REF: p. 789 TOP: Integrated Process: Teaching/Learning MSC: Area of Client Needs: Physiologic Integrity: Physiologic Adaptation

28. When caring for the child with Reye syndrome, what is the priority nursing intervention? a. Monitor intake and output b. Prevent skin breakdown c. Observe for petechiae d. Do range-of-motion exercises

a. Monitor intake and output ANS: A Accurate and frequent monitoring of intake and output is essential for adjusting fluid volumes to prevent both dehydration and cerebral edema. Preventing skin breakdown, observing for petechiae, and doing range-of-motion exercises are important interventions in the care of a critically ill or comatose child. Careful monitoring of intake and output is a priority. DIF: Cognitive Level: Apply REF: p. 895 TOP: Integrated Process: Nursing Process: Implementation MSC: Area of Client Needs: Physiologic Integrity: Physiologic Adaptation

5. The nurse is teaching a group of nursing students about newborns born with the congenital defect of myelomeningocele. Which common problem is associated with this defect? a. Neurogenic bladder b. Cognitive impairment c. Respiratory compromise d. Cranioschisis

a. Neurogenic bladder ANS: A Myelomeningocele is one of the most common causes of neuropathic (neurogenic) bladder dysfunction among children. Risk of cognitive impairment is minimized through early intervention and management of hydrocephalus. Respiratory compromise is not a common problem in myelomeningocele. Cranioschisis is a skull defect through which various tissues protrude. It is not associated with myelomeningocele. DIF: Cognitive Level: Understand REF: p. 987 TOP: Integrated Process: Teaching/Learning MSC: Area of Client Needs: Physiologic Integrity: Physiologic Adaptation

9. Parents of a child with sickle cell anemia ask the nurse, "What happens to the hemoglobin in sickle cell anemia?" Which statement by the nurse explains the disease process? a. Normal adult hemoglobin is replaced by abnormal hemoglobin. b. There is a lack of cellular hemoglobin being produced. c. There is a deficiency in the production of globulin chains. d. The size and depth of the hemoglobin are affected.

a. Normal adult hemoglobin is replaced by abnormal hemoglobin ANS: A Sickle cell anemia is one of a group of diseases collectively called hemoglobinopathies, in which normal adult hemoglobin is replaced by abnormal hemoglobin. Aplastic anemia is a lack of cellular elements being produced. Thalassemia major refers to a variety of inherited disorders characterized by deficiencies in production of certain globulin chains. Iron-deficiency anemia affects the size, depth, and color of hemoglobin. DIF: Cognitive Level: Apply REF: p. 791 TOP: Integrated Process: Teaching/Learning MSC: Area of Client Needs: Physiologic Integrity: Physiologic Adaptation.

6. The nurse is taking care of a child who is alert but showing signs of increased intracranial pressure. Which test is contraindicated in this case? a. Oculovestibular response b. Doll's head maneuver c. Funduscopic examination for papilledema d. Assessment of pyramidal tract lesions

a. Oculovestibular response ANS: A The oculovestibular response (caloric test) involves the instillation of ice water into the ear of a comatose child. The caloric test is painful and is never performed on a child who is awake or one who has a ruptured tympanic membrane. Doll's head maneuver, funduscopic examination for papilledema, and assessment of pyramidal tract lesions can be performed on children who are awake. DIF: Cognitive Level: Analyze REF: p. 887 TOP: Integrated Process: Nursing Process: Implementation MSC: Area of Client Needs: Physiologic Integrity: Physiologic Adaptation

30. Which painful, tender, pea-sized nodules may appear on the pads of the fingers or toes in bacterial endocarditis? a. Osler nodes b. Janeway lesions c. Subcutaneous nodules d. Aschoff nodes

a. Osler nodes ANS: A Osler nodes are red, painful, intradermal nodes found on pads of the phalanges in bacterial endocarditis. Janeway lesions are painless hemorrhagic areas on palms and soles in bacterial endocarditis. Subcutaneous nodules are nontender swellings, located over bony prominences, commonly found in rheumatic fever. Aschoff nodules are small nodules composed of cells and leukocytes found in the interstitial tissues of the heart in rheumatic myocarditis. DIF: Cognitive Level: Understand REF: p. 766 TOP: Integrated Process: Nursing Process: Assessment MSC: Area of Client Needs: Physiologic Integrity: Physiologic Adaptation

33. The nurse is conducting a staff in-service on childhood-acquired heart diseases. Which is a major clinical manifestation of rheumatic fever? a. Polyarthritis b. Osler nodes c. Janeway spots d. Splinter hemorrhages of distal third of nails

a. Polyarthritis ANS: A Polyarthritis, which is swollen, hot, red, and painful joints, is a major clinical manifestation of rheumatic fever. The affected joints will change every 1 to 2 days. Primarily the large joints are affected. Osler nodes, Janeway spots, and splinter hemorrhages are characteristic of infective endocarditis. DIF: Cognitive Level: Apply REF: p. 767 TOP: Integrated Process: Teaching/Learning MSC: Area of Client Needs: Physiologic Integrity: Physiologic Adaptation

10. A goiter is an enlargement or hypertrophy of which gland? a. Thyroid b. Adrenal c. Anterior pituitary d. Posterior pituitary

a. Thyroid ANS: A A goiter is an enlargement or hypertrophy of the thyroid gland. Goiter is not associated with the adrenal, anterior pituitary, or posterior pituitary organs. DIF: Cognitive Level: Remember REF: p. 919 TOP: Integrated Process: Nursing Process: Assessment MSC: Area of Client Needs: Physiologic Integrity: Physiologic Adaptation

25. The nurse is planning care for an adolescent with AIDS. Which is the priority nursing goal? a. Preventing infection b. Preventing secondary cancers c. Restoring immunologic defenses d. Identifying source of infection

a. Preventing infection ANS: A Because the child is immunocompromised in association with HIV infection, the prevention of infection is paramount. Although certain precautions are justified in limiting exposure to infection, these must be balanced with the concern for the child's normal developmental needs. Preventing secondary cancers is not currently possible. Current drug therapy is affecting the disease progression; although not a cure, these drugs can suppress viral replication, preventing further deterioration. Case finding is not a priority nursing goal. DIF: Cognitive Level: Apply REF: p. 806 TOP: Integrated Process: Nursing Process: Planning MSC: Area of Client Needs: Physiologic Integrity: Reduction of Risk Potential

13. A 4-year-old child has just been diagnosed with pseudohypertrophic (Duchenne) muscular dystrophy. The management plan should include which action? a. Recommend genetic counseling. b. Explain that the disease is easily treated. c. Suggest ways to limit use of muscles. d. Assist family in finding a nursing facility to provide child's care.

a. Recommend genetic counseling. ANS: A Pseudohypertrophic (Duchenne) muscular dystrophy is inherited as an X-linked recessive gene. Genetic counseling is recommended for parents, female siblings, maternal aunts, and their female offspring. No effective treatment exists at this time for childhood muscular dystrophy. Maintaining optimal function of all muscles for as long as possible is the primary goal. It has been found that children who remain as active as possible are able to avoid wheelchair confinement for a longer time. Assisting the family in finding a nursing facility to provide the child's care is inappropriate at the time of diagnosis. When the child becomes increasingly incapacitated, the family may consider home-based care, a skilled nursing facility, or respite care to provide the necessary care. DIF: Cognitive Level: Understand REF: p. 992 TOP: Integrated Process: Nursing Process: Implementation MSC: Area of Client Needs: Physiologic Integrity: Reduction of Risk Potential

13. A young child with leukemia has anorexia and severe stomatitis. The nurse should suggest that the parents try which intervention? a. Relax any eating pressures. b. Firmly insist that child eat normally. c. Begin gavage feedings to supplement diet. d. Serve foods that are either hot or cold.

a. Relax any eating pressures. ANS: A A multifaceted approach is necessary for children with severe stomatitis and anorexia. First, the parents should relax eating pressures. The nurse should suggest that the parents try soft, bland foods; normal saline or bicarbonate mouthwashes; and local anesthetics. The stomatitis is a temporary condition. The child can resume good food habits as soon as the condition resolves. DIF: Cognitive Level: Apply REF: p. 826 TOP: Integrated Process: Teaching/Learning MSC: Area of Client Needs: Physiologic Integrity: Physiologic Adaptation

35. A 10-year-old child, without a history of previous seizures, experiences a tonic-clonic seizure at school. Breathing is not impaired, but some postictal confusion occurs. What is the most appropriate initial action by the school nurse? a. Stay with child and have someone call emergency medical service (EMS) b. Notify parent and regular practitioner c. Notify parent that child should go home d. Stay with child, offering calm reassurance

a. Stay with child and have someone call emergency medical service (EMS) ANS: A The EMS should be called to transport the child because this is the child's first seizure. Because this is the first seizure, evaluation should be performed as soon as possible. The nurse should stay with the child while someone else notifies the EMS. DIF: Cognitive Level: Apply REF: p. 897 TOP: Integrated Process: Nursing Process: Implementation MSC: Area of Client Needs: Physiologic Integrity: Physiologic Adaptation

11. The nurse is administering an IV chemotherapeutic agent to a child with leukemia. The child suddenly begins to wheeze and have severe urticaria. Which is the most appropriate nursing action? a. Stop drug infusion immediately. b. Recheck rate of drug infusion. c. Observe child closely for next 10 minutes. d. Explain to child that this is an expected side effect.

a. Stop drug infusion immediately.

27. The nurse is conducting a staff in-service on inherited childhood blood disorders. Which statement describes severe combined immunodeficiency syndrome (SCIDS)? a. There is a deficit in both the humoral and cellular immunity with this disease. b. Production of red blood cells is affected with this disease. c. Adult hemoglobin is replaced by abnormal hemoglobin in this disease. d. There is a deficiency of T and B lymphocyte production with this disease.

a. There is a deficit in both the humoral and cellular immunity with this disease. ANS: A Severe combined immunodeficiency syndrome (SCIDS) is a genetic disorder that results in deficits of both humoral and cellular immunity. Wiskott-Aldrich is an X-linked recessive disorder with selected deficiencies of T and B lymphocytes. Fanconi syndrome is a hereditary disorder of red cell production. Sickle cell disease is characterized by the replacement of adult hemoglobin with an abnormal hemoglobin S. DIF: Cognitive Level: Understand REF: p. 809 TOP: Integrated Process: Teaching/Learning MSC: Area of Client Needs: Physiologic Integrity: Physiologic Adaptation

19. A toddler fell out of a second-story window. She had a brief loss of consciousness and vomited four times. Since admission, she has been alert and oriented. Her mother asks why a computed tomography (CT) scan is required when she "seems fine." Which explanation should the nurse give? a. Your child may have a brain injury and the CT can rule one out. b. The CT needs to be done because of your child's age. c. Your child may start to have seizures and a baseline CT should be done. d. Your child probably has a skull fracture and the CT can confirm this diagnosis.

a. Your child may have a brain injury and the CT can rule one out. ANS: A The child's history of the fall, brief loss of consciousness, and vomiting four times necessitates evaluation of a potential brain injury. The severity of a head injury may not be apparent on clinical examination but will be detectable on a CT scan. The need for the CT scan is related to the injury and symptoms, not the child's age. The CT scan is necessary to determine whether a brain injury has occurred. DIF: Cognitive Level: Apply REF: p. 876 TOP: Integrated Process: Teaching/Learning MSC: Area of Client Needs: Physiologic Integrity: Physiologic Adaptation

52. One of the supervisors for a home health agency asks the nurse to give the family a survey evaluating the nurses and other service providers. How should the nurse interpret this request? a. Inappropriate, unless nurses are able to evaluate family. b. Appropriate to improve quality of care. c. Inappropriate, unless nurses and other providers agree to participate. d. Inappropriate, because family lacks knowledge necessary to evaluate professionals.

b. Appropriate to improve quality of care. ANS: B Quality assessment and improvement activities are essential for virtually all organizations. Family involvement is essential in evaluating a home care plan and can occur on several levels. The nurse can ask the family open-ended questions at regular intervals to assess their opinion of the effectiveness of care. Families should also be given an opportunity to evaluate the individual home care nurses, the home care agency, and other service providers periodically. The nurse is the care provider. The evaluation is of the provision of care to the patient and family. The nurse's role is not to evaluate the family. Quality-monitoring activities are required by virtually all health care agencies. During the evaluation process, the family is requested to provide their perceptions of care. DIF: Cognitive Level: Apply REF: p. 708 TOP: Integrated Process: Communication and Documentation MSC: Area of Client Needs: Safe and Effective Care Environment: Management of Care

36. The nurse is caring for an infant whose cleft lip was repaired. What important aspects of this infant's postoperative care should be included? a. Arm restraints, postural drainage, mouth irrigations b. Cleansing the suture line, supine and side-lying positions, arm restraints c. Mouth irrigations, prone position, cleansing the suture line d. Supine and side-lying positions, postural drainage, arm restraints

b. Cleansing the suture line, supine and side-lying positions, arm restraints ANS: B The suture line should be cleansed gently after feeding. The child should be positioned on the back, on the side, or in an infant seat. Elbows are restrained to prevent the child from accessing the operative site. Postural drainage is not indicated. This would increase the pressure on the operative site when the child is placed in different positions. There is no reason to perform mouth irrigations, and the child should not be placed in the prone position where injury to the suture site can occur. DIF: Cognitive Level: Apply REF: p. 725 TOP: Integrated Process: Nursing Process: Planning MSC: Area of Client Needs: Physiologic Integrity: Reduction of Risk Potential

27. One of the clinical manifestations of chronic renal failure is uremic frost. Which best describes this term? a. Deposits of urea crystals in urine b. Deposits of urea crystals on skin c. Overexcretion of blood urea nitrogen d. Inability of body to tolerate cold temperatures

b. Deposits of urea crystals on skin ANS: B Uremic frost is the deposition of urea crystals on the skin. The urea crystals are present on the skin, not in the urine. The kidneys are unable to excrete blood urea nitrogen, leading to elevated levels. There is no relation between cold temperatures and uremic frost. PTS: 1 DIF: Cognitive Level: Understand REF: 922 TOP: Integrated Process: Nursing Process: Assessment MSC: Area of Client Needs: Physiologic Integrity: Physiologic Adaptation

35. A mother who intended to breastfeed has given birth to an infant with a cleft palate. What nursing interventions should be included? a. Giving medication to suppress lactation. b. Encouraging and helping mother to breastfeed. c. Teaching mother to feed breast milk by gavage. d. Recommending use of a breast pump to maintain lactation until infant can suck.

b. Encouraging and helping mother to breastfeed. ANS: B The mother who wishes to breastfeed may need encouragement and support because the defect does present some logistical issues. The nipple must be positioned and stabilized well back in the infant's oral cavity so that the tongue action facilitates milk expression. Because breastfeeding is an option, if the mother wishes to breastfeed, medications should not be given to suppress lactation. Because breastfeeding can usually be accomplished, gavage feedings are not indicated. The suction required to stimulate milk, absent initially, may be useful before nursing to stimulate the let-down reflex. DIF: Cognitive Level: Apply REF: p. 724 TOP: Integrated Process: Nursing Process: Implementation MSC: Area of Client Needs: Physiologic Integrity: Reduction of Risk Potential

33. A newborn was admitted to the nursery with a complete bilateral cleft lip and palate. The physician explained the plan of therapy and its expected good results. However, the mother refuses to see or hold her baby. What is the initial therapeutic approach for the mother? a. Restating what the physician has told her about plastic surgery. b. Encouraging her to express her feelings. c. Emphasizing the normalcy of her baby and the baby's need for mothering. d. Recognizing that negative feelings toward the child continue throughout childhood.

b. Encouraging her to express her feelings. ANS: B For parents, cleft lip and cleft palate deformities are particularly disturbing. The nurse must emphasize not only the infant's physical needs but also the parents' emotional needs. The mother needs to be able to express her feelings before she can accept her child. Although the nurse will restate what the physician has told the mother about plastic surgery, it is not part of the initial therapeutic approach. As the mother expresses her feelings, the nurse's actions should convey to the parents that the infant is a precious human being. The nurse emphasizes the child's normalcy and helps the mother recognize the child's uniqueness. DIF: Cognitive Level: Apply REF: p. 723 TOP: Integrated Process: Nursing Process: Implementation MSC: Area of Client Needs: Psychosocial Integrity

30. Which vaccine is now recommended for the immunization of all newborns? a. Hepatitis A vaccine b. Hepatitis B vaccine c. Hepatitis C vaccine d. Hepatitis A, B, and C vaccines

b. Hepatitis B vaccine ANS: B Universal vaccination for hepatitis B is now recommended for all newborns. A vaccine is available for hepatitis A, but it is not yet universally recommended. No vaccine is currently available for hepatitis C. Only hepatitis B vaccine is recommended for newborns. DIF: Cognitive Level: Understand REF: p. 717 TOP: Integrated Process: Nursing Process: Evaluation MSC: Area of Client Needs: Health Promotion and Maintenance

40. What is the best description of pyloric stenosis? a. Dilation of the pylorus b. Hypertrophy of the pyloric muscle c. Hypotonicity of the pyloric muscle d. Reduction of tone in the pyloric muscle

b. Hypertrophy of the pyloric muscle ANS: B Hypertrophic pyloric stenosis occurs when the circumferential muscle of the pyloric sphincter becomes thickened, resulting in elongation and narrowing of the pyloric channel. Dilation of the pylorus, hypotonicity of the pyloric muscle, and reduction of tone in the pyloric muscle are not the definition of pyloric stenosis. DIF: Cognitive Level: Understand REF: p. 728 TOP: Integrated Process: Nursing Process: Assessment MSC: Area of Client Needs: Physiologic Integrity: Reduction of Risk Potential

32. Which statement is descriptive of renal transplantation in children? a. It is an acceptable means of treatment after age 10 years. b. It is the preferred means of renal replacement therapy in children. c. Children can receive kidneys only from other children. d. The decision for transplantation is difficult because a relatively normal lifestyle is not possible.

b. It is the preferred means of renal replacement therapy in children. ANS: B Renal transplant offers the opportunity for a relatively normal life and is the preferred means of renal replacement therapy in end-stage renal disease. Renal transplantation can be done in children as young as age 6 months. Both children and adults can serve as donors for renal transplant purposes. Renal transplantation affords the child a more normal lifestyle than dependence on dialysis. PTS: 1 DIF: Cognitive Level: Understand REF: 925 TOP: Integrated Process: Nursing Process: Assessment MSC: Area of Client Needs: Physiologic Integrity: Physiologic Adaptation

53. The home care nurse has been visiting an adolescent with recently acquired tetraplegia. The teen's mother tells the nurse, "I'm sick of providing all the care while my husband does whatever he wants to, whenever he wants to do it." Which should be the initial action of the nurse? a. Refer mother for counseling. b. Listen and reflect mother's feelings. c. Ask father, in private, why he does not help. d. Suggest ways the mother can get her husband to help.

b. Listen and reflect mother's feelings. ANS: B It is appropriate for the nurse to reflect with the mother about her feelings, exploring issues such as an additional home health aide to help care for the child and provide respite for the mother. It is inappropriate for the nurse to agree with the mother that her husband is not helping enough. It is a judgment beyond the role of the nurse and can undermine the family relationship. Counseling is not necessary at this time. A support group for caregivers may be indicated. Asking the father why he does not help and suggesting ways to the mother to get her husband to help are interventions based on the mother's assumption of minimal contribution to the child's care. The father may have a full-time job and other commitments. The parents need to have an involved third person help them through the negotiation of responsibilities for the loss of their normal child and new parenting responsibilities. DIF: Cognitive Level: Apply REF: p. 708 TOP: Integrated Process: Communication and Documentation MSC: Area of Client Needs: Psychosocial Integrity

17. Enemas are ordered to empty the bowel preoperatively for a child with Hirschsprung disease. What enema solution should be used? a. Tap water b. Normal saline c. Oil retention d. Phosphate preparation

b. Normal saline ANS: B Isotonic solutions should be used in children. Saline is the solution of choice. Plain water is not used. This is a hypotonic solution and can cause rapid fluid shift, resulting in fluid overload. Oil-retention enemas will not achieve the "until clear" result. Phosphate enemas are not advised for children because of the harsh action of the ingredients. The osmotic effects of the phosphate enema can result in diarrhea, which can lead to metabolic acidosis. DIF: Cognitive Level: Apply REF: p. 703 TOP: Integrated Process: Nursing Process: Implementation MSC: Area of Client Needs: Physiologic Integrity: Reduction of Risk Potential

14. Which best describes acute glomerulonephritis? a. Occurs after a urinary tract infection b. Occurs after a streptococcal infection c. Associated with renal vascular disorders d. Associated with structural anomalies of genitourinary tract

b. Occurs after a streptococcal infection ANS: B Acute glomerulonephritis is an immune-complex disease that occurs after a streptococcal infection with certain strains of the group A â-hemolytic streptococcus. Acute glomerulonephritis usually follows streptococcal pharyngitis and is not associated with renal vascular disorders or genitourinary tract structural anomalies. PTS: 1 DIF: Cognitive Level: Understand REF: 860 TOP: Integrated Process: Nursing Process: Assessment MSC: Area of Client Needs: Physiologic Integrity: Physiologic Adaptation

7. 7. The nurse is conducting a staff in-service on newborn defects of the genitourinary system. Which describes the narrowing of the preputial opening of the foreskin? a. Chordee b. Phimosis c. Epispadias d. Hypospadias

b. Phimosis ANS: B Phimosis is the narrowing or stenosis of the preputial opening of the foreskin. Chordee is the ventral curvature of the penis. Epispadias is the meatal opening on the dorsal surface of the penis. Hypospadias is a congenital condition in which the urethral opening is located anywhere along the ventral surface of the penis. PTS: 1 DIF: Cognitive Level: Remember REF: 912 TOP: Integrated Process: Teaching/Learning MSC: Area of Client Needs: Physiologic Integrity: Physiologic Adaptation

8. Which is an objective of care for a 10-year-old child with minimal change nephrotic syndrome? a. Reduce blood pressure. b. Reduce excretion of urinary protein. c. Increase excretion of urinary protein. d. Increase ability of tissues to retain fluid.

b. Reduce excretion of urinary protein. ANS: B The objectives of therapy for the child with minimal change nephrotic syndrome include reduction of the excretion of urinary protein, reduction of fluid retention, prevention of infection, and minimization of complications associated with therapy. Blood pressure is usually not elevated in minimal change nephrotic syndrome. Excretion of urinary protein and fluid retention are part of the disease process and must be reversed. PTS: 1 DIF: Cognitive Level: Apply REF: 858 TOP: Integrated Process: Nursing Process: Planning MSC: Area of Client Needs: Physiologic Integrity: Physiologic Adaptation

13. Which is included in the diet of a child with minimal change nephrotic syndrome? a. High protein b. Salt restriction c. Low fat d. High carbohydrate

b. Salt restriction ANS: B Salt is usually restricted (but not eliminated) during the edema phase. The child has little appetite during the acute phase. Favorite foods are provided (with the exception of high-salt ones) in an attempt to provide nutritionally complete meals. PTS: 1 DIF: Cognitive Level: Understand REF: 914 TOP: Integrated Process: Nursing Process: Implementation MSC: Area of Client Needs: Physiologic Integrity: Basic Care and Comfort

4. The nurse is teaching parents about prevention of urinary tract infections in children. Which factor predisposes the urinary tract to infection? a. Increased fluid intake b. Short urethra in young girls c. Prostatic secretions in males d. Frequent emptying of the bladder

b. Short urethra in young girls ANS: B The short urethra in females provides a ready pathway for invasion of organisms. Increased fluid intake and frequent emptying of the bladder offer protective measures against urinary tract infections. Prostatic secretions have antibacterial properties that inhibit bacteria. PTS: 1 DIF: Cognitive Level: Understand REF: 908 TOP: Integrated Process: Teaching/Learning MSC: Area of Client Needs: Physiologic Integrity: Reduction of Risk Potential

36. A school-age child with chronic renal failure is admitted to the hospital with a serum potassium level of 5.2 mEq/L. Which prescribed medication should the nurse plan to administer? a. Spironolactone (Aldactone) b. Sodium polystyrene sulfonate (Kayexalate) c. Lactulose (Cephulac) d. Calcium carbonate (Calcitab)

b. Sodium polystyrene sulfonate (Kayexalate) ANS: B Normal serum potassium levels in a school-age child are 3.5 to 5 mEq/L. Sodium polystyrene sulfonate is administered to reduce serum potassium levels. Spironolactone is a potassium sparing diuretic and should not be used if the serum potassium is elevated. Lactulose is administered to reduce ammonia levels in patients with liver disease. Calcium carbonate may be prescribed as a calcium supplement, but it will not reduce serum potassium levels. PTS: 1 DIF: Cognitive Level: Apply REF: 920 TOP: Integrated Process: Nursing Process: Implementation MSC: Area of Client Needs: Physiologic Integrity: Pharmacologic and Parenteral Therapy

20. A 4-month-old infant has gastroesophageal reflux (GER) but is thriving without other complications. Which should the nurse suggest to minimize reflux? a. Place in Trendelenburg position after eating. b. Thicken formula with rice cereal. c. Give continuous nasogastric tube feedings. d. Give larger, less frequent feedings.

b. Thicken formula with rice cereal. ANS: B Small, frequent feedings of formula combined with 1 teaspoon to 1 tablespoon of rice cereal per ounce of formula have been recommended. Milk-thickening agents have been shown to decrease the number of episodes of vomiting and to increase the caloric density of the formula. This may benefit infants who are underweight as a result of GER disease. Placing the child in a Trendelenburg position would increase the reflux. Continuous nasogastric feedings are reserved for infants with severe reflux and failure to thrive. DIF: Cognitive Level: Apply REF: p. 726 TOP: Integrated Process: Teaching/Learning MSC: Area of Client Needs: Physiologic Integrity: Reduction of Risk Potential

6. The nurse is preparing an adolescent for discharge after a cardiac catheterization. Which statement by the adolescent would indicate a need for further teaching? a. "I should avoid tub baths but may shower." b. "I have to stay on strict bed rest for 3 days." c. "I should remove the pressure dressing the day after the procedure." d. "I may attend school but should avoid exercise for several days."

b. "I have to stay on strict bed rest for 3 days." ANS: B The child does not need to be on strict bed rest for 3 days. Showers are recommended; children should avoid a tub bath. The pressure dressing is removed the day after the catheterization and replaced by an adhesive bandage to keep the area clean. Strenuous activity must be avoided for several days, but the child can return to school. DIF: Cognitive Level: Analyze REF: p. 740 TOP: Integrated Process: Teaching/Learning MSC: Area of Client Needs: Physiologic Integrity: Reduction of Risk Potential

7. The nurse is preparing a school-age child for computed tomography (CT scan) to assess cerebral function. The nurse should include which statement in preparing the child? a. "Pain medication will be given." b. "The scan will not hurt." c. "You will be able to move once the equipment is in place." d. "Unfortunately, no one can remain in the room with you during the test."

b. "The scan will not hurt." ANS: B For CT scans, the child must be immobilized. It is important to emphasize to the child that at no time is the procedure painful. Pain medication is not required; however, sedation is sometimes necessary. Someone is able to remain with the child during the procedure. DIF: Cognitive Level: Apply REF: p. 876 TOP: Integrated Process: Teaching/Learning MSC: Area of Client Needs: Health Promotion and Maintenance

7. How much folic acid is recommended for women of childbearing age? a. 1.0 mg b. 0.4 mg c. 1.5 mg d. 2.0 mg

b. 0.4 mg ANS: B It has been estimated that a daily intake of 0.4 mg of folic acid in women of childbearing age will prevent 50% to 70% of cases of neural tube defects; 1.0 mg is too low a dose; 1.5 to 2.0 mg are not the recommended dosages of folic acid. DIF: Cognitive Level: Remember REF: p. 988 TOP: Integrated Process: Nursing Process: Evaluation MSC: Area of Client Needs: Physiologic Integrity: Pharmacologic and Parenteral Therapies

15. An 8-year-old child is receiving digoxin (Lanoxin). The nurse should notify the practitioner and withhold the medication if the apical pulse is less than _____ beats/min. a. 60 b. 70 c. 90 d. 100

b. 70 ANS: B If a 1-minute apical pulse is less than 70 beats/min for an older child, the digoxin is withheld; 60 beats/min is the cut-off for holding the digoxin dose in an adult. A pulse below 90 to 110 beats/min is the determination for not giving a digoxin dose to infants and young children. DIF: Cognitive Level: Apply REF: p. 752 TOP: Integrated Process: Nursing Process: Assessment MSC: Area of Client Needs: Physiologic Integrity: Pharmacologic and Parenteral Therapies

2. The nurse has received report on four children. Which child should the nurse assess first? a. A school-age child in a coma with stable vital signs b. A preschool child with a head injury and decreasing level of consciousness c. An adolescent admitted after a motor vehicle accident is oriented to person and place d. A toddler in a persistent vegetative state with a low-grade fever

b. A preschool child with a head injury and decreasing level of consciousness ANS: B The nurse should assess the child with a head injury and decreasing level of consciousness first (LOC). Assessment of LOC remains the earliest indicator of improvement or deterioration in neurologic status. The next child the nurse should assess is a toddler in a persistent vegetative state with a low-grade fever. The school-age child in a coma with stable vital signs and the adolescent admitted to the hospital who is oriented to his surroundings would be of least worry to the nurse. DIF: Cognitive Level: Apply REF: p. 873 TOP: Integrated Process: Nursing Process: Implementation MSC: Area of Client Needs: Safe and Effective Care Environment: Management of Care

16. What is chronic adrenocortical insufficiency also called? a. Graves disease b. Addison disease c. Cushing syndrome d. Hashimoto disease

b. Addison disease ANS: B Addison disease is chronic adrenocortical insufficiency. Graves and Hashimoto diseases involve the thyroid gland. Cushing syndrome is a result of excessive circulation of free cortisol. DIF: Cognitive Level: Remember REF: p. 924 TOP: Integrated Process: Nursing Process: Problem Identification MSC: Area of Client Needs: Physiologic Integrity: Physiologic Adaptation`

14. A school-age child is admitted in vasoocclusive sickle cell crisis. What should be included in the child's care? a. Correction of acidosis b. Adequate hydration and pain management c. Pain management and administration of heparin d. Adequate oxygenation and replacement of factor VIII

b. Adequate hydration and pain management ANS: B The management of crises includes adequate hydration, minimization of energy expenditures, pain management, electrolyte replacement, and blood component therapy if indicated. Hydration and pain control are two of the major goals of therapy. The acidosis will be corrected as the crisis is treated. Heparin and factor VIII are not indicated in the treatment of vasoocclusive sickle cell crisis. Oxygen may prevent further sickling, but it is not effective in reversing sickling because it cannot reach the clogged blood vessels. DIF: Cognitive Level: Apply REF: p. 796 TOP: Integrated Process: Nursing Process: Implementation MSC: Area of Client Needs: Physiologic Integrity: Physiologic Adaptation

17. The nurse is admitting a school-age child with suspected Guillain-Barré syndrome (GBS). Which is a priority in the care for this child? a. Monitoring intake and output b. Assessing respiratory efforts c. Placing on a telemetry monitor d. Obtaining laboratory studies

b. Assessing respiratory efforts ANS: B Treatment of GBS is primarily supportive. In the acute phase, patients are hospitalized because respiratory and pharyngeal involvement may require assisted ventilation, sometimes with a temporary tracheotomy. Treatment modalities include aggressive ventilatory support in the event of respiratory compromise, intravenous (IV) administration of immunoglobulin (IVIG), and sometimes steroids; plasmapheresis and immunosuppressive drugs may also be used. Intake and output, telemetry monitoring, and obtaining laboratory studies may be part of the plan of care but are not the priority. DIF: Cognitive Level: Analyze REF: p. 996 TOP: Integrated Process: Nursing Process: Evaluation MSC: Area of Client Needs: Physiologic Integrity: Reduction of Risk Potential

14. Which statement best describes a subdural hematoma? a. Bleeding occurs between the dura and the skull. b. Bleeding occurs between the dura and the cerebrum. c. Bleeding is generally arterial, and brain compression occurs rapidly. d. The hematoma commonly occurs in the parietotemporal region.

b. Bleeding occurs between the dura and the cerebrum. ANS: B A subdural hematoma is bleeding that occurs between the dura and the cerebrum as a result of a rupture of cortical veins that bridge the subdural space. An epidural hemorrhage occurs between the dura and the skull, is usually arterial with rapid brain concussion, and occurs most often in the parietotemporal region. DIF: Cognitive Level: Understand REF: p. 883 TOP: Integrated Process: Nursing Process: Assessment MSC: Area of Client Needs: Physiologic Integrity: Physiologic Adaptation

39. The nurse is teaching nursing students about shock that occurs in children. What is one of the most frequent causes of hypovolemic shock in children? a. Sepsis b. Blood loss c. Anaphylaxis d. Congenital heart disease

b. Blood loss ANS: B Blood loss is the most frequent cause of hypovolemic shock in children. Sepsis causes septic shock, which is overwhelming sepsis and circulating bacterial toxins. Anaphylactic shock results from extreme allergy or hypersensitivity to a foreign substance. Congenital heart disease contributes to hypervolemia, not hypovolemia. DIF: Cognitive Level: Understand REF: p. 778 TOP: Integrated Process: Teaching/Learning MSC: Area of Client Needs: Physiologic Integrity: Physiologic Adaptation

4. A boy with leukemia screams whenever he needs to be turned or moved. Which is the most probable cause of this pain? a. Edema b. Bone involvement c. Petechial hemorrhages d. Changes within the muscles

b. Bone involvement ANS: B The invasion of the bone marrow with leukemic cells gradually causes a weakening of the bone and a tendency toward fractures. As leukemic cells invade the periosteum, increasing pressure causes severe pain. Edema, petechial hemorrhages, and changes within the muscles would not cause severe pain. DIF: Cognitive Level: Analyze REF: p. 826 TOP: Integrated Process: Nursing Process: Planning MSC: Area of Client Needs: Physiologic Integrity: Physiologic Adaptation

13. The nurse is teaching nursing students about childhood fractures. Which describes a compound skull fracture? a. Involves the basilar portion of the occipital bone b. Bone is exposed through the skin c. Traumatic separations of the cranial sutures d. Bone is pushed inward, causing pressure on the brain

b. Bone is exposed through the skin ANS: B A compound fracture has the bone exposed through the skin. A basilar fracture involves the basilar portion of the frontal, ethmoid, sphenoid, temporal, or occipital bone. Diastatic skull fractures are traumatic separations of the cranial sutures. A depressed fracture has the bone pushed inward, causing pressure on the brain. DIF: Cognitive Level: Understand REF: p. 883 TOP: Integrated Process: Teaching/Learning MSC: Area of Client Needs: Physiologic Integrity: Physiologic Adaptation

31. What is the primary nursing intervention to prevent bacterial endocarditis? a. Institute measures to prevent dental procedures. b. Counsel parents of high-risk children about prophylactic antibiotics. c. Observe children for complications, such as embolism and heart failure. d. Encourage restricted mobility in susceptible children.

b. Counsel parents of high-risk children about prophylactic antibiotics. ANS: B The objective of nursing care is to counsel the parents of high-risk children about both the need for prophylactic antibiotics for dental procedures and the necessity of maintaining excellent oral health. The child's dentist should be aware of the child's cardiac condition. Dental procedures should be done to maintain a high level of oral health. Prophylactic antibiotics are necessary. Children should be observed for complications such as embolism and heart failure and restricted mobility should be encouraged in susceptible children, but maintaining good oral health and prophylactic antibiotics is important. DIF: Cognitive Level: Apply REF: p. 765 TOP: Integrated Process: Nursing Process: Assessment MSC: Area of Client Needs: Physiologic Integrity: Reduction of Risk Potential

28. The nurse is caring for an 11-year-old boy who has recently been diagnosed with diabetes. Which should be included in the teaching plan for daily injections? a. The parents do not need to learn the procedure. b. He is old enough to give most of his own injections. c. Self-injections will be possible when he is closer to adolescence. d. He can learn about self-injections when he is able to reach all injection sites.

b. He is old enough to give most of his own injections. ANS: B School-age children are able to give their own injections. Parents should participate in learning and giving the insulin injections. He is already old enough to administer his own insulin. The child is able to use thighs, abdomen, part of the hip, and arm. Assistance can be obtained if other sites are used. DIF: Cognitive Level: Apply REF: p. 931 TOP: Integrated Process: Teaching/Learning MSC: Area of Client Needs: Physiologic Integrity: Pharmacologic and Parenteral Therapies

16. The nurse is caring for an infant with myelomeningocele scheduled for surgical closure in the morning. Which intervention should the nurse plan for the care of the myelomeningocele sac? a. Open to air b. Covered with a sterile, moist, nonadherent dressing c. Reinforcement of the original dressing if drainage noted d. A diaper secured over the dressing

b. Covered with a sterile, moist, nonadherent dressing ANS: B Before surgical closure, the myelomeningocele is prevented from drying by the application of a sterile, moist, nonadherent dressing over the defect. The moistening solution is usually sterile normal saline. Dressings are changed frequently (every 2 to 4 hours), and the sac is closely inspected for leaks, abrasions, irritation, and any signs of infection. The sac must be carefully cleansed if it becomes soiled or contaminated. The original dressing would not be reinforced but changed as needed. A diaper is not placed over the dressing because stool contamination can occur. DIF: Cognitive Level: Apply REF: p. 987 TOP: Integrated Process: Nursing Process: Implementation MSC: Area of Client Needs: Physiologic Integrity: Reduction of Risk Potential

33. A nurse is reviewing the laboratory results on a school-age child with hypoparathyroidism. Which results are consistent with this condition? a. Decreased serum phosphorus b. Decreased serum calcium c. Increased serum glucose d. Decreased serum cortisol level

b. Decreased serum calcium ANS: B The diagnosis of hypoparathyroidism is made on the basis of clinical manifestations associated with decreased serum calcium and increased serum phosphorus. A decreased serum phosphorus level would be seen in hyperparathyroidism, elevated glucose in diabetes, and a decreased serum cortisol level in adrenocortical insufficiency (Addison disease). DIF: Cognitive Level: Analyze REF: p. 921 TOP: Integrated Process: Nursing Process: Evaluation MSC: Area of Client Needs: Physiologic Integrity: Physiologic Adaptation

23. A young child with human immunodeficiency virus (HIV) is receiving several antiretroviral drugs. What is the purpose of these drugs? a. Cure the disease b. Delay disease progression c. Prevent spread of disease d. Treat Pneumocystis carinii pneumonia

b. Delay disease progression ANS: B Although not a cure, these antiviral drugs can suppress viral replication, preventing further deterioration of the immune system and delaying disease progression. At this time, cure is not possible. These drugs do not prevent the spread of the disease. P. carinii prophylaxis is accomplished with antibiotics. DIF: Cognitive Level: Understand REF: p. 806 TOP: Integrated Process: Nursing Process: Implementation MSC: Area of Client Needs: Physiologic Inte

8. A nasal spray of desmopressin acetate (DDAVP) is used to treat which disorder? a. Hypopituitarism b. Diabetes insipidus c. Acute adrenocortical insufficiency d. Syndrome of inappropriate antidiuretic hormone

b. Diabetes insipidus ANS: B The drug of choice for the treatment of diabetes insipidus is DDAVP, which is a synthetic analogue of vasopressin. DDAVP is not used to treat hypopituitarism, acute adrenocortical insufficiency, or syndrome of inappropriate antidiuretic hormone. DIF: Cognitive Level: Understand REF: p. 917 TOP: Integrated Process: Nursing Process: Planning MSC: Area of Client Needs: Physiologic Integrity: Pharmacologic and Parenteral Therapies

21. A school-age child has sustained a head injury and multiple fractures after being thrown from a horse. The child's level of consciousness is variable. The parents tell the nurse that they think their child is in pain because of periodic crying and restlessness. What is the most appropriate nursing action? a. Discuss with parents the child's previous experiences with pain b. Discuss with practitioner what analgesia can be safely administered c. Explain that analgesia is contraindicated with a head injury d. Explain that analgesia is unnecessary when child is not fully awake and alert

b. Discuss with practitioner what analgesia can be safely administered ANS: B A key nursing role is to provide sedation and analgesia for the child. Consultation with the appropriate practitioner is necessary to avoid conflict between the necessity to monitor the child's neurologic status and the promotion of comfort and relief of anxiety. Information on the child's previous experiences with pain should be obtained as part of the assessment, but because of the severity of injury, analgesia should be provided as soon as possible. Analgesia can be safely used in individuals who have sustained head injuries and can decrease anxiety and resultant increased ICP. DIF: Cognitive Level: Apply REF: p. 905 TOP: Integrated Process: Teaching/Learning MSC: Area of Client Needs: Physiologic Integrity: Pharmacologic and Parenteral Therapies

18. The parents of a neonate with adrenogenital hyperplasia tell the nurse that they are afraid to have any more children. The nurse should explain which statement about adrenogenital hyperplasia? a. It is not hereditary. b. Genetic counseling is indicated. c. It can be prevented during pregnancy. d. All future children will have the disorder.

b. Genetic counseling is indicated. ANS: B Some forms of adrenogenital hyperplasia are hereditary and should be referred for genetic counseling. Affected offspring should also be referred for genetic counseling. There is an autosomal recessive form of adrenogenital hyperplasia. A prenatal treatment with glucocorticoids can be offered to the mother during pregnancy to avoid the sex ambiguity, but it does not affect the presence of the disease. If it is the heritable form, for each pregnancy, a 25% risk occurs that the child will be affected. DIF: Cognitive Level: Apply REF: p. 923 TOP: Integrated Process: Teaching/Learning MSC: Area of Client Needs: Physiologic Integrity: Physiologic Adaptation

2. Which is the usual presenting symptom for testicular cancer? a. Hard, painful mass b. Hard, painless mass c. Epididymis easily palpated d. Scrotal swelling and pain

b. Hard, painless mass ANS: B The usual presenting symptom for testicular cancer is a heavy, hard, painless mass that is either smooth or nodular and palpated on the testes. A hard, painful mass, an epididymis easily palpated, and scrotal swelling and pain are not the clinical presentations of testicular cancer. DIF: Cognitive Level: Understand REF: p. 842 TOP: Integrated Process: Nursing Process: Assessment MSC: Area of Client Needs: Physiologic Integrity: Physiologic Adaptation

4. Which should the nurse expect to find in the cerebral spinal fluid (CSF) results of a child with Guillain-Barré syndrome (GBS)? (Select all that apply.) a. Decreased protein concentration b. Normal glucose c. Fewer than 10 white blood cells (WBCs/mm3 ) d. Elevated red blood cell (RBC) count

b. Normal glucose c. Fewer than 10 white blood cells (WBCs/mm3 ) ANS: B, C Diagnosis of GBS is based on clinical manifestations, CSF analysis, and EMG findings. CSF analysis reveals an abnormally elevated protein concentration, normal glucose, and fewer than 10 WBCs/mm3 . CSF fluid should not contain RBCs. DIF: Cognitive Level: Understand REF: p. 996 TOP: Integrated Process: Nursing Process: Evaluation MSC: Area of Client Needs: Physiologic Integrity: Physiologic Adaptation

2. Which is true concerning hepatitis B? (Select all that apply.) a. Hepatitis B cannot exist in carrier state. b. Hepatitis B can be prevented by HBV vaccine. c. Hepatitis B can be transferred to an infant of a breastfeeding mother. d. Onset of hepatitis B is insidious. e. Principal mode of transmission for hepatitis B is fecal-oral route. f. Immunity to hepatitis B occurs after one attack.

b. Hepatitis B can be prevented by HBV vaccine. c. Hepatitis B can be transferred to an infant of a breastfeeding mother. d. Onset of hepatitis B is insidious. f. Immunity to hepatitis B occurs after one attack. ANS: B, C, D, F The vaccine elicits the formation of an antibody to the hepatitis B surface antigen, which is protective against hepatitis B. Hepatitis B can be transferred to an infant of a breastfeeding mother, especially if the mother's nipples are cracked. The onset of hepatitis B is insidious. Immunity develops after one exposure to hepatitis B. Hepatitis B has a carrier state. The fecal-oral route is the principal mode of transmission for hepatitis A. Hepatitis B is transmitted through the parenteral route. DIF: Cognitive Level: Understand REF: p. 717 TOP: Integrated Process: Nursing Process: Diagnosis MSC: Area of Client Needs: Physiologic Integrity: Reduction of Risk Potential

11. What condition may cause exophthalmos (protruding eyeballs) in children? a. Hypothyroidism b. Hyperthyroidism c. Hypoparathyroidism d. Hyperparathyroidism

b. Hyperthyroidism ANS: B Exophthalmos is a clinical manifestation of hyperthyroidism. Hypothyroidism, hypoparathyroidism, and hyperparathyroidism are not associated with exophthalmos. DIF: Cognitive Level: Understand REF: p. 920 TOP: Integrated Process: Nursing Process: Assessment MSC: Area of Client Needs: Physiologic Integrity: Physiologic Adaptation

42. Which occurs in septic shock? a. Hypothermia b. Increased cardiac output c. Vasoconstriction d. Angioneurotic edema

b. Increased cardiac output ANS: B Increased cardiac output, which results in warm, flushed skin, is one of the manifestations of septic shock. Fever and chills are characteristic of septic shock. Vasodilation is more common than vasoconstriction. Angioneurotic edema occurs as a manifestation in anaphylactic shock. DIF: Cognitive Level: Understand REF: p. 779 TOP: Integrated Process: Nursing Process: Assessment MSC: Area of Client Needs: Physiologic Integrity: Physiologic Adaptation

24. Which should the nurse consider when preparing a school-age child and the family for heart surgery? a. Unfamiliar equipment should not be shown. b. Let the child hear the sounds of an ECG monitor. c. Avoid mentioning postoperative discomfort and interventions. d. Explain that an endotracheal tube will not be needed if the surgery goes well.

b. Let the child hear the sounds of an ECG monitor. ANS: B The child and family should be exposed to the sights and sounds of the intensive care unit (ICU). All positive, nonfrightening aspects of the environment are emphasized. The child should be shown unfamiliar equipment and its use demonstrated on a doll. Carefully prepare the child for the postoperative experience, including intravenous (IV) lines, incision, and endotracheal tube. DIF: Cognitive Level: Analyze REF: p. 763 TOP: Integrated Process: Teaching/Learning MSC: Area of Client Needs: Health Promotion and Maintenance

33. The home care nurse has been visiting an adolescent with recently acquired tetraplegia. The teen's mother tells the nurse, "I'm sick of providing all the care while my husband does whatever he wants to, whenever he wants to do it." Which should be the initial action of the nurse? a. Refer mother for counseling. b. Listen and reflect mother's feelings. c. Ask father, in private, why he does not help. d. Suggest ways the mother can get her husband to help. \

b. Listen and reflect mother's feelings. ANS: B It is appropriate for the nurse to reflect with the mother about her feelings, exploring issues such as an additional home health aide to help care for the child and provide respite for the mother. It is inappropriate for the nurse to agree with the mother that her husband is not helping enough. It is a judgment beyond the role of the nurse and can undermine the family relationship. Counseling is not necessary at this time. A support group for caregivers may be indicated. Asking the father why he does not help and suggesting ways to the mother to get her husband to help are interventions based on the mother's assumption of minimal contribution to the child's care. The father may have a full-time job and other commitments. The parents need to have an involved third person help them through the negotiation of responsibilities for the loss of their normal child and new parenting responsibilities. DIF: Cognitive Level: Apply REF: p. 842 TOP: Integrated Process: Communication and Documentation MSC: Area of Client Needs: Psychosocial Integrity

25. What are the vector reservoirs for agents causing viral encephalitis in the United States? a. Tarantula spiders b. Mosquitoes c. Carnivorous wild animals d. Domestic and wild animals

b. Mosquitoes ANS: B Viral encephalitis, not attributable to a childhood viral disease, is usually transmitted by mosquitoes. The vector reservoir for most agents pathogenic for humans and detected in the United States are mosquitoes and ticks; therefore, most cases of encephalitis appear during the hot summer months. Tarantula spiders, carnivorous wild animals, and domestic and wild animals are not reservoirs for the agents that cause viral encephalitis. DIF: Cognitive Level: Understand REF: p. 890 TOP: Integrated Process: Nursing Process: Planning MSC: Area of Client Needs: Physiologic Integrity: Reduction of Risk Potential

4. The nurse is closely monitoring a child who is unconscious after a fall and notices that the child suddenly has a fixed and dilated pupil. How should the nurse interpret these findings? a. Eye trauma b. Neurosurgical emergency c. Severe brainstem damage d. Indication of brain death

b. Neurosurgical emergency ANS: B The sudden appearance of a fixed and dilated pupil(s) is a neurosurgical emergency. The nurse should immediately report this finding. Although a dilated pupil may be associated with eye trauma, this child has experienced a neurologic insult. Pinpoint pupils or bilateral fixed pupils for more than 5 minutes are indicative of brainstem damage. The unilateral fixed and dilated pupil is suggestive of damage on the same side of the brain. One fixed and dilated pupil is not suggestive of brain death. DIF: Cognitive Level: Analyze REF: p. 875 TOP: Integrated Process: Nursing Process: Assessment MSC: Area of Client Needs: Physiologic Integrity: Physiologic Adaptation

22. In which position should the nurse place a 10-year-old child after a large tumor was removed through a supratentorial craniotomy? a. On the inoperative side with the bed flat b. On the inoperative side with the head of bed elevated 20 to 30 degrees c. On the operative side with the bed flat and pillows behind the head d. On the operative side with the head of bed elevated 45 degrees

b. On the inoperative side with the head of bed elevated 20 to 30 degrees ANS: B If a large tumor was removed, the child is not placed on the operative side because the brain may suddenly shift to that cavity, causing trauma to the blood vessels, linings, and the brain itself. The child with an infratentorial procedure is usually positioned on either side with the bed flat. When a supratentorial craniotomy is performed, the head of bed is elevated 20 to 30 degrees with the child on either side or on the back. In a supratentorial craniotomy, the head elevation facilitates CSF drainage and decreases excessive blood flow to the brain to prevent hemorrhage. Pillows should be placed against the child's back, not head, to maintain the desired position. DIF: Cognitive Level: Apply REF: p. 831 TOP: Integrated Process: Nursing Process: Implementation MSC: Area of Client Needs: Physiologic Integrity: Physiologic Adaptation

21. An 8-month-old infant has a hypercyanotic spell while blood is being drawn. What is the priority nursing action? a. Assess for neurologic defects b. Place the child in the knee-chest position c. Begin cardiopulmonary resuscitation d. Prepare family for imminent death

b. Place the child in the knee-chest position ANS: B The first action is to place the infant in the knee-chest position. Blow-by oxygen may be indicated. Neurologic defects are unlikely. The child should be assessed for airway, breathing, and circulation. Often, calming the child and administering oxygen and morphine can alleviate the hypercyanotic spell. DIF: Cognitive Level: Apply REF: p. 759 TOP: Integrated Process: Nursing Process: Implementation MSC: Area of Client Needs: Physiologic Integrity: Physiologic Adaptation

An infant with an unrepaired tetralogy of Fallot defect is becoming extremely cyanotic during a routine blood draw. Which interventions should the nurse implement? Place in order from the highest-priority intervention to the lowest-priority intervention. Provide the answer using lowercase letters separated by commas (e.g., a, b, c, d). a. Administer 100% oxygen by blow-by. b. Place the infant in knee-chest position. c. Remain calm. d. Give morphine subcutaneously or by an existing intravenous line.

b. Place the infant in knee-chest position. a. Administer 100% oxygen by blow-by. d. Give morphine subcutaneously or by an existing intravenous line. c. Remain calm. ANS: b, a, d, c Hypercyanotic spells, also referred to as blue spells or tet spells because they are often seen in infants with tetralogy of Fallot, may occur in any child whose heart defect includes obstruction to pulmonary blood flow and communication between the ventricles. The infant becomes acutely cyanotic and hyperpneic because sudden infundibular spasm decreases pulmonary blood flow and increases right-to- left shunting. Because profound hypoxemia causes cerebral hypoxia, hypercyanotic spells require prompt assessment and treatment to prevent brain damage or possibly death. The infant should first be placed in the knee-chest position to reduce blood returning to the heart. Next 100% oxygen is given to alleviate the hypoxemia. Morphine is next administered to reduce infundibular spasms. Last, the nurse should remain calm. DIF: Cognitive Level: Apply REF: p. 741 TOP: Integrated Process: Nursing Process: Implementation MSC: Area of Client Needs: Physiologic Integrity: Physiologic Adaptation

10. Which is often administered to prevent or control hemorrhage in a child with cancer? a. Nitrosoureas b. Platelets c. Whole blood d. Corticosteroids

b. Platelets ANS: B Most bleeding episodes can be prevented or controlled with the administration of platelet concentrate or platelet-rich plasma. Nitrosoureas, whole blood, and corticosteroids would not prevent or control hemorrhage. DIF: Cognitive Level: Apply REF: p. 826 TOP: Integrated Process: Nursing Process: Planning MSC: Area of Client Needs: Physiologic Integrity: Pharmacologic and Parenteral Therapies

20. As part of the treatment for heart failure, the child takes the diuretic furosemide (Lasix). As part of teaching home care, the nurse encourages the family to give the child foods such as bananas, oranges, and leafy vegetables. These foods are recommended because they are high in which nutrient? a. Chlorides b. Potassium c. Sodium d. Vitamins

b. Potassium ANS: B Diuretics that work on the proximal and distal renal tubules contribute to increased losses of potassium. The child's diet should be supplemented with this electrolyte. With this type of diuretic, potassium must be monitored and supplemented as needed. DIF: Cognitive Level: Understand REF: p. 754 TOP: Integrated Process: Teaching/Learning MSC: Area of Client Needs: Physiologic Integrity: Pharmacologic and Parenteral Therapies

22. The nurse is caring for a child with persistent hypoxia secondary to a cardiac defect. The nurse recognizes that a risk exists of cerebrovascular accidents (strokes). Which is an important objective to decrease this risk? a. Minimize seizures b. Prevent dehydration c. Promote cardiac output d. Reduce energy expenditure

b. Prevent dehydration ANS: B In children with persistent hypoxia, polycythemia develops. Dehydration must be prevented in hypoxemic children because it potentiates the risk of strokes. Minimizing seizures, promoting cardiac output, and reducing energy expenditure will not reduce the risk of cerebrovascular accidents. DIF: Cognitive Level: Analyze REF: p. 759 TOP: Integrated Process: Nursing Process: Implementation MSC: Area of Client Needs: Physiologic Integrity: Physiologic Adaptation

8. The nurse is caring for a neonate born with a myelomeningocele. Surgery to repair the defect is scheduled the next day. Which describes the most appropriate way to position and feed this neonate? a. Prone and tube-fed b. Prone, head turned to side, and nipple-fed c. Supine in an infant carrier and nipple-fed d. Supine, with defect supported with rolled blankets, and nipple-fed

b. Prone, head turned to side, and nipple-fed ANS: B In the prone position, feeding is a problem. The infant's head is turned to one side for feeding. If the child is able to nipple-feed, tube feeding is not needed. Before surgery, the infant is kept in the prone position to minimize tension on the sac and risk of trauma. DIF: Cognitive Level: Apply REF: p. 988 TOP: Integrated Process: Nursing Process: Implementation MSC: Area of Client Needs: Physiologic Integrity: Basic Care and Comfort

15. The parents of a child hospitalized with sickle cell anemia tell the nurse that they are concerned about narcotic analgesics causing addiction. Which is appropriate for the nurse to explain about narcotic analgesics? a. Are often ordered but not usually needed b. Rarely cause addiction because they are medically indicated c. Are given as a last resort because of the threat of addiction d. Are used only if other measures, such as ice packs, are ineffective

b. Rarely cause addiction because they are medically indicated ANS: B The pain of sickle cell anemia is best treated by a multidisciplinary approach. Mild to moderate pain can be controlled by ibuprofen and acetaminophen. When narcotics are indicated, they are titrated to effect and are given around the clock. Patient-controlled analgesia reinforces the patient's role and responsibility in managing the pain and provides flexibility in dealing with pain. Few, if any, patients who receive opioids for severe pain become behaviorally addicted to the drug. Narcotics are often used because of the severe nature of the pain of vasoocclusive crisis. Ice is contraindicated because of its vasoconstrictive effects. DIF: Cognitive Level: Apply REF: p. 796 TOP: Integrated Process: Teaching/Learning MSC: Area of Client Needs: Physiologic Integrity: Pharmacologic and Parenteral Therapies

37. Which is the leading cause of death after heart transplantation? a. Infection b. Rejection c. Cardiomyopathy d. Heart failure

b. Rejection ANS: B The posttransplant course is complex. The leading cause of death after cardiac transplantation is rejection. Infection is a continued risk secondary to the immunosuppression necessary to prevent rejection. Cardiomyopathy is one of the indications for cardiac transplant. Heart failure is not a leading cause of death. DIF: Cognitive Level: Remember REF: p. 775 TOP: Integrated Process: Nursing Process: Assessment MSC: Area of Client Needs: Physiologic Integrity: Physiologic Adaptation

32. The nurse should teach parents of a preschool child with type 1 diabetes that which can raise the blood glucose level? a. Exercise b. Steroids c. Decreased food intake d. Lantus insulin

b. Steroids ANS: B Parents should understand how to adjust food, activity, and insulin at the time of illness or when the child is treated for an illness with a medication known to raise the blood glucose level (e.g., steroids). Exercise, insulin, and decreased food intake can cause hypoglycemia. DIF: Cognitive Level: Apply REF: p. 939 TOP: Integrated Process: Teaching/Learning MSC: Area of Client Needs: Physiologic Integrity: Pharmacologic and Parenteral Therapies

A 9 month old is sitting on his father's lap at the bedside. The nurse needs to do a shift assessment. How should the nurse proceed? a. Ask the father to put the child in bed, and proceed with the exam. b. Talk with the father for a few minutes, before examining child. c. Listen to the heart and lungs of the child. d. Take the child from the father, and proceed with the exam.

b. Talk with the father for a few minutes, before examining child.

Which hospitalized child would the nurse be most worried about as needing support or follow-up? a. The 9-month-old that cries when the nurse walks in the room. b. The 2-year-old that holds still during an IV start. c. An adolescent that asks her father to leave the room during an assessment. d. A school-age child that angrily throws his food tray on the floor.

b. The 2-year-old that holds still during an IV start.

2. Which expected appearance will the nurse explain to parents of an infant returning from surgery after an enucleation was performed to treat retinoblastoma? (Select all that apply.) a. A lot of drainage will come from the affected socket. b. The face may be edematous or ecchymotic. c. The eyelids will be sutured shut for the first week. d. There will be an eye pad dressing taped over the surgical site. e. The implanted sphere is covered with conjunctiva and resembles the lining of the mouth.

b. The face may be edematous or ecchymotic. d. There will be an eye pad dressing taped over the surgical site. e. The implanted sphere is covered with conjunctiva and resembles the lining of the mouth. ANS: B, D, E After enucleation surgery, the parents are prepared for the child's facial appearance. An eye patch is in place, and the child's face may be edematous or ecchymotic. Parents often fear seeing the surgical site because they imagine a cavity in the skull. A surgically implanted sphere maintains the shape of the eyeball, and the implant is covered with conjunctiva. When the eyelids are open, the exposed area resembles the mucosal lining of the mouth. The dressing, consisting of an eye pad taped over the surgical site, is changed daily. The wound itself is clean and has little or no drainage. So expecting a lot of drainage is not accurate to tell parents. The eyelids are not sutured shut after enucleation surgery. DIF: Cognitive Level: Apply REF: p. 839 TOP: Integrated Process: Teaching/Learning MSC: Area of Client Needs: Health Promotion and Maintenance

4. An adolescent is being seen in the clinic for evaluation of acromegaly. The nurse understands that which occurs with acromegaly? a. There is a lack of growth hormone (GH) being produced. b. There is excess growth hormone (GH) after closure of the epiphyseal plates. c. There is an excess of growth hormone (GH) before the closure of the epiphyseal plates. d. There is a lack of thyroid hormone being produced.

b. There is excess growth hormone (GH) after closure of the epiphyseal plates. ANS: B Excess GH after closure of the epiphyseal plates results in acromegaly. A lack of growth hormone results in delayed growth or even dwarfism. Gigantism occurs when there is hypersecretion of GH before the closure of the epiphyseal plates. Cretinism is associated with hypothyroidism. DIF: Cognitive Level: Understand REF: p. 912 TOP: Integrated Process: Nursing Process: Assessment MSC: Area of Client Needs: Physiologic Integrity: Reduction of Risk Potential

35. A ventilator-dependent child is cared for at home by his parents. Nurses come for 4 hours each day giving the parents some relief. Which other strategy should the nurse recommend to give the parents a break from the responsibilities of caring for a ventilator-dependent child? a. Encourage members from the parent's church group to provide some relief care. b. Train a trusted grandparent to provide an occasional break from the responsibilities of care. c. Encourage the parents to pay out of pocket for additional private duty nurses. d. Suggest the parents place the child in a care facility.

b. Train a trusted grandparent to provide an occasional break from the responsibilities of care. ANS: B Respite care provides temporary relief to parents and allows a break from the responsibilities of caring for the ventilator-dependent child on a daily basis. For example, a trusted and trained grandparent or extended family member may be called in to give the family a break from caring for the child. Members of the parent's church group would not have the training necessary to care for a ventilator-dependent child. Asking the parents to pay out of pocket for additional care would put a financial burden on the family. Suggesting the family place the child in a care facility is inappropriate. DIF: Cognitive Level: Apply REF: p. 842 TOP: Integrated Process: Nursing Process: Planning MSC: Area of Client Needs: Psychosocial Integrity

19. A 5-year-old boy is being prepared for surgery to remove a brain tumor. Nursing actions should be based on which statement? a. Removal of tumor will stop the various symptoms. b. Usually the postoperative dressing covers the entire scalp. c. He is not old enough to be concerned about his head being shaved. d. He is not old enough to understand the significance of the brain.

b. Usually the postoperative dressing covers the entire scalp. ANS: B The child should be told what he will look and feel like after surgery. This includes the size of the dressing. The nurse can demonstrate on a doll the expected size and shape of the dressing. Some of the symptoms may be alleviated by the removal of the tumor, but postsurgical headaches and cerebellar symptoms such as ataxia may be aggravated. Children should be prepared for the loss of their hair, and it should be removed in a sensitive, positive manner if the child is awake. Children at this age have poorly defined body boundaries and little knowledge of internal organs. Intrusive experiences are frightening, especially those that disrupt the integrity of the skin. DIF: Cognitive Level: Apply REF: p. 831 TOP: Integrated Process: Teaching/Learning MSC: Area of Client Needs: Health Promotion and Maintenance

24. Which immunization should be given with caution to children infected with human immunodeficiency virus (HIV)? a. Influenza b. Varicella c. Pneumococcal d. Inactivated poliovirus (IPV)

b. Varicella ANS: B The children should be carefully evaluated before being given live viral vaccines such as varicella, measles, mumps, and rubella. The child must be immunocompetent and not have contact with other severely immunocompromised individuals. Influenza, pneumococcal, and inactivated poliovirus (IPV) are not live vaccines. DIF: Cognitive Level: Apply REF: p. 806 TOP: Integrated Process: Nursing Process: Implementation MSC: Area of Client Needs: Physiologic Integrity: Pharmacologic and Parenteral Therapies

A nurse is conducting an admission interview with the mother of a 2 year old child. What history question is most important at this time? a. How many children are in the family? b. What is your child's normal routine? c. Does your child attend daycare? d. What toys are most important to your child?

b. What is your child's normal routine?

32. One of the supervisors for a home health agency asks the nurse to give the family a survey evaluating the nurses and other service providers. The nurse should recognize this as: a. inappropriate, unless nurses are able to evaluate family. b. appropriate to improve quality of care. c. inappropriate, unless nurses and other providers agree to participate. d. inappropriate, because family lacks knowledge necessary to evaluate professionals.

b. appropriate to improve quality of care ANS: B Quality assessment and improvement activities are essential for virtually all organizations. Family involvement is essential in evaluating a home care plan and can occur on several levels. The nurse can ask the family open-ended questions at regular intervals to assess their opinion of the effectiveness of care. Families should also be given an opportunity to evaluate the individual home care nurses, the home care agency, and other service providers periodically. The nurse is the care provider. The evaluation is of the provision of care to the patient and family. The nurse's role is not to evaluate the family. Quality- monitoring activities are required by virtually all health care agencies. During the evaluation process, the family is requested to provide their perceptions of care. DIF: Cognitive Level: Apply REF: p. 842 TOP: Integrated Process: Communication and Documentation MSC: Area of Client Needs: Safe and Effective Care Environment: Management of Care

6. The nurse is conducting a staff in-service on childhood endocrine disorders. Diabetes insipidus is a disorder of: a. anterior pituitary. b. posterior pituitary. c. adrenal cortex. d. adrenal medulla.

b. posterior pituitary. ANS: B The principal disorder of posterior pituitary hypofunction is diabetes insipidus. The anterior pituitary produces hormones such as GH, thyroid-stimulating hormone, adrenocorticotropic hormone, gonadotropin, prolactin, and melanocyte-stimulating hormone. The adrenal cortex produces aldosterone, sex hormones, and glucocorticoids. The adrenal medulla produces catecholamines. DIF: Cognitive Level: Understand REF: p. 912 TOP: Integrated Process: Teaching/Learning MSC: Area of Client Needs: Physiologic Integrity: Physiologic Adaptation

27. When taking the history of a child hospitalized with Reye syndrome, the nurse should not be surprised that a week ago the child had recovered from: a. measles. b. varicella. c. meningitis. d. hepatitis.

b. varicella. ANS: B Most cases of Reye syndrome follow a common viral illness such as varicella or influenza. Measles, meningitis, and hepatitis are not associated with Reye syndrome. DIF: Cognitive Level: Understand REF: p. 895 TOP: Integrated Process: Nursing Process: Planning MSC: Area of Client Needs: Physiologic Integrity: Physiologic Adaptation

19. The nurse is explaining to a parent how to care for a school-age child with vomiting associated with a viral illness. Which action should the nurse include? a. Avoid carbohydrate-containing liquids. b. Give nothing by mouth for 24 hours. c. Brush teeth or rinse mouth after vomiting. d. Give plain water until vomiting ceases for at least 24 hours.

c. Brush teeth or rinse mouth after vomiting. ANS: C It is important to emphasize the need for the child to brush the teeth or rinse the mouth after vomiting to dilute the hydrochloric acid that comes in contact with the teeth. Ad libitum administration of glucose-electrolyte solution to an alert child will help restore water and electrolytes satisfactorily. It is important to include carbohydrate to spare body protein and avoid ketosis. DIF: Cognitive Level: Apply REF: p. 709 TOP: Integrated Process: Teaching/Learning MSC: Area of Client Needs: Physiologic Integrity: Reduction of Risk Potential

27. Which is used to treat moderate to severe inflammatory bowel disease? a. Antacids b. Antibiotics c. Corticosteroids d. Antidiarrheal medications

c. Corticosteroids ANS: C Corticosteroids, such as prednisone and prednisolone, are used in short bursts to suppress the inflammatory response in inflammatory bowel disease. Antacids and antidiarrheal medications are not drugs of choice in the treatment of inflammatory bowel disease. Antibiotics may be used as an adjunctive therapy to treat complications. DIF: Cognitive Level: Understand REF: p. 713 TOP: Integrated Process: Nursing Process: Planning MSC: Area of Client Needs: Physiologic Integrity: Pharmacologic and Parenteral Therapies

2. The nurse is admitting a school-age child in acute renal failure with reduced glomerular filtration rate. Which urine test is the most useful clinical indication of glomerular filtration rate? a. pH b. Osmolality c. Creatinine d. Protein level

c. Creatinine ANS: C The most useful clinical indication of glomerular filtration is the clearance of creatinine. It is a substance that is freely filtered by the glomerulus and secreted by the renal tubule cells. The pH and osmolality are not estimates of glomerular filtration. Although protein in the urine demonstrates abnormal glomerular permeability, it is not a measure of filtration rate. PTS: 1 DIF: Cognitive Level: Understand REF: 904 TOP: Integrated Process: Nursing Process: Assessment MSC: Area of Client Needs: Physiologic Integrity: Reduction of Risk Potential

19. Which is the most appropriate nursing diagnosis for the child with acute glomerulonephritis? a. Risk for Injury related to malignant process and treatment b. Fluid Volume Deficit related to excessive losses c. Fluid Volume Excess related to decreased plasma filtration d. Fluid Volume Excess related to fluid accumulation in tissues and third spaces

c. Fluid Volume Excess related to decreased plasma filtration Glomerulonephritis has a decreased filtration of plasma, which results in an excessive accumulation of water and sodium that expands plasma and interstitial fluid volumes, leading to circulatory congestion and edema. No malignant process is involved in acute glomerulonephritis. A fluid volume excess is found. The fluid accumulation is secondary to the decreased plasma filtration. PTS: 1 DIF: Cognitive Level: Analyze REF: 915 TOP: Integrated Process: Nursing Process: Nursing Diagnosis MSC: Area of Client Needs: Physiologic Integrity: Physiologic Adaptation

25. A nurse is conducting an in-service on childhood gastrointestinal disorders. Which statement is most descriptive of Meckel diverticulum? a. It is more common in females than in males. b. It is acquired during childhood. c. Intestinal bleeding may be mild or profuse. d. Medical interventions are usually sufficient to treat the problem.

c. Intestinal bleeding may be mild or profuse. ANS: C Bloody stools are often a presenting sign of Meckel diverticulum. It is associated with mild to profuse intestinal bleeding. It is twice as common in males as in females, and complications are more frequent in males. Meckel diverticulum is the most common congenital malformation of the GI tract and is present in 1% to 4% of the general population. The standard therapy is surgical removal of the diverticulum. DIF: Cognitive Level: Apply REF: p. 710 TOP: Integrated Process: Teaching/Learning MSC: Area of Client Needs: Physiologic Integrity: Physiologic Adaptation

29. Which should the nurse recommend for the diet of a child with chronic renal failure? a. High in protein b. Low in vitamin D c. Low in phosphorus d. Supplemented with vitamins A, E, and K

c. Low in phosphorus ANS: C Dietary phosphorus is controlled by the reduction of protein and milk intake to prevent or control the calcium-phosphorus imbalance. Protein should be limited in chronic renal failure to decrease intake of phosphorus. Vitamin D therapy is administered in chronic renal failure to increase calcium absorption. Supplementation of vitamins A, E, and K is not part of dietary management in chronic renal disease. PTS: 1 DIF: Cognitive Level: Apply REF: 922 TOP: Integrated Process: Nursing Process: Implementation MSC: Area of Client Needs: Physiologic Integrity: Basic Care and Comfort

10. Which therapeutic management should the nurse prepare to initiate first for a child with acute diarrhea and moderate dehydration? a. Clear liquids b. Adsorbents, such as kaolin and pectin c. Oral rehydration solution (ORS) d. Antidiarrheal medications such as paregoric

c. Oral rehydration solution (ORS) ANS: C ORS is the first treatment for acute diarrhea. Clear liquids are not recommended because they contain too much sugar, which may contribute to diarrhea. Adsorbents are not recommended. Antidiarrheals are not recommended because they do not get rid of pathogens. DIF: Cognitive Level: Apply REF: p. 700 TOP: Integrated Process: Nursing Process: Implementation MSC: Area of Client Needs: Physiologic Integrity: Reduction of Risk Potential

41. Which observation made of the exposed abdomen is most indicative of pyloric stenosis? a. Abdominal rigidity b. Substernal retraction c. Palpable olive-like mass d. Marked distention of lower abdomen

c. Palpable olive-like mass ANS: C The diagnosis of pyloric stenosis is often made after the history and physical examination. The olive-like mass is easily palpated when the stomach is empty, the infant is quiet, and the abdominal muscles are relaxed. Abdominal rigidity and substernal retraction are usually not present. The upper abdomen, not lower abdomen, is distended. DIF: Cognitive Level: Understand REF: p. 728 TOP: Integrated Process: Nursing Process: Assessment MSC: Area of Client Needs: Physiologic Integrity: Reduction of Risk Potential

5. An infant is brought to the emergency department with dehydration. Which physical assessment finding does the nurse expect? a. Weight gain b. Bradycardia c. Poor skin turgor d. Brisk capillary refill

c. Poor skin turgor ANS: C Clinical manifestations of dehydration include poor skin turgor, weight loss, lethargy, and tachycardia. The infant would have prolonged capillary refill, not brisk. DIF: Cognitive Level: Understand REF: p. 691 TOP: Integrated Process: Nursing Process: Assessment MSC: Area of Client Needs: Physiologic Integrity: Physiologic Adaptation

37. During the first few days after surgery for cleft lip, which intervention should the nurse do? a. Leave infant in crib at all times to prevent suture strain. b. Keep infant heavily sedated to prevent suture strain. c. Remove restraints periodically to cuddle infant. d. Alternate position from prone to side-lying to supine.

c. Remove restraints periodically to cuddle infant. ANS: C Remove restraints periodically, while supervising the infant, to allow him or her to exercise arms and to provide cuddling and tactile stimulation. The infant should not be left in the crib, but should be removed for appropriate holding and stimulation. Analgesia and sedation are administered for pain. Heavy sedation is not indicated. The child should not be placed in the prone position. DIF: Cognitive Level: Apply REF: p. 725 TOP: Integrated Process: Nursing Process: Planning MSC: Area of Client Needs: Physiologic Integrity: Reduction of Risk Potential

7. Which pathogen is the viral pathogen that frequently causes acute diarrhea in young children? a. Giardia organisms b. Shigella organisms c. Rotavirus d. Salmonella organisms

c. Rotavirus ANS: C Rotavirus is the most frequent viral pathogen that causes diarrhea in young children. Giardia (parasite) and Salmonella are bacterial pathogens that cause diarrhea. Shigella is a bacterial pathogen that is uncommon in the United States. DIF: Cognitive Level: Understand REF: p. 697 TOP: Integrated Process: Nursing Process: Assessment MSC: Area of Client Needs: Physiologic Integrity: Physiologic Adaptation

23. When caring for a child with probable appendicitis, the nurse should be alert to recognize that which condition or symptom is a sign of perforation? a. Bradycardia b. Anorexia c. Sudden relief from pain d. Decreased abdominal distention

c. Sudden relief from pain ANS: C Signs of peritonitis, in addition to fever, include sudden relief from pain after perforation. Tachycardia, not bradycardia, is a manifestation of peritonitis. Anorexia is already a clinical manifestation of appendicitis. Abdominal distention usually increases. DIF: Cognitive Level: Understand REF: p. 709 TOP: Integrated Process: Nursing Process: Assessment MSC: Area of Client Needs: Physiologic Integrity: Physiologic Adaptation

2. A nurse is conducting an in-service on gastrointestinal disorders. The nurse includes that melena, the passage of black, tarry stools, suggests bleeding from which area? a. Perianal or rectal area b. Hemorrhoids or anal fissures c. Upper gastrointestinal (GI) tract d. Lower GI tract

c. Upper gastrointestinal (GI) tract ANS: C Melena is denatured blood from the upper GI tract or bleeding from the right colon. Blood from the perianal or rectal area, hemorrhoids, or lower GI tract would be bright red. DIF: Cognitive Level: Apply REF: p. 696 TOP: Integrated Process: Teaching/Learning MSC: Area of Client Needs: Physiologic Integrity: Reduction of Risk Potential

42. The nurse is caring for an infant with suspected pyloric stenosis. Which clinical manifestation would indicate pyloric stenosis? a. Abdominal rigidity and pain on palpation b. Rounded abdomen and hypoactive bowel sounds c. Visible peristalsis and weight loss d. Distention of lower abdomen and constipation

c. Visible peristalsis and weight loss pg 728 ANS: C Visible gastric peristaltic waves that move from left to right across the epigastrium and weight loss are observed in pyloric stenosis. Abdominal rigidity and pain on palpation or a rounded abdomen and hypoactive bowel sounds are usually not present. The upper abdomen, not lower abdomen, is distended.

25. Which is a major complication in a child with chronic renal failure? a. Hypokalemia b. Metabolic alkalosis c. Water and sodium retention d. Excessive excretion of blood urea nitrogen

c. Water and sodium retention ANS: C Chronic renal failure leads to water and sodium retention, which contributes to edema and vascular congestion. Hyperkalemia, metabolic acidosis, and retention of blood urea nitrogen are complications of chronic renal failure. PTS: 1 DIF: Cognitive Level: Understand REF: 921 TOP: Integrated Process: Nursing Process: Evaluation MSC: Area of Client Needs: Physiologic Integrity: Physiologic Adaptation

18. A 3-year-old child with Hirschsprung disease is hospitalized for surgery. A temporary colostomy will be necessary. The nurse should recognize that preparing this child psychologically is: a. not necessary because of child's age. b. not necessary because colostomy is temporary. c. necessary because it will be an adjustment. d. necessary because the child must deal with a negative body image.

c. necessary because it will be an adjustment. ANS: C The child's age dictates the type and extent of psychological preparation. When a colostomy is performed, the child who is at least preschool age is told about the procedure and what to expect in concrete terms, with the use of visual aids. It is necessary to prepare a 3-year-old child for procedures. The preschooler is not yet concerned with body image. DIF: Cognitive Level: Understand REF: p. 705 TOP: Integrated Process: Teaching/Learning MSC: Area of Client Needs: Psychosocial Integrity: Coping and Adaptation

17. The nurse notes that a child has lost 8 pounds after 4 days of hospitalization for acute glomerulonephritis. This is most likely the result of: a. poor appetite. b. increased potassium intake. c. reduction of edema. d. restriction to bed rest.

c. reduction of edema. ANS: C This amount of weight loss in this period is a result of the improvement of renal function and mobilization of edema fluid. Poor appetite and bed rest would not result in a weight loss of 8 pounds in 4 days. Foods with substantial amounts of potassium are avoided until renal function is normalized. PTS: 1 DIF: Cognitive Level: Understand REF: 860 TOP: Integrated Process: Nursing Process: Assessment MSC: Area of Client Needs: Physiologic Integrity: Physiologic Adaptation

22. A 5-year-old girl sustained a concussion when she fell out of a tree. In preparation for discharge, the nurse is discussing home care with her mother. Which statement made by the mother indicates a correct understanding of the teaching? a. "I should expect my child to have a few episodes of vomiting." b. "If I notice sleep disturbances, I should contact the physician immediately." c. "I should expect my child to have some behavioral changes after the accident." d. "If I notice diplopia, I will have my child rest for 1 hour."

c. "I should expect my child to have some behavioral changes after the accident." ANS: C The parents are advised of probable posttraumatic symptoms that may be expected. These include behavioral changes and sleep disturbances. If the child has these clinical signs, they should be immediately reported for evaluation. Sleep disturbances are to be expected. DIF: Cognitive Level: Apply REF: p. 883 TOP: Integrated Process: Teaching/Learning MSC: Area of Client Needs: Physiologic Integrity: Physiologic Adaptation

12. The nurse is teaching the parents of a child who is receiving methimazole (Tapazole) for the treatment of hyperthyroidism (Graves disease). Which statement made by the parent indicates a correct understanding of the teaching? a. "I would expect my child to gain weight while taking this medication." b. "I would expect my child to experience episodes of ear pain while taking this medication." c. "If my child develops a sore throat and fever, I should contact the physician immediately." d. "If my child develops the stomach flu, my child will need to be hospitalized."

c. "If my child develops a sore throat and fever, I should contact the physician ANS: C Children being treated with Tapazole must be carefully monitored for the side effects of the medication. Parents must be alerted that sore throat and fever accompany the grave complication of leukopenia. These symptoms should be immediately reported. Weight gain, episodes of ear pain, and concern for hospitalization with the stomach flu are not concerns related to taking Tapazole. DIF: Cognitive Level: Apply REF: p. 920 TOP: Integrated Process: Teaching/Learning MSC: Area of Client Needs: Physiologic Integrity: Pharmacologic and Parenteral Therapies

23. A 3-year-old child is hospitalized after a submersion injury. The child's mother complains to the nurse, "Being at the hospital seems unnecessary when he is perfectly fine." What is the nurse's best reply? a. "He still needs a little extra oxygen." b. "I'm sure he is fine, but the doctor wants to make sure." c. "The reason for this is that complications could still occur." d. "It is important to observe for possible central nervous system problems."

c. "The reason for this is that complications could still occur." ANS: C All children who have a submersion injury should be admitted to the hospital for observation. Although many children do not appear to have suffered adverse effects from the event, complications such as respiratory compromise and cerebral edema may occur 24 hours after the incident. The mother would not think the child is fine if oxygen were still required. The nurse should clarify that different complications can occur up to 24 hours later and that observations are necessary. DIF: Cognitive Level: Apply REF: p. 888 TOP: Integrated Process: Teaching/Learning MSC: Area of Client Needs: Physiologic Integrity: Physiologic Adaptation

30. A child is brought to the emergency department after experiencing a seizure at school. There is no previous history of seizures. The father tells the nurse that he cannot believe the child has epilepsy. What is the nurse's best response? a. "Epilepsy is easily treated." b. "Very few children have actual epilepsy." c. "The seizure may or may not mean that your child has epilepsy." d. "Your child has had only one convulsion; it probably won't happen again."

c. "The seizure may or may not mean that your child has epilepsy." ANS: C Seizures are the indispensable characteristic of epilepsy; however, not every seizure is epileptic. Epilepsy is a chronic seizure disorder with recurrent and unprovoked seizures. The treatment of epilepsy involves a thorough assessment to determine the type of seizure the child is having and the cause, followed by individualized therapy to allow the child to have as normal a life as possible. The nurse should not make generalized comments regarding the incidence of epilepsy until further assessment is made. DIF: Cognitive Level: Apply REF: p. 896 TOP: Integrated Process: Teaching/Learning MSC: Area of Client Needs: Physiologic Integrity: Physiologic Adaptation

45. A preschool child is scheduled for an echocardiogram. Parents ask the nurse whether they can hold the child during the procedure. The nurse should answer with which response? a. "You will be able to hold your child during the procedure." b. "Your child can be active during the procedure, but can't sit in your lap." c. "Your child must lie quietly; sometimes a mild sedative is administered before the procedure." d. "The procedure is invasive so your child will be restrained during the echocardiogram."

c. "Your child must lie quietly; sometimes a mild sedative is administered before the procedure." ANS: C Although an echocardiogram is noninvasive, painless, and associated with no known side effects, it can be stressful for children. The child must lie quietly in the standard echocardiographic positions; crying, nursing, or sitting up often leads to diagnostic errors or omissions. Therefore, infants and young children may need a mild sedative; older children benefit from psychological preparation for the test. The distraction of a video or movie is often helpful. DIF: Cognitive Level: Apply REF: p. 740 TOP: Integrated Process: Teaching/Learning MSC: Area of Client Needs: Physiologic Integrity: Physiologic Adaptation

16. A 6-month-old infant is receiving digoxin (Lanoxin). The nurse should notify the practitioner and withhold the medication if the apical pulse is less than _______ beats/min. a. 60 b. 70 c. 90 to 110 d. 110 to 120

c. 90 to 110 ANS: C If the 1-minute apical pulse is below 90 to 110 beats/min, the digoxin should not be given to a 6-month- old. Sixty beats/min is the cut-off for holding the digoxin dose in an adult; 70 beats/min is the determining heart rate to hold a dose of digoxin for an older child; 110 to 120 beats/min is an acceptable heart rate to administer digoxin to a 6-month-old. DIF: Cognitive Level: Apply REF: p. 752 TOP: Integrated Process: Nursing Process: Implementation MSC: Area of Client Needs: Physiologic Integrity: Pharmacologic and Parenteral Therapies

36. A child has been seizure-free for 2 years. A father asks the nurse how much longer the child will need to take the antiseizure medications. The nurse includes which intervention in the response? a. Medications can be discontinued at this time. b. The child will need to take the drugs for 5 years after the last seizure. c. A step-wise approach will be used to reduce the dosage gradually. d. Seizure disorders are a lifelong problem. Medications cannot be discontinued.

c. A step-wise approach will be used to reduce the dosage gradually. ANS: C A predesigned protocol is used to wean a child gradually off antiseizure medications, usually when the child is seizure-free for 2 years and has a normal electroencephalogram (EEG). Medications must be gradually reduced to minimize the recurrence of seizures. Seizure medications can be safely discontinued. The risk of recurrence is greatest within the first year. DIF: Cognitive Level: Apply REF: p. 899 TOP: Integrated Process: Teaching/Learning MSC: Area of Client Needs: Physiologic Integrity: Pharmacologic and Parenteral Therapies

19. Which is characteristic of the immune-mediated type 1 diabetes mellitus? a. Ketoacidosis is infrequent. b. Onset is gradual. c. Age at onset is usually younger than 20 years. d. Oral agents are often effective for treatment.

c. Age at onset is usually younger than 20 years. ANS: C The immune-mediated type 1 diabetes mellitus typically has its onset in children or young adults. Infrequent ketoacidosis, gradual onset, and effectiveness of oral agents for treatment are more consistent with type 2 diabetes. DIF: Cognitive Level: Analyze REF: p. 931 TOP: Integrated Process: Nursing Process: Assessment MSC: Area of Client Needs: Physiologic Integrity: Physiologic Adaptation

2. Several blood tests are ordered for a preschool child with severe anemia. The child is crying and upset because of memories of the venipuncture done at the clinic 2 days ago. What should the nurse explain? a. The venipuncture discomfort is very brief b. Only one venipuncture will be needed c. A topical application of local anesthetic can eliminate venipuncture pain d. Most blood tests on children require only a finger puncture because a small amount of blood is needed

c. A topical application of local anesthetic can eliminate venipuncture pain ANS: C Preschool children are concerned with both pain and the loss of blood. When preparing the child for venipuncture, the nurse will use a topical anesthetic to eliminate any pain. This is a traumatic experience for preschool children. They are concerned about their bodily integrity. A local anesthetic should be used, and a bandage should be applied to maintain bodily integrity. The nurse should not promise one attempt in case multiple attempts are required. Both finger punctures and venipunctures are traumatic for children. Both require preparation. DIF: Cognitive Level: Apply REF: p. 789 TOP: Integrated Process: Teaching/Learning MSC: Area of Client Needs: Health Promotion and Maintenance

22. Which is caused by a virus that primarily infects a specific subset of T lymphocytes, the CD4+ T cells? a. Wiskott-Aldrich syndrome b. Idiopathic thrombocytopenic purpura c. Acquired immunodeficiency syndrome (AIDS) d. Severe combined immunodeficiency disease

c. Acquired immunodeficiency syndrome (AIDS) ANS: C AIDS is caused by the human immunodeficiency virus (HIV), which primarily attacks the CD4+ T cells. Wiskott-Aldrich syndrome, idiopathic thrombocytopenic purpura, and severe combined immunodeficiency disease are not viral illnesses. DIF: Cognitive Level: Remember REF: p. 806 TOP: Integrated Process: Nursing Process: Assessment MSC: Area of Client Needs: Physiologic Integrity: Physiologic Adaptation

6. Which should the nurse include when teaching the mother of a 9-month-old infant about administering liquid iron preparations? a. They should be given with meals. b. They should be stopped immediately if nausea and vomiting occur. c. Adequate dosage will turn the stools a tarry green color. d. Allow preparation to mix with saliva and bathe the teeth before swallowing.

c. Adequate dosage will turn the stools a tarry green color. ANS: C The nurse should prepare the mother for the anticipated change in the child's stools. If the iron dose is adequate, the stools will become a tarry green color. The lack of the color change may indicate insufficient iron. The iron should be given in two divided doses between meals when the presence of free hydrochloric acid is greatest. Iron is absorbed best in an acidic environment. Vomiting and diarrhea may occur with iron administration. If these occur, the iron should be given with meals, and the dosage reduced, then gradually increased as the child develops tolerance. Liquid preparations of iron stain the teeth. They should be administered through a straw and the mouth rinsed after administration. DIF: Cognitive Level: Apply REF: p. 789 TOP: Integrated Process: Teaching/Learning

12. A school-age child with leukemia experienced severe nausea and vomiting when receiving chemotherapy for the first time. Which is the most appropriate nursing action to prevent or minimize these reactions with subsequent treatments? a. Encourage drinking large amounts of favorite fluids. b. Encourage child to take nothing by mouth (remain NPO) until nausea and vomiting subside. c. Administer an antiemetic before chemotherapy begins. d. Administer an antiemetic as soon as child has nausea.

c. Administer an antiemetic before chemotherapy begins ANS: C The most beneficial regimen to minimize nausea and vomiting associated with chemotherapy is to administer the antiemetic before the chemotherapy is begun. The goal is to prevent anticipatory symptoms. Drinking fluids will add to the discomfort of the nausea and vomiting. Waiting until nausea and vomiting subside will help with this episode, but the child will have the discomfort and be at risk for dehydration. Administering an antiemetic as soon as the child has nausea does not prevent anticipatory nausea. DIF: Cognitive Level: Apply REF: p. 826 TOP: Integrated Process: Nursing Process: Implementation MSC: Area of Client Needs: Physiologic Integrity: Pharmacologic and Parenteral Therapies.

28. Which is an important nursing consideration when chest tubes will be removed from a child? a. Explain that it is not painful. b. Explain that only a Band-Aid will be needed. c. Administer analgesics before the procedure. d. Expect bright red drainage for several hours after removal.

c. Administer analgesics before the procedure. ANS: C It is appropriate to prepare the child for the removal of chest tubes with analgesics. Short-acting medications can be used that are administered through an existing IV line. A sharp, momentary pain is felt. This should not be misrepresented to the child. A petroleum gauze, air-tight dressing will be needed, but it is not a pain-free procedure. Little or no drainage should be found on removal. DIF: Cognitive Level: Apply REF: p. 764 TOP: Integrated Process: Nursing Process: Planning MSC: Area of Client Needs: Physiologic Integrity: Pharmacologic and Parenteral Therapies

26. Which is an important nursing consideration when suctioning a young child who has had heart surgery? a. Perform suctioning at least every hour. b. Suction for no longer than 30 seconds at a time. c. Administer supplemental oxygen before and after suctioning. d. Expect symptoms of respiratory distress when suctioning.

c. Administer supplemental oxygen before and after suctioning. ANS: C If suctioning is indicated, supplemental oxygen is administered with a manual resuscitation bag before and after the procedure to prevent hypoxia. Suctioning should be done only as indicated, not on a routine basis. The child should be suctioned for no more than 5 seconds at one time. Symptoms of respiratory distress are avoided by using appropriate technique. DIF: Cognitive Level: Apply REF: p. 764 TOP: Integrated Process: Nursing Process: Implementation MSC: Area of Client Needs: Physiologic Integrity: Physiologic Adaptation

15. What secretes glucocorticoids, mineralocorticoids, and sex steroids? a. Thyroid gland b. Parathyroid glands c. Adrenal cortex d. Anterior pituitary

c. Adrenal cortex ANS: C These hormones are secreted by the adrenal cortex. The thyroid gland produces thyroid hormone and thyrocalcitonin. The parathyroid gland produces parathyroid hormone. The anterior pituitary produces hormones such as GH, thyroid-stimulating hormone, adrenocorticotropic hormone, gonadotropin, prolactin, and melanocyte-stimulating hormone. DIF: Cognitive Level: Understand REF: p. 923 TOP: Integrated Process: Nursing Process: Assessment MSC: Area of Client Needs: Physiologic Integrity: Physiologic Adaptation

29. A young child's parents call the nurse after their child was bitten by a raccoon in the woods. The nurse's recommendation should be based on which statement? a. The child should be hospitalized for close observation. b. No treatment is necessary if thorough wound cleaning is done. c. Antirabies prophylaxis must be initiated. d. Antirabies prophylaxis must be initiated if clinical manifestations appear.

c. Antirabies prophylaxis must be initiated. ANS: C Current therapy for a rabid animal bite consists of a thorough cleansing of the wound and passive immunization with human rabies immune globulin (HRIG) as soon as possible. Hospitalization is not necessary. The wound cleansing, passive immunization, and immune globulin administration can be done as an outpatient. The child needs to receive both HRIG and rabies vaccine. DIF: Cognitive Level: Apply REF: p. 895 TOP: Integrated Process: Teaching/Learning MSC: Area of Client Needs: Physiologic Integrity: Physiologic Adaptation

1. The nurse has documented that a child's level of consciousness is obtunded. Which describes this level of consciousness? a. Slow response to vigorous and repeated stimulation b. Impaired decision making c. Arousable with stimulation d. Confusion regarding time and place

c. Arousable with stimulation ANS: C Obtunded describes a level of consciousness in which the child is arousable with stimulation. Stupor is a state in which the child remains in a deep sleep, responsive only to vigorous and repeated stimulation. Confusion is impaired decision making. Disorientation is confusion regarding time and place. DIF: Cognitive Level: Understand REF: p. 874 TOP: Integrated Process: Nursing Process: Assessment MSC: Area of Client Needs: Physiologic Integrity: Physiologic Adaptation

Nursing interventions for the child after a cardiac catheterization should include which actions? (Select all that apply.) a. Allow ambulation as tolerated. b. Monitor vital signs every 2 hours. c. Assess the affected extremity for temperature and color. d. Check pulses above the catheterization site for equality and symmetry. e. Remove pressure dressing after 4 hours. f. Maintain a patent peripheral intravenous catheter until discharge.

c. Assess the affected extremity for temperature and color. f. Maintain a patent peripheral intravenous catheter until discharge. ANS: C, F The extremity that was used for access for the cardiac catheterization must be checked for temperature and color. Coolness and blanching may indicate arterial occlusion. The child should have a patent peripheral intravenous line (PIV) to ensure adequate hydration. The child should remain on bed rest with the leg extended for a minimum of 4 hours. Initially vital signs are taken every 15 minutes, with emphasis on a heart rate counted for 1 minute. Pulses above the catheterization site should not be affected by the catheterization. Pulses distal to the site should be monitored. The pressure dressings should not be removed for 24 hours. DIF: Cognitive Level: Apply REF: p. 748 TOP: Integrated Process: Nursing Process: Implementation MSC: Area of Client Needs: Physiologic Integrity: Reduction of Risk Potential

8. Which defect results in increased pulmonary blood flow? a. Pulmonic stenosis b. Tricuspid atresia c. Atrial septal defect d. Transposition of the great arteries

c. Atrial septal defect ANS: C Atrial septal defect results in increased pulmonary blood flow. Blood flows from the left atrium (higher pressure) into the right atrium (lower pressure) and then to the lungs via the pulmonary artery. Pulmonic stenosis is an obstruction to blood flowing from the ventricles. Tricuspid atresia results in decreased pulmonary blood flow. Transposition of the great arteries results in mixed blood flow.

21. The nurse is monitoring a 7-year-old child post surgical resection of an infratentorial brain tumor. Which vital sign findings indicate Cushing's triad? a. Increased temperature, tachycardia, tachypnea b. Decreased temperature, bradycardia, bradypnea c. Bradycardia, hypertension, irregular respirations d. Bradycardia, hypotension, tachypnea

c. Bradycardia, hypertension, irregular respirations ANS: C Cushing's triad is a hallmark sign of increased intracranial pressure (ICP). The triad includes bradycardia, hypertension, and irregular respirations. Increased or decreased temperature is not a sign of Cushing's triad. DIF: Cognitive Level: Understand REF: p. 831 TOP: Integrated Process: Nursing Process: Assessment MSC: Area of Client Needs: Physiologic Integrity: Physiologic Adaptation

8. Which neurologic diagnostic test gives a visualized horizontal and vertical cross-section of the brain at any axis? a. Nuclear brain scan b. Echoencephalography c. CT scan d. Magnetic resonance imaging (MRI)

c. CT scan ANS: C A CT scan provides a visualization of the horizontal and vertical cross-sections of the brain at any axis. A nuclear brain scan uses a radioisotope that accumulates where the blood-brain barrier is defective. Echoencephalography identifies shifts in midline structures of the brain as a result of intracranial lesions. MRI permits visualization of morphologic features of target structures and permits tissue discrimination that is unavailable with any other techniques. DIF: Cognitive Level: Understand REF: p. 877 TOP: Integrated Process: Nursing Process: Assessment MSC: Area of Client Needs: Physiologic Integrity: Reduction of Risk Potential

23. A parent asks the nurse why self-monitoring of blood glucose is being recommended for her child with diabetes. The nurse should base the explanation on which knowledge? a. It is a less expensive method of testing. b. It is not as accurate as laboratory testing. c. Children are better able to manage the diabetes. d. Parents are better able to manage the disease.

c. Children are better able to manage the diabetes. ANS: C Blood glucose self-management has improved diabetes management and can be used successfully by children from the time of diagnosis. Insulin dosages can be adjusted based on blood glucose results. Blood glucose monitoring is more expensive but provides improved management. It is as accurate as equivalent testing done in laboratories. The ability to self-test allows the child to balance diet, exercise, and insulin. The parents are partners in the process, but the child should be taught how to manage the disease. DIF: Cognitive Level: Apply REF: p. 928 TOP: Integrated Process: Teaching/Learning MSC: Area of Client Needs: Physiologic Integrity: Physiologic Adaptation

2. A child is brought to the emergency department experiencing an anaphylactic reaction to a bee sting. Once the airway is established, the nurse should do which action? Place in correct sequence. Provide the answer using lowercase letters separated by commas (e.g., a, b, c). a. Administer epinephrine. b. Keep the child warm and calm. c. Obtain vascular access.

c. Obtain vascular access a. Administer epinephrine. b. Keep the child warm and calm. ANS: c, a, b The correct sequence of actions is to obtain vascular access, administer epinephrine, and then to keep the child warm and calm. DIF: Cognitive Level: Apply REF: p. 781 TOP: Integrated Process: Nursing Process: Implementation MSC: Area of Client Needs: Physiologic Integrity: Physiologic Adaptation

30. The home health nurse is caring for a child who requires complex care. The family expresses frustration related to obtaining accurate information about their child's illness and its management. Which is the best action for the nurse? a. Determine why family is easily frustrated. b. Refer family to child's primary care practitioner. c. Clarify family's request, and provide information they want. d. Answer only questions that family needs to know about.

c. Clarify family's request, and provide information they want. ANS: C The philosophic basis for family-centered practice is the recognition that the family is the constant in the child's life. It is essential and appropriate that the family have complete and accurate information about their child's illness and management. The nurse may first have to clarify what information the family believes has not been communicated. The family's frustration arises from their perception that they are not receiving information pertinent to their child's care. Referring the family to the child's primary care practitioner does not help the family. The home health nurse should have access to the necessary information. Questions about what they need and want to know concerning their child's care should be addressed. DIF: Cognitive Level: Apply REF: p. 842 TOP: Integrated Process: Communication and Documentation

5. The nurse is teaching parents of an infant about the causes of iron-deficiency anemia. Which statement best describes iron-deficiency anemia in infants? a. It is caused by depression of the hematopoietic system. b. It is easily diagnosed because of an infant's emaciated appearance. c. Clinical manifestations are similar regardless of the cause of the anemia. d. Clinical manifestations result from a decreased intake of milk and the preterm addition of solid foods.

c. Clinical manifestations are similar regardless of the cause of the anemia. ANS: C In iron-deficiency anemia, the child's clinical appearance is a result of the anemia, not the underlying cause. Usually the hematopoietic system is not depressed in iron-deficiency anemia. The bone marrow produces red cells that are smaller and contain less hemoglobin than normal red cells. Children who are iron deficient from drinking excessive quantities of milk are usually pale and overweight. They are receiving sufficient calories, but are deficient in essential nutrients. The clinical manifestations result from decreased intake of iron-fortified solid foods and an excessive intake of milk. DIF: Cognitive Level: Apply REF: p. 789 TOP: Integrated Process: Teaching/Learning MSC: Area of Client Needs: Physiologic Integrity: Physiologic Adaptation

2. Which clinical manifestation should the nurse expect to see as shock progresses in a child and becomes decompensated shock? (Select all that apply.) a. Thirst and diminished urinary output b. Irritability and apprehension c. Cool extremities and decreased skin turgor d. Confusion and somnolence e. Normal blood pressure and narrowing pulse pressure f. Tachypnea and poor capillary refill time

c. Cool extremities and decreased skin turgor d. Confusion and somnolence f. Tachypnea and poor capillary refill time aNS: C, D, F Cool extremities, decreased skin turgor, confusion, somnolence, tachypnea, and poor capillary refill time are beginning signs of decompensated shock. Thirst, diminished urinary output, irritability, apprehension, normal blood pressure, and narrowing pulse pressure are signs of compensated shock. DIF: Cognitive Level: Analyze REF: p. 779 TOP: Integrated Process: Nursing Process: Assessment MSC: Area of Client Needs: Physiologic Integrity: Physiologic Adaptation

3. The nurse is conducting discharge teaching about signs and symptoms of heart failure to parents of an infant with a repaired tetralogy of Fallot. Which signs and symptoms should the nurse include? (Select all that apply.) a. Warm flushed extremities b. Weight loss c. Decreased urinary output d. Sweating (inappropriate) e. Fatigue

c. Decreased urinary output d. Sweating (inappropriate) e. Fatigue ANS: C, D, E The signs and symptoms of heart failure include decreased urinary output, sweating, and fatigue. Other signs include pale, cool extremities, not warm and flushed, and weight gain, not weight loss. DIF: Cognitive Level: Apply REF: p. 741 TOP: Integrated Process: Teaching/Learning MSC: Area of Client Needs: Physiologic Integrity: Physiologic Adaptation

9. The nurse is admitting a toddler with the diagnosis of juvenile hypothyroidism. Which is a common clinical manifestation of this disorder? a. Insomnia b. Diarrhea c. Dry skin d. Accelerated growth

c. Dry skin ANS: C Dry skin, mental decline, and myxedematous skin changes are associated with juvenile hypothyroidism. Children with hypothyroidism are usually sleepy. Constipation is associated with hypothyroidism. Decelerated growth is common in juvenile hypothyroidism. DIF: Cognitive Level: Apply REF: p. 919 TOP: Integrated Process: Nursing Process: Assessment MSC: Area of Client Needs: Physiologic Integrity: Physiologic Adaptation

43. A child is brought to the emergency department experiencing an anaphylactic reaction to a bee sting. While an airway is being established, the nurse should prepare which medication for immediate administration? a. Diphenhydramine (Benadryl) b. Dobutamine (Dobutarex) c. Epinephrine (Adrenalin) d. Calcium chloride (calcium chloride)

c. Epinephrine (Adrenalin) ANS: C After the first priority of establishing an airway, administration of epinephrine is the drug of choice. Diphenhydramine, an antihistamine, is usually not used for severe reactions. Dobutamine and calcium chloride are not appropriate drugs for this type of reaction. DIF: Cognitive Level: Apply REF: p. 780 TOP: Integrated Process: Nursing Process: Implementation MSC: Area of Client Needs: Physiologic Integrity: Pharmacologic and Parenteral Therapies

44. What clinical manifestation is included in toxic shock syndrome? a. Severe hypertension b. Subnormal temperature c. Erythematous macular rash d. Papular rash over extremities

c. Erythematous macular rash ANS: C One of the diagnostic criteria for toxic shock syndrome is a diffuse macular erythroderma. Hypotension is one of the manifestations. Fever of 38.9° C or higher is a characteristic. Desquamation of the palms and soles of the feet occurs in about 1 to 2 weeks. DIF: Cognitive Level: Understand REF: p. 783 TOP: Integrated Process: Nursing Process: Assessment MSC: Area of Client Needs: Physiologic Integrity: Physiologic Adaptation

12. Which is a clinical manifestation of the systemic venous congestion that can occur with heart failure? a. Tachypnea b. Tachycardia c. Peripheral edema d. Pale, cool extremities

c. Peripheral edema ANS: C Peripheral edema, especially periorbital edema, is a clinical manifestation of systemic venous congestion. Tachypnea is a manifestation of pulmonary congestion. Tachycardia and pale, cool extremities are clinical manifestations of impaired myocardial function. DIF: Cognitive Level: Understand REF: p. 744 TOP: Integrated Process: Nursing Process: Assessment MSC: Area of Client Needs: Physiologic Integrity: Physiologic Adaptation

28. The home health nurse asks a child's mother many questions as part of the assessment. The mother answers many questions and then stops and says, "I don't know why you ask me all this. Who gets to know this information?" The nurse should take which action? a. Determine why the mother is so suspicious. b. Determine what the mother does not want to tell. c. Explain who will have access to the information. d. Explain that everything is confidential and that no one else will know what is said.

c. Explain who will have access to the information. ANS: C Communication with the family should not be invasive. The nurse needs to explain the importance of collecting the information, its applicability to the child's care, and who will have access to the information. The mother is not being suspicious and is not necessarily withholding important information. She has a right to understand how the information she provides will be used. The nurse will need to share, through both oral and written communication, clinically relevant information with other involved health professionals. DIF: Cognitive Level: Apply REF: p. 842 TOP: Integrated Process: Communication and Documentation MSC: Area of Client Needs: Safe and Effective Care Environment: Management of Care

25. The parents of a child who has just been diagnosed with type 1 diabetes ask about exercise. Which should the nurse explain about exercise in type 1 diabetes? a. Exercise will increase blood glucose. b. Exercise should be restricted. c. Extra snacks are needed before exercise. d. Extra insulin is required during exercise.

c. Extra snacks are needed before exercise. ANS: C Exercise lowers blood glucose levels, which can be compensated for by extra snacks. Exercise lowers blood glucose and is encouraged and not restricted, unless indicated by other health conditions. Extra insulin is contraindicated because exercise decreases blood glucose levels. DIF: Cognitive Level: Apply REF: p. 929 TOP: Integrated Process: Teaching/Learning MSC: Area of Client Needs: Physiologic Integrity: Physiologic Adaptation

A six year old is in the recovery room following an appendectomy. He is not yet fully awake, though he opens his eyes when his name is called. Which pain assessment tool would be most effective for the nurse to use at this time? a. OUCHER assessment tool. b. Wong's FACES assessment tool. c. FLAACC pain assessment tool. d. 1-10 verbal assessment scale.

c. FLAACC pain assessment tool.

6. The nurse is reviewing prenatal vitamin supplements with an expectant client. Which supplement should be included in the teaching? a. Vitamin A throughout pregnancy b. Multivitamin preparations as soon as pregnancy is suspected c. Folic acid for all women of childbearing age d. Folic acid during the first and second trimesters of pregnancy

c. Folic acid for all women of childbearing age ANS: C The widespread use of folic acid among women of childbearing age has decreased the incidence of spina bifida significantly. Vitamin A is not related to the prevention of spina bifida. Folic acid supplementation is recommended for the preconception period and during the pregnancy. Only 42% of women actually follow these guidelines. DIF: Cognitive Level: Understand REF: p. 988 TOP: Integrated Process: Nursing Process: Implementation MSC: Area of Client Needs: Physiologic Integrity: Pharmacologic and Parenteral Therapies

11. Which is best described as the inability of the heart to pump an adequate amount of blood to the systemic circulation at normal filling pressures? a. Pulmonary congestion b. Congenital heart defect c. Heart failure d. Systemic venous congestion

c. Heart failure ANS: C The definition of heart failure is the inability of the heart to pump an adequate amount of blood to the systemic circulation at normal filling pressures to meet the body's metabolic demands. Pulmonary congestion is an excessive accumulation of fluid in the lungs. Congenital heart defect is a malformation of the heart present at birth. Systemic venous congestion is an excessive accumulation of fluid in the systemic vasculature. DIF: Cognitive Level: Understand REF: p. 744 TOP: Integrated Process: Nursing Process: Assessment MSC: Area of Client Needs: Physiologic Integrity: Physiologic Adaptation

16. An adolescent will receive a bone marrow transplant (BMT). The nurse should explain that the bone marrow will be administered by which route? a. Bone grafting b. Bone marrow injection c. IV infusion d. Intra-abdominal infusion

c. IV infusion ANS: C Bone marrow from a donor is infused intravenously, and the transfused stem cells will repopulate the marrow. Because the stem cells migrate to the recipient's marrow when given intravenously, this is the method of administration. DIF: Cognitive Level: Apply REF: p. 818 TOP: Integrated Process: Teaching/Learning MSC: Area of Client Needs: Physiologic Integrity: Physiologic Adaptation

27. The nurse is caring for a child after heart surgery. What should the nurse do if evidence of cardiac tamponade is found? a. Increase analgesia b. Apply warming blankets c. Immediately report this to physician d. Encourage child to cough, turn, and breathe deeply

c. Immediately report this to physician ANS: C If evidence is noted of cardiac tamponade, which is blood or fluid in the pericardial space constricting the heart, the physician is notified immediately of this life-threatening complication. Increasing analgesia may be done before the physician drains the fluid, but the physician must be notified. Warming blankets are not indicated at this time. Encouraging the child to cough, turn, and breathe deeply should be deferred till after the evaluation by the physician. DIF: Cognitive Level: Apply REF: p. 764 TOP: Integrated Process: Nursing Process: Implementation MSC: Area of Client Needs: Physiologic Integrity: Physiologic Adaptation

1. A chest radiograph film is ordered for a child with suspected cardiac problems. Thechild's parent asks the nurse, "What will the radiograph show about the heart?" What knowledge about the x-ray should the nurse include in the response to the parents? a. Bones of chest but not the heart b. Measurement of electrical potential generated from heart muscle c. Permanent record of heart size and configuration d. Computerized image of heart vessels and tissues

c. Permanent record of heart size and configuration ANS: C A chest radiograph will provide information on the heart size and pulmonary blood-flow patterns. It will provide a baseline for future comparisons. The heart will be visible, as well as the sternum and ribs. Electrocardiography (ECG) measures the electrical potential generated from heart muscle. Echocardiography will produce a computerized image of the heart vessels and tissues by using sound waves. DIF: Cognitive Level: Understand REF: p. 738 TOP: Integrated Process: Teaching/Learning MSC: Area of Client Needs: Physiologic Integrity: Reduction of Risk Potential

19. A home care nurse is caring for an adolescent with a T1 spinal cord injury. The adolescent suddenly becomes flushed, hypertensive, and diaphoretic. Which intervention should the nurse perform first? a. Place the adolescent in a flat right side-lying position. b. Place a cool washcloth on the adolescent's forehead and continue to monitor the blood pressure. c. Implement a standing prescription to empty the bladder with a sterile in and out Foley catheter. d. Take a full set of vital signs and notify the health care provider.

c. Implement a standing prescription to empty the bladder with a sterile in and out ANS: C The adolescent is experiencing an autonomic dysreflexia episode. The paralytic nature of autonomic function is replaced by autonomic dysreflexia, especially when the lesions are above the mid-thoracic level. This autonomic phenomenon is caused by visceral distention or irritation, particularly of the bowel or bladder. Sensory impulses are triggered and travel to the cord lesion, where they are blocked, which causes activation of sympathetic reflex action with disturbed central inhibitory control. Excessive sympathetic activity is manifested by a flushing face, sweating forehead, pupillary constriction, marked hypertension, headache, and bradycardia. The precipitating stimulus may be merely a full bladder or rectum or other internal or external sensory input. It can be a catastrophic event unless the irritation is relieved. Placing a cool washcloth on the adolescent's forehead, continuing to monitor blood pressure and vital signs, and notifying the health care provider would not reverse the sympathetic reflex situation. DIF: Cognitive Level: Apply REF: p. 1000 TOP: Integrated Process: Nursing Process: Implementation MSC: Area of Client Needs: Physiologic Integrity: Reduction of Risk Potential

16. Which statement best describes b-thalassemia major (Cooley anemia)? a. All formed elements of the blood are depressed. b. Inadequate numbers of red blood cells are present. c. Increased incidence occurs in families of Mediterranean extraction. d. Increased incidence occurs in persons of West African descent.

c. Increased incidence occurs in families of Mediterranean extraction. ANS: C Individuals who live near the Mediterranean Sea and their descendants have the highest incidence of thalassemia. An overproduction of red cells occurs. Although numerous, the red cells are relatively unstable. Sickle cell disease is common in persons of West African descent. DIF: Cognitive Level: Understand REF: p. 799 TOP: Integrated Process: Nursing Process: Assessment MSC: Area of Client Needs: Physiologic Integrity: Physiologic Adaptation

11. The nurse is conducting a staff in-service on sickle cell anemia. Which describes the pathologic changes of sickle cell anemia? a. Sickle-shaped cells carry excess oxygen. b. Sickle-shaped cells decrease blood viscosity. c. Increased red blood cell destruction occurs. d. Decreased adhesion of sickle-shaped cells occurs.

c. Increased red blood cell destruction occurs. ANS: C The clinical features of sickle cell anemia are primarily the result of increased red blood cell destruction and obstruction caused by the sickle-shaped red blood cells. Sickled red cells have decreased oxygen- carrying capacity and transform into the sickle shape in conditions of low oxygen tension. When the sickle cells change shape, they increase the viscosity in the area where they are involved in the microcirculation. Increased adhesion and entanglement of cells occurs. DIF: Cognitive Level: Apply REF: p. 791 TOP: Integrated Process: Teaching/Learning MSC: Area of Client Needs: Physiologic Integrity: Physiologic Adaptation

7. Iron dextran is ordered for a young child with severe iron-deficiency anemia. What nursing considerations should be included? a. Administer with meals b. Administer between meals c. Inject deeply into a large muscle d. Massage injection site for 5 minutes after administration of drug

c. Inject deeply into a large muscle ANS: C Iron dextran is a parenteral form of iron. When administered intramuscularly, it must be injected into a large muscle. Iron dextran is for intramuscular or intravenous (IV) administration. The site should not be massaged to prevent leakage, potential irritation, and staining of the skin. DIF: Cognitive Level: Apply REF: p. 790 TOP: Integrated Process: Nursing Process: Implementation MSC: Area of Client Needs: Physiologic Integrity: Pharmacologic and Parenteral Therapies

12. The nurse is planning care for an 8-year-old child with a concussion. Which is descriptive of a concussion? a. Petechial hemorrhages cause amnesia. b. Visible bruising and tearing of cerebral tissue occur. c. It is a transient and reversible neuronal dysfunction. d. A slight lesion develops remotely from the site of trauma.

c. It is a transient and reversible neuronal dysfunction. ANS: C A concussion is a transient, reversible neuronal dysfunction with instantaneous loss of awareness and responsiveness resulting from trauma to the head. Petechial hemorrhages along the superficial aspects of the brain along the point of impact are a type of contusion, but are not necessarily associated with amnesia. A contusion is visible bruising and tearing of cerebral tissue. Contrecoup is a lesion that develops remote from the site of trauma as a result of an acceleration-deceleration injury. DIF: Cognitive Level: Understand REF: p. 883 TOP: Integrated Process: Nursing Process: Planning MSC: Area of Client Needs: Physiologic Integrity: Physiologic Adaptation

32. Which is the initial clinical manifestation of generalized seizures? a. Being confused b. Feeling frightened c. Losing consciousness d. Seeing flashing lights

c. Losing consciousness ANS: C Loss of consciousness is a frequent occurrence in generalized seizures and is the initial clinical manifestation. Being confused, feeling frightened, and seeing flashing lights are clinical manifestations of a complex partial seizure. DIF: Cognitive Level: Understand REF: p. 896 TOP: Integrated Process: Nursing Process: Assessment MSC: Area of Client Needs: Physiologic Integrity: Physiologic Adaptation

11. What is an appropriate nursing intervention when caring for an unconscious child? a. Change the child's position infrequently to minimize the chance of increased ICP b. Avoid using narcotics or sedatives to provide comfort and pain relief c. Monitor fluid intake and output carefully to avoid fluid overload and cerebral edema d. Give tepid sponge baths to reduce fever because antipyretics are contraindicated

c. Monitor fluid intake and output carefully to avoid fluid overload and cerebral edema ANS: C Often comatose patients cannot cope with the quantity of fluids that they normally tolerate. Overhydration must be avoided to prevent fatal cerebral edema. The child's position should be changed frequently to avoid complications such as pneumonia and skin breakdown. Narcotics and sedatives should be used as necessary to reduce pain and discomfort, which can increase ICP. Antipyretics are the method of choice for fever reduction. DIF: Cognitive Level: Apply REF: p. 879 TOP: Integrated Process: Nursing Process: Implementation MSC: Area of Client Needs: Physiologic Integrity: Physiologic Adaptation

27. Home care is being considered for a young child who is ventilator-dependent. Which factor is most important in deciding whether home care is appropriate? a. Level of parents' education b. Presence of two parents in the home c. Preparation and training of family d. Family's ability to assume all health care costs

c. Preparation and training of family ANS: C One of the essential elements is the family's training and preparation. The family must be able to demonstrate all aspects of care for the child. In many areas, it cannot be guaranteed that nursing care will be available on a continual basis, and the family will have to care for the child. The amount of formal education reached by the parents is not the important issue. The determinant is the family's ability to care adequately for the child in the home. At least two family members should learn and demonstrate all aspects of the child's care in the hospital, but it does not have to be two parents. Few families can assume all health care costs. Creative financial planning, including negotiating arrangements with the insurance company and/or public programs, may be required. DIF: Cognitive Level: Analyze REF: p. 842 TOP: Integrated Process: Nursing Process: Implementation MSC: Area of Client Needs: Safe and Effective Care Environment: Safety and Infection Control

11. The nurse is admitting a child with Werdnig-Hoffmann disease (spinal muscular atrophy type 1). Which signs and symptoms are associated with this disease? a. Spinal muscular atrophy b. Neural atrophy of muscles c. Progressive weakness and wasting of skeletal muscle d. Pseudohypertrophy of certain muscle groups

c. Progressive weakness and wasting of skeletal muscle ANS: C Werdnig-Hoffmann disease (spinal muscular atrophy type 1) is the most common paralytic form of floppy infant syndrome (congenital hypotonia). It is characterized by progressive weakness and wasting of skeletal muscle caused by degeneration of anterior horn cells. Kugelberg-Welander disease is a juvenile spinal muscular atrophy with a later onset. Charcot-Marie-Tooth disease is a form of progressive neural atrophy of muscles supplied by the peroneal nerves. Progressive weakness is found of the distal muscles of the arms and feet. Duchenne muscular dystrophy is characterized by muscles, especially in the calves, thighs, and upper arms, which become enlarged from fatty infiltration and feel unusually firm or woody on palpation. The term pseudohypertrophy is derived from this muscular enlargement. DIF: Cognitive Level: Understand REF: p. 991 TOP: Integrated Process: Nursing Process: Assessment MSC: Area of Client Needs: Physiologic Integrity: Physiologic Adaptation

3. The nurse is planning activity for a 4-year-old child with anemia. Which activity should the nurse plan for this child? a. Game of "hide and seek" in the children's outdoor play area b. Participation in dance activities in the playroom c. Puppet play in the child's room d. A walk down to the hospital lobby

c. Puppet play in the child's room ANS: C Because the basic pathologic process in anemia is a decrease in oxygen-carrying capacity, an important nursing responsibility is to assess the child's energy level and minimize excess demands. The child's level of tolerance for activities of daily living and play is assessed, and adjustments are made to allow as much self-care as possible without undue exertion. Puppet play in the child's room would not be overly tiring. Hide and seek, dancing, and walking to the lobby would not conserve the anemic child's energy. DIF: Cognitive Level: Apply REF: p. 789 TOP: Integrated Process: Nursing Process: Implementation MSC: Area of Client Needs: Physiologic Integrity: Physiologic Adaptation 4. The nurse is teaching parents about the importance of iron in a

35. Which action by the school nurse is important in the prevention of rheumatic fever? a. Encourage routine cholesterol screenings. b. Conduct routine blood pressure screenings. c. Refer children with sore throats for throat cultures. d. Recommend salicylates instead of acetaminophen for minor discomforts.

c. Refer children with sore throats for throat cultures. ANS: C Nurses have a role in prevention—primarily in screening school-age children for sore throats caused by group A b-hemolytic streptococci. They can achieve this by actively participating in throat culture screening or by referring children with possible streptococcal sore throats for testing. Cholesterol and blood pressure screenings do not facilitate the recognition and treatment of group A b-hemolytic streptococci. Salicylates should be avoided routinely because of the risk of Reye syndrome after viral illnesses. DIF: Cognitive Level: Apply REF: p. 768 TOP: Integrated Process: Nursing Process: Planning MSC: Area of Client Needs: Physiologic Integrity: Reduction of Risk Potential

2. A child with hypopituitarism is being started on growth hormone (GH) therapy. Nursing considerations should be based on which knowledge? a. Treatment is most successful if it is started during adolescence. b. Treatment is considered successful if children attain full stature by adulthood. c. Replacement therapy requires daily subcutaneous injections. d. Replacement therapy will be required throughout the child's lifetime.

c. Replacement therapy requires daily subcutaneous injections. ANS: C Additional support is required for children who require hormone replacement therapy, such as preparation for daily subcutaneous injections and education for self-management during the school-age years. Young children, obese children, and those who are severely GH deficient have the best response to therapy. When therapy is successful, children can attain their actual or near-final adult height at a slower rate than their peers. Replacement therapy is not needed after attaining final height. They are no longer GH deficient. DIF: Cognitive Level: Analyze REF: p. 911 TOP: Integrated Process: Nursing Process: Planning MSC: Area of Client Needs: Physiologic Integrity: Pharmacologic and Parenteral Therapies

16. A 10-year-old boy on a bicycle has been hit by a car in front of the school. The school nurse immediately assesses airway, breathing, and circulation. What is the next nursing action? a. Place on side b. Take blood pressure c. Stabilize neck and spine d. Check scalp and back for bleeding

c. Stabilize neck and spine ANS: C After determining that the child is breathing and has adequate circulation, the next action is to stabilize the neck and spine to prevent any additional trauma. The child's position should not be changed until the neck and spine are stabilized. Blood pressure is a later assessment. Less urgent, but an important assessment, is inspection of the scalp for bleeding. DIF: Cognitive Level: Apply REF: p. 886 TOP: Integrated Process: Nursing Process: Implementation MSC: Area of Client Needs: Physiologic Integrity: Physiologic Adaptation

1. The school nurse is discussing testicular self-examination with adolescent boys. Why is this important? a. Epididymitis is common during adolescence. b. Asymptomatic sexually transmitted diseases may be present. c. Testicular tumors during adolescence are generally malignant. d. Testicular tumors, although usually benign, are common during adolescence.

c. Testicular tumors during adolescence are generally malignant. ANS: C Tumors of the testes are not common, but when manifested in adolescence, they are generally malignant and demand immediate evaluation. Epididymitis is not common in adolescence. Asymptomatic sexually transmitted disease would not be evident during testicular self-examination. The focus of this examination is on testicular cancer. Testicular tumors are most commonly malignant. DIF: Cognitive Level: Apply REF: p. 842 TOP: Integrated Process: Teaching/Learning MSC: Area of Client Needs: Physiologic Integrity: Reduction of Risk Potential

3. Which is most descriptive of the pathophysiology of leukemia? a. Increased blood viscosity occurs. b. Thrombocytopenia (excessive destruction of platelets) occurs. c. Unrestricted proliferation of immature white blood cells (WBCs) occurs. d. First stage of coagulation process is abnormally stimulated.

c. Unrestricted proliferation of immature white blood cells (WBCs) occurs ANS: C Leukemia is a group of malignant disorders of the bone marrow and lymphatic system. It is defined as an unrestricted proliferation of immature WBCs in the blood-forming tissues of the body. Increased blood viscosity may occur secondary to the increased number of WBCs. Thrombocytopenia may occur secondary to the overproduction of WBCs in the bone marrow. The coagulation process is unaffected by leukemia. DIF: Cognitive Level: Understand REF: p. 826 TOP: Integrated Process: Nursing Process: Assessment MSC: Area of Client Needs: Physiologic Integrity: Physiologic Adaptation

14. A child with hypoparathyroidism is receiving vitamin D therapy. The parents should be advised to watch for which sign of vitamin D toxicity? a. Headache and seizures b. Physical restlessness and voracious appetite without weight gain c. Weakness and lassitude d. Anorexia and insomnia

c. Weakness and lassitude ANS: C Vitamin D toxicity can be a serious consequence of therapy. Parents are advised to watch for signs, including weakness, fatigue, lassitude, headache, nausea, vomiting, and diarrhea. Renal impairment is manifested through polyuria, polydipsia, and nocturia. Headaches may be a sign of vitamin D toxicity, but seizures are not. Physical restlessness and a voracious appetite with weight loss are manifestations of hyperthyroidism. Anorexia and insomnia are not characteristic of vitamin D toxicity. DIF: Cognitive Level: Apply REF: p. 921 TOP: Integrated Process: Teaching/Learning MSC: Area of Client Needs: Physiologic Integrity: Pharmacologic and Parenteral Therapies

20. Parents of a hemophiliac child ask the nurse, "Can you describe hemophilia to us?" Which response by the nurse is descriptive of most cases of hemophilia? a. Autosomal dominant disorder causing deficiency in a factor involved in the blood- clotting reaction b. X-linked recessive inherited disorder causing deficiency of platelets and prolonged bleeding c. X-linked recessive inherited disorder in which a blood-clotting factor is deficient d. Y-linked recessive inherited disorder in which the red blood cells become moon- shaped

c. X-linked recessive inherited disorder in which a blood-clotting factor is deficient ANS: C The inheritance pattern in 80% of all of the cases of hemophilia is X-linked recessive. The two most common forms of the disorder are factor VIII deficiency, hemophilia A or classic hemophilia; and factor IX deficiency, hemophilia B or Christmas disease. The inheritance pattern is X-linked recessive. The disorder involves coagulation factors, not platelets, and does not involve red cells or the Y chromosomes. DIF: Cognitive Level: Understand REF: p. 801 TOP: Integrated Process: Teaching/Learning MSC: Area of Client Needs: Physiologic Integrity: Physiologic Adaptation

34. A mother of a 5-year-old child, with complex health care needs and cared for at home, expresses anxiety about attending a kindergarten graduation exercise of a neighbor's child. The mother says, "I wish it could be my child graduating from kindergarten." The nurse recognizes that the mother is experiencing: a. abnormal anxiety. b. ineffective coping. c. chronic sorrow. d. denial.

c. chronic sorrow. ANS: C Home care nurses should be aware that parents may experience chronic sorrow as a parental stressor. Chronic sorrow as a normal grief response is associated with a living loss (the loss of a healthy child) that is cyclical in nature. This is a normal response and does not indicate abnormal anxiety, ineffective coping, or denial. DIF: Cognitive Level: Understand REF: p. 842 TOP: Integrated Process: Communication and Documentation MSC: Area of Client Needs: Psychosocial Integrity

29. When communicating with other professionals, it is important for home care nurses to: a. ask others what they want to know. b. share everything known about the family. c. restrict communication to clinically relevant information. d. recognize that confidentiality is not possible.

c. restrict communication to clinically relevant information. ANS: C The nurse will need to share, through both oral and written communication, clinically relevant information with other involved health professionals. Asking others what they want to know and sharing everything known about the family is inappropriate. Patients have a right to confidentiality. The nurse is not permitted to share information about clients, except clinically relevant information that pertains to the child's care. Confidentiality permits the disclosure of information to other health professionals on a need- to-know basis. DIF: Cognitive Level: Apply REF: p. 842 TOP: Integrated Process: Communication and Documentation MSC: Area of Client Needs: Safe and Effective Care Environment: Management of Care

11. A school-age child with diarrhea has been rehydrated. The nurse is discussing the child's diet with the family. Which statement by the parent would indicate a correct understanding of the teaching? a. "I will keep my child on a clear liquid diet for the next 24 hours." b. "I should encourage my child to drink carbonated drinks but avoid food for the next 24 hours." c. "I will offer my child bananas, rice, applesauce, and toast for the next 48 hours." d. "I should have my child eat a normal diet with easily digested foods for the next 48 hours."

d. "I should have my child eat a normal diet with easily digested foods for the next 48 hours." ANS: D Easily digested foods such as cereals, cooked vegetables, and meats should be provided for the child. Early reintroduction of nutrients is desirable. Continued feeding or reintroduction of a regular diet has no adverse effects and actually lessens the severity and duration of the illness. Clear liquids and carbonated drinks have high carbohydrate content and few electrolytes. Caffeinated beverages should be avoided because caffeine is a mild diuretic. The BRAT diet has little nutritional value and is high in carbohydrates.

51. A family wants to begin oral feeding of their 4-year-old son, who is ventilator-dependent and currently tube-fed. They ask the home health nurse to feed him the baby food orally. The nurse recognizes a high risk of aspiration and an already compromised respiratory status. What is the most appropriate nursing action? a. Refuse to feed him orally because the risk is too high. b. Explain the risks involved, and then let the family decide what should be done. c. Feed him orally because the family has the right to make this decision for their child. d. Acknowledge their request, explain the risks, and explore with the family the available options.

d. Acknowledge their request, explain the risks, and explore with the family the available options. ANS: D Parents want to be included in the decision making for their child's care. The nurse should discuss the request with the family to ensure this is the issue of concern, and then they can explore potential options together. Merely refusing to feed the child orally does not determine why the parents wish the oral feedings to begin and does not involve them in the problem solving. The decision to begin or not change feedings should be a collaborative one, made in consultation with the family, nurse, and appropriate member of the health care team. DIF: Cognitive Level: Analyze REF: p. 735 TOP: Integrated Process: Teaching/Learning MSC: Area of Client Needs: Safe and Effective Care Environment: Management of Care

4. A nurse is admitting an infant with dehydration caused from water loss in excess of electrolyte loss. Which type of dehydration is this infant experiencing? a. Isotonic b. Isosmotic c. Hypotonic d. Hypertonic

d. Hypertonic ANS: D Hypertonic dehydration results from water loss in excess of electrolyte loss. This is the most dangerous type of dehydration. It is caused by feeding children fluids with high amounts of solute. Isotonic dehydration occurs in conditions in which electrolyte and water deficits are present in balanced proportion and is another term for isomotic dehydration. Hypotonic dehydration occurs when the electrolyte deficit exceeds the water deficit, leaving the serum hypotonic. DIF: Cognitive Level: Understand REF: p. 694 TOP: Integrated Process: Nursing Process: Assessment MSC: Area of Client Needs: Physiologic Integrity: Reduction of Risk Potential

10. Which is a common side effect of short-term corticosteroid therapy? a. Fever b. Hypertension c. Weight loss d. Increased appetite

d. Increased appetite ANS: D Side effects of short-term corticosteroid therapy include an increased appetite. Fever is not a side effect of therapy. It may be an indication of infection. Hypertension is not usually associated with initial corticosteroid therapy. Weight gain, not weight loss, is associated with corticosteroid therapy. PTS: 1 DIF: Cognitive Level: Understand REF: 860 TOP: Integrated Process: Nursing Process: Implementation MSC: Area of Client Needs: Physiologic Integrity: Pharmacologic and Parenteral Therapy

44. What is invagination of one segment of bowel within another called? a. Atresia b. Stenosis c. Herniation d. Intussusception

d. Intussusception ANS: D Intussusception occurs when a proximal section of the bowel telescopes into a more distal segment, pulling the mesentery with it. The mesentery is compressed and angled, resulting in lymphatic and venous obstruction. Invagination of one segment of bowel within another is the definition of intussusception, not atresia, stenosis, or herniation. DIF: Cognitive Level: Understand REF: p. 728 TOP: Integrated Process: Nursing Process: Assessment MSC: Area of Client Needs: Physiologic Integrity: Reduction of Risk Potential

43. What are the results of excessive vomiting in an infant with pyloric stenosis? a. Hyperchloremia b. Hypernatremia c. Metabolic acidosis d. Metabolic alkalosis

d. Metabolic alkalosis ANS: D Infants with excessive vomiting are prone to metabolic alkalosis from the loss of hydrogen ions. Chloride ions and sodium are lost with vomiting. Metabolic alkalosis, not acidosis, is likely. DIF: Cognitive Level: Understand REF: p. 728 TOP: Integrated Process: Nursing Process: Assessment MSC: Area of Client Needs: Physiologic Integrity: Reduction of Risk Potential

9. A child is admitted with bacterial gastroenteritis. Which lab results of a stool specimen confirm this diagnosis? a. Eosinophils b. Occult blood c. pH less than 6 d. Neutrophils and red blood cells

d. Neutrophils and red blood cells ANS: D Neutrophils and red blood cells in stool indicate bacterial gastroenteritis. Protein intolerance and parasitic infections are suspected in the presence of eosinophils. Occult blood may indicate pathogens such as Shigella, Campylobacter, or hemorrhagic Escherichia coli strains. A pH of less than 6 may indicate carbohydrate malabsorption or secondary lactase insufficiency. DIF: Cognitive Level: Understand REF: p. 700 TOP: Integrated Process: Nursing Process: Evaluation MSC: Area of Client Needs: Physiologic Integrity: Reduction of Risk Potential

31. Which is an advantage of peritoneal dialysis? a. Treatments are done in hospitals. b. Protein loss is less extensive. c. Dietary limitations are not necessary. d. Parents and older children can perform treatments.

d. Parents and older children can perform treatments. ANS: D Peritoneal dialysis is the preferred form of dialysis for parents, infants, and children who wish to remain independent. Parents and older children can perform the treatments themselves. Treatments can be done at home. Protein loss is not significantly different. The dietary limitations are necessary, but they are not as stringent as those for hemodialysis. PTS: 1 DIF: Cognitive Level: Analyze REF: 923 TOP: Integrated Process: Nursing Process: Planning MSC: Area of Client Needs: Physiologic Integrity: Physiologic Adaptation

18. The nurse is caring for an intubated infant with botulism in the pediatric intensive care unit. Which health care provider prescription should the nurse clarify with the health care provider before implementing? a. Administer 250 mg botulism immune globulin intravenously (BIG-IV) one time. b. Provide total parenteral nutrition (TPN) at 25 ml/hr intravenously. c. Titrate oxygen to keep pulse oximetry saturations greater than 92. d. Administer gentamicin sulfate (Garamycin) 10 mg per intravenous piggyback every 12 hours.

d. Administer gentamicin sulfate (Garamycin) 10 mg per intravenous piggyback every ANS: D The nurse should clarify the administration of an aminoglycoside antibiotic. Antibiotic therapy is not part of the management of infant botulism because the botulinum toxin is an intracellular molecule, and antibiotics would not be effective; aminoglycosides in particular should not be administered because they may potentiate the blocking effects of the neurotoxin. Treatment consists of immediate administration of botulism immune globulin intravenously (BIG-IV) without delaying for laboratory diagnosis. Early administration of BIG-IV neutralizes the toxin and stops the progression of the disease. The human- derived botulism antitoxin (BIG-IV) has been evaluated and is now available nationwide for use only in infant botulism. Approximately 50% of affected infants require intubation and mechanical ventilation; therefore, respiratory support is crucial, as is nutritional support, because these infants are unable to feed. DIF: Cognitive Level: Analyze REF: p. 999 TOP: Integrated Process: Nursing Process: Evaluation MSC: Area of Client Needs: Physiologic Integrity: Pharmacologic and Parenteral Therapies

21. The nurse is conducting a staff in-service on childhood blood disorders. Which describes the pathology of idiopathic thrombocytopenic purpura? a. Bone marrow failure in which all elements are suppressed b. Deficiency in the production rate of globin chains c. Diffuse fibrin deposition in the microvasculature d. An excessive destruction of platelets

d. An excessive destruction of platelets ANS: D Idiopathic thrombocytopenic purpura is an acquired hemorrhagic disorder characterized by an excessive destruction of platelets, discolorations caused by petechiae beneath the skin, and a normal bone marrow. Aplastic anemia refers to a bone marrow-failure condition in which the formed elements of the blood are simultaneously depressed. Thalassemia major is a group of blood disorders characterized by deficiency in the production rate of specific hemoglobin globin chains. Disseminated intravascular coagulation is characterized by diffuse fibrin deposition in the microvasculature, consumption of coagulation factors, and endogenous generation of thrombin and plasma. DIF: Cognitive Level: Understand REF: p. 804 TOP: Integrated Process: Teaching/Learning MSC: Area of Client Needs: Physiologic Integrity: Physiologic Adaptation

40. Which type of shock is characterized by a hypersensitivity reaction causing massive vasodilation and capillary leaks, which may occur with drug or latex allergy? a. Neurogenic b. Cardiogenic c. Hypovolemic d. Anaphylactic

d. Anaphylactic ANS: D Anaphylactic shock results from extreme allergy or hypersensitivity to a foreign substance. Neurogenic shock results from loss of neuronal control, such as the interruption of neuronal transmission that occurs from a spinal cord injury. Cardiogenic shock is decreased cardiac output. Hypovolemic shock is a reduction in the size of the vascular compartment, decreasing blood pressure, and low central venous pressure. DIF: Cognitive Level: Understand REF: p. 779 TOP: Integrated Process: Nursing Process: Assessment MSC: Area of Client Needs: Physiologic Integrity: Physiologic Adaptation

5. The nurse is caring for a school-age child who has had a cardiac catheterization. The child tells the nurse that the bandage is "too wet." The nurse finds the bandage and bed soaked with blood. What is the priority nursing action? a. Notify physician b. Apply new bandage with more pressure c. Place the child in Trendelenburg position d. Apply direct pressure above catheterization site

d. Apply direct pressure above catheterization site ANS: D If bleeding occurs, direct continuous pressure is applied 2.5 cm (1 inch) above the percutaneous skin site to localize pressure over the vessel puncture. Notifying a physician and applying a new bandage can be done after pressure is applied. The nurse can have someone else notify the physician while the pressure is being maintained. It is not a helpful intervention to place the girl in the Trendelenburg position. It would increase the drainage from the lower extremities.

26. What is beneficial in reducing the risk of Reye syndrome? a. Immunization against the disease b. Medical attention for all head injuries c. Prompt treatment of bacterial meningitis d. Avoidance of aspirin to treat fever associated with influenza

d. Avoidance of aspirin to treat fever associated with influenza ANS: D Although the etiology of Reye syndrome is obscure, most cases follow a common viral illness, either varicella or influenza. A potential association exists between aspirin therapy and the development of Reye syndrome, so use of aspirin is avoided. No immunization currently exists for Reye syndrome. Reye syndrome is not correlated with head injuries or bacterial meningitis. DIF: Cognitive Level: Understand REF: p. 895 TOP: Integrated Process: Nursing Process: Planning MSC: Area of Client Needs: Physiologic Integrity: Reduction of Risk Potential

32. Which is a common, serious complication of rheumatic fever? a. Seizures b. Cardiac arrhythmias c. Pulmonary hypertension d. Cardiac valve damage

d. Cardiac valve damage ANS: D Cardiac valve damage is the most significant complication of rheumatic fever. Seizures, cardiac arrhythmias, and pulmonary hypertension are not common complications of rheumatic fever. DIF: Cognitive Level: Understand REF: p. 767 TOP: Integrated Process: Nursing Process: Assessment MSC: Area of Client Needs: Physiologic Integrity: Reduction of Risk Potential

6. A child with leukemia is receiving triple intrathecal chemotherapy consisting of methotrexate, cytarabine, and hydrocortisone. What will the triple intrathecal chemotherapy prevent? a. Infection b. Brain tumor c. Drug side effects d. Central nervous system (CNS) disease

d. Central nervous system (CNS) disease ANS: D For certain children, CNS prophylactic therapy is indicated. This drug regimen is used to prevent CNS leukemia and will not prevent infection or drug side effects. If the child has a brain tumor in addition to leukemia, additional therapy would be indicated. DIF: Cognitive Level: Analyze REF: p. 831 TOP: Integrated Process: Nursing Process: Planning MSC: Area of Client Needs: Physiologic Integrity: Pharmacologic and Parenteral Therapies

29. An 8-year-old girl is receiving a blood transfusion when the nurse notes that she has developed precordial pain, dyspnea, distended neck veins, slight cyanosis, and a dry cough. Of what are these manifestations most suggestive? a. Air emboli b. Allergic reaction c. Hemolytic reaction d. Circulatory overload

d. Circulatory overload ANS: D The signs of circulatory overload include distended neck veins, hypertension, crackles, dry cough, cyanosis, and precordial pain. Signs of air embolism are sudden difficulty breathing, sharp pain in the chest, and apprehension. Allergic reactions are manifested by urticaria, pruritus, flushing, asthmatic wheezing, and laryngeal edema. Hemolytic reactions are characterized by chills, shaking, fever, pain at infusion site, nausea, vomiting, tightness in chest, flank pain, red or black urine, and progressive signs of shock and renal failure. DIF: Cognitive Level: Apply REF: p. 811 TOP: Integrated Process: Nursing Process: Assessment MSC: Area of Client Needs: Physiologic Integrity: Physiologic Adaptation

26. A child eats some sugar cubes after experiencing symptoms of hypoglycemia. What should follow this rapid-releasing sugar? a. Fat b. Fruit juice c. Several glasses of water d. Complex carbohydrate and protein

d. Complex carbohydrate and protein ANS: D Symptoms of hypoglycemia are treated with a rapid-releasing sugar source followed by a complex carbohydrate and protein. Fat, fruit juice, and several glasses of water do not provide the child with complex carbohydrate and protein necessary to stabilize the blood glucose. DIF: Cognitive Level: Apply REF: p. 930 TOP: Integrated Process: Nursing Process: Planning MSC: Area of Client Needs: Physiologic Integrity: Physiologic Adaptation

41. Which clinical manifestation should the nurse expect to see as shock progresses in a child and becomes decompensated shock? a. Thirst b. Irritability c. Apprehension d. Confusion and somnolence

d. Confusion and somnolence ANS: D Confusion and somnolence are beginning signs of decompensated shock. Thirst, irritability, and apprehension are signs of compensated shock. DIF: Cognitive Level: Understand REF: p. 779 TOP: Integrated Process: Nursing Process: Assessment MSC: Area of Client Needs: Physiologic Integrity: Physiologic Adaptation

8. Which is the most effective pain-management approach for a child who is having a bone marrow aspiration? a. Relaxation techniques b. Administration of an opioid c. EMLA cream applied over site d. Conscious or unconscious sedation

d. Conscious or unconscious sedation ANS: D Effective pharmacologic and nonpharmacologic measures should be used to minimize pain associated with procedures. For bone marrow aspiration, conscious or unconscious sedation should be used. Relaxation, opioids, and EMLA can be used to augment the conscious or unconscious sedation. DIF: Cognitive Level: Apply REF: p. 824 TOP: Integrated Process: Nursing Process: Planning MSC: Area of Client Needs: Physiologic Integrity: Pharmacologic and Parenteral Therapies

15. Which is a common clinical manifestation of Hodgkin disease? a. Petechiae b. Bone and joint pain c. Painful, enlarged lymph nodes d. Enlarged, firm, nontender lymph nodes

d. Enlarged, firm, nontender lymph nodes ANS: D Asymptomatic, enlarged, cervical or supraclavicular lymphadenopathy is the most common presentation of Hodgkin disease. Petechiae are usually associated with leukemia. Bone and joint pain are not likely in Hodgkin disease. The enlarged nodes are rarely painful. DIF: Cognitive Level: Understand REF: p. 829 TOP: Integrated Process: Nursing Process: Assessment MSC: Area of Client Needs: Physiologic Integrity: Physiologic Adaptation

14. The nurse is preparing a child for possible alopecia from chemotherapy. Which should be included? a. Explain to child that hair usually regrows in 1 year. b. Advise child to expose head to sunlight to minimize alopecia. c. Explain to child that wearing a hat or scarf is preferable to wearing a wig. d. Explain to child that when hair regrows, it may have a slightly different color or texture.

d. Explain to child that when hair regrows, it may have a slightly different color or ANS: D Alopecia is a side effect of certain chemotherapeutic agents. When the hair regrows, it may be a different color or texture. The hair usually grows back within 3 to 6 months after cessation of treatment. The head should be protected from sunlight to avoid sunburn. Children should choose the head covering they prefer. DIF: Cognitive Level: Apply REF: p. 819 TOP: Integrated Process: Teaching/Learning MSC: Area of Client Needs: Physiologic Integrity: Physiologic Adaptation

15. The nurse is conducting reflex testing on infants at a well-child clinic. Which reflex finding should be reported as abnormal and considered as a possible sign of cerebral palsy? a. Tonic neck reflex at 5 months of age b. Absent Moro reflex at 8 months of age c. Moro reflex at 3 months of age d. Extensor reflex at 7 months of age

d. Extensor reflex at 7 months of age ANS: D Establishing a diagnosis of cerebral palsy (CP) may be confirmed with the persistence of primitive reflexes: (1) either the asymmetric tonic neck reflex or persistent Moro reflex (beyond 4 months of age) and (2) the crossed extensor reflex. The tonic neck reflex normally disappears between 4 and 6 months of age. The crossed extensor reflex, which normally disappears by 4 months, is elicited by applying a noxious stimulus to the sole of one foot with the knee extended. Normally, the contralateral foot responds with extensor, abduction, and then adduction movements. The possibility of CP is suggested if these reflexes occur after 4 months. DIF: Cognitive Level: Apply REF: p. 979 TOP: Integrated Process: Nursing Process: Implementation MSC: Area of Client Needs: Physiologic Integrity: Reduction of Risk Potential

20. Which is considered a cardinal sign of diabetes mellitus? a. Nausea b. Seizures c. Impaired vision d. Frequent urination

d. Frequent urination ANS: D Hallmarks of diabetes mellitus are glycosuria, polyuria, and polydipsia. Nausea and seizures are not clinical manifestations of diabetes mellitus. Impaired vision is a long-term complication of the disease. DIF: Cognitive Level: Understand REF: p. 936 TOP: Integrated Process: Nursing Process: Assessment MSC: Area of Client Needs: Physiologic Integrity: Physiologic Adaptation

36. When discussing hyperlipidemia with a group of adolescents, which high level labs should the nurse explain can prevent cardiovascular disease? a. Cholesterol b. Triglycerides c. Low-density lipoproteins (LDLs) d. High-density lipoproteins (HDLs)

d. High-density lipoproteins (HDLs) ANS: D HDLs contain very low concentrations of triglycerides, relatively little cholesterol, and high levels of proteins. It is thought that HDLs protect against cardiovascular disease. Cholesterol, triglycerides, and LDLs are not protective against cardiovascular disease.

8. The nurse is recommending how to prevent iron-deficiency anemia in a healthy, term, breastfed infant. Which should be suggested? a. Iron (ferrous sulfate) drops after age 1 month b. Iron-fortified commercial formula by age 4 to 6 months c. Iron-fortified infant cereal by age 2 months d. Iron-fortified infant cereal by age 4 to 6 months

d. Iron-fortified infant cereal by age 4 to 6 months ANS: D Breast milk supplies inadequate iron for growth and development after age 5 months. Supplementation is necessary at this time. The mother can supplement the breastfeeding with iron-fortified infant cereal. Iron supplementation or the introduction of solid foods in a breastfed baby is not indicated. Providing iron- fortified commercial formula by age 4 to 6 months should be done only if the mother is choosing to discontinue breastfeeding. DIF: Cognitive Level: Apply REF: p. 789 TOP: Integrated Process: Teaching/Learning MSC: Area of Client Needs: Physiologic Integrity: Basic Care and Comfort

1. Which are clinical manifestations of increased intracranial pressure (ICP) in infants? (Select all that apply.) a. Low-pitched cry b. Sunken fontanel c. Diplopia and blurred vision d. Irritability e. Distended scalp veins f. Increased blood pressure

d. Irritability e. Distended scalp veins ANS: D, E Diplopia and blurred vision, irritability, and distended scalp veins are signs of increased ICP in infants. Diplopia and blurred vision are indicative of elevated ICP in children. A high-pitched cry and a tense or bulging fontanel are characteristics of increased ICP. Increased blood pressure, common in adults, is rarely seen in children. DIF: Cognitive Level: Understand REF: p. 872 TOP: Integrated Process: Nursing Process: Assessment MSC: Area of Client Needs: Physiologic Integrity: Physiologic Adaptation

15. When should the nurse recommend medical attention for a child with a slight head injury? a. Experiences sleepiness b. Vomits c. Has a headache d. Is confused or has abnormal behavior

d. Is confused or has abnormal behavior ANS: D Medical attention should be sought if the child exhibits confusion or abnormal behavior, loses consciousness, has amnesia, has fluid leaking from the nose or ears, complains of blurred vision, or has an unsteady gait. Sleepiness alone does not require evaluation. If the child is difficult to arouse from sleep, medical attention should be obtained. Vomiting more than three times requires medical attention. Severe or worsening headache or one that interferes with sleep should be evaluated. DIF: Cognitive Level: Apply REF: p. 883 TOP: Integrated Process: Teaching/Learning MSC: Area of Client Needs: Physiologic Integrity: Physiologic Adaptation

24. The parent of a child with diabetes mellitus asks the nurse when urine testing will be necessary. The nurse should explain that urine testing is necessary for which? a. Glucose is needed before administration of insulin. b. Glucose is needed four times a day. c. Glycosylated hemoglobin is required. d. Ketonuria is suspected.

d. Ketonuria is suspected. ANS: D Urine testing is still performed to detect evidence of ketonuria. Urine testing for glucose is no longer indicated because of the poor correlation between blood glucose levels and glycosuria. Glycosylated hemoglobin analysis is performed on a blood sample. DIF: Cognitive Level: Apply REF: p. 929 TOP: Integrated Process: Teaching/Learning MSC: Area of Client Needs: Physiologic Integrity: Physiologic Adaptation

13. Which clinical manifestation may occur in the child who is receiving too much methimazole (Tapazole) for the treatment of hyperthyroidism (Graves disease)? a. Seizures b. Enlargement of all lymph glands c. Pancreatitis or cholecystitis d. Lethargy and somnolence

d. Lethargy and somnolence ANS: D Parents should be aware of the signs of hypothyroidism that can occur from overdosage of the drug. The most common manifestations are lethargy and somnolence. Seizures and pancreatitis are not associated with the administration of Tapazole. Enlargement of the salivary and cervical lymph glands occurs. DIF: Cognitive Level: Apply REF: p. 920 TOP: Integrated Process: Nursing Process: Implementation MSC: Area of Client Needs: Physiologic Integrity: Pharmacologic and Parenteral Therapies

20. The nurse is assessing a child who was just admitted to the hospital for observation after a head injury. Which is the most essential part of the nursing assessment to detect early signs of a worsening condition? a. Posturing b. Vital signs c. Focal neurologic signs d. Level of consciousness

d. Level of consciousness ANS: D The most important nursing observation is assessment of the child's level of consciousness. Alterations in consciousness appear earlier in the progression of an injury than do alterations of vital signs or focal neurologic signs. Neurologic posturing is indicative of neurologic damage. Vital signs and focal neurologic signs are later signs of progression when compared with level-of-consciousness changes. DIF: Cognitive Level: Analyze REF: p. 886 TOP: Integrated Process: Nursing Process: Assessment MSC: Area of Client Needs: Physiologic Integrity: Physiologic Adaptation

5. A child will start treatment for precocious puberty. The nurse recognizes that this will involve the injection of which synthetic medication? a. Thyrotropin b. Gonadotropins c. Somatotropic hormone d. Luteinizing hormone-releasing hormone

d. Luteinizing hormone-releasing hormone ANS: D Precocious puberty of central origin is treated with monthly subcutaneous injections of luteinizing hormone-releasing hormone. Thyrotropin, gonadotropins, and somatotropic hormone are not the appropriate therapies for precocious puberty. DIF: Cognitive Level: Understand REF: p. 912 TOP: Integrated Process: Nursing Process: Implementation MSC: Area of Client Needs: Physiologic Integrity: Pharmacologic and Parenteral Therapies

9. Which immunization should not be given to a child receiving chemotherapy for cancer? a. Tetanus vaccine b. Inactivated poliovirus vaccine c. Diphtheria, pertussis, tetanus (DPT) d. Measles, rubella, mumps

d. Measles, rubella, mumps ANS: D The vaccine used for measles, mumps, and rubella is a live virus and can result in an overwhelming infection. Tetanus vaccine, inactivated poliovirus vaccine, and diphtheria, pertussis, tetanus (DPT) are not live virus vaccines. DIF: Cognitive Level: Apply REF: p. 825 TOP: Integrated Process: Nursing Process: Assessment MSC: Area of Client Needs: Physiologic Integrity: Pharmacologic and Parenteral Therapies

28. Several complications can occur when a child receives a blood transfusion. Which is an immediate sign or symptom of an air embolus? a. Chills and shaking b. Nausea and vomiting c. Irregular heart rate d. Sudden difficulty in breathing

d. Sudden difficulty in breathing ANS: D Signs of air embolism are sudden difficulty breathing, sharp pain in the chest, and apprehension. Air emboli should be avoided by carefully flushing all tubing of air before connecting to patient. Chills, shaking, nausea, and vomiting are associated with hemolytic reactions. Irregular heart rate is associated with electrolyte disturbances and hypothermia. DIF: Cognitive Level: Understand REF: p. 810 TOP: Integrated Process: Nursing Process: Assessment MSC: Area of Client Needs: Physiologic Integrity: Physiologic Adaptation

12. Which clinical manifestation should the nurse expect when a child with sickle cell anemia experiences an acute vasoocclusive crisis? a. Circulatory collapse b. Cardiomegaly, systolic murmurs c. Hepatomegaly, intrahepatic cholestasis d. Painful swelling of hands and feet; painful joints

d. Painful swelling of hands and feet; painful joints ANS: D A vasoocclusive crisis is characterized by severe pain in the area of involvement. If in the extremities, painful swelling of the hands and feet is seen; if in the abdomen, severe pain resembles that of acute surgical abdomen; and if in the head, stroke and visual disturbances occur. Circulatory collapse results from sequestration crises. Cardiomegaly, systolic murmurs, hepatomegaly, and intrahepatic cholestasis result from chronic vasoocclusive phenomena. DIF: Cognitive Level: Understand REF: p. 791 TOP: Integrated Process: Nursing Process: Assessment MSC: Area of Client Needs: Physiologic Integrity: Physiologic Adaptation

18. The parents of a young child with heart failure tell the nurse that they are "nervous" about giving digoxin (Lanoxin). The nurse's response should be based on which statement? a. It is a safe, frequently used drug. b. It is difficult to either overmedicate or undermedicate with digoxin. c. Parents lack the expertise necessary to administer digoxin. d. Parents must learn specific, important guidelines for administration of digoxin.

d. Parents must learn specific, important guidelines for administration of digoxin. ANS: D Digoxin has a narrow therapeutic range. The margin of safety between therapeutic, toxic, and lethal doses is very small. Specific guidelines are available for parents to learn how to administer the drug safely and to monitor for side effects. Digoxin is a frequently used drug, but it has a narrow therapeutic range. Small amounts of the liquid are given to infants, making it easy to overmedicate or undermedicate. Parents may lack the necessary expertise to administer the drug at first, but with discharge preparation, they should be prepared to administer the drug safely. DIF: Cognitive Level: Apply REF: p. 754 TOP: Integrated Process: Teaching/Learning MSC: Area of Client Needs: Physiologic Integrity: Pharmacologic and Parenteral Therapies

5. The nurse is caring for a child with severe head trauma after a car accident. Which is an ominous sign that often precedes death? a. Papilledema b. Delirium c. Doll's head maneuver d. Periodic and irregular breathing

d. Periodic and irregular breathing ANS: D Periodic or irregular breathing is an ominous sign of brainstem (especially medullary) dysfunction that often precedes complete apnea. Papilledema is edema and inflammation of optic nerve. It is commonly a sign of increased intracranial pressure Delirium is a state of mental confusion and excitement marked by disorientation for time and place. The doll's head maneuver is a test for brainstem or oculomotor nerve dysfunction. DIF: Cognitive Level: Understand REF: p. 880 TOP: Integrated Process: Nursing Process: Assessment MSC: Area of Client Needs: Physiologic Integrity: Physiologic Adaptation

7. The nurse is caring for a preschool child with suspected diabetes insipidus. Which clinical manifestation should the nurse expect to observe? a. Oliguria b. Glycosuria c. Nausea and vomiting d. Polyuria and polydipsia

d. Polyuria and polydipsia ANS: D Excessive urination accompanied by insatiable thirst is the primary clinical manifestation of diabetes. These symptoms may be so severe that the child does little other than drink and urinate. Oliguria is decreased urine production and is not associated with diabetes insipidus. Glycosuria is associated with diabetes mellitus. Nausea and vomiting are associated with inappropriate antidiuretic hormone (ADH) secretion. DIF: Cognitive Level: Apply REF: p. 917 TOP: Integrated Process: Nursing Process: Assessment MSC: Area of Client Needs: Physiologic Integrity: Physiologic Adaptation

22. Type 1 diabetes mellitus is suspected in an adolescent. Which clinical manifestation may be present? a. Moist skin b. Weight gain c. Fluid overload d. Poor wound healing

d. Poor wound healing ANS: D Poor wound healing may be present in an individual with type 1 diabetes mellitus. Dry skin, weight loss, and dehydration are clinical manifestations of type 1 diabetes mellitus. DIF: Cognitive Level: Understand REF: p. 928 TOP: Integrated Process: Nursing Process: Assessment MSC: Area of Client Needs: Physiologic Integrity: Physiologic Adaptation

What does the surgical closure of the ductus arteriosus do? a. Stop the loss of unoxygenated blood to the systemic circulation b. Decrease the edema in legs and feet c. Increase the oxygenation of blood d. Prevent the return of oxygenated blood to the lungs ANS: D

d. Prevent the return of oxygenated blood to the lungs ANS: D The ductus arteriosus allows blood to flow from the higher-pressure aorta to the lower-pressure pulmonary artery, causing a right-to-left shunt. If this is surgically closed, no additional oxygenated blood (from the aorta) will return to the lungs through the pulmonary artery. The aorta carries oxygenated blood to the systemic circulation. Because of the higher pressure in the aorta, blood is shunted into the pulmonary artery and the pulmonary circulation. Edema in the legs and feet is usually a sign of heart failure. This repair would not directly affect the edema. Increasing the oxygenation of blood would not interfere with the return of oxygenated blood to the lungs.

25. Seventy-two hours after cardiac surgery, a young child has a temperature of 101° F. Which action should the nurse take? a. Keep child warm with blankets. b. Apply a hypothermia blanket. c. Record temperature on nurses' notes. d. Report findings to physician.

d. Report findings to physician. ANS: D In the first 24 to 48 hours after surgery, the body temperature may increase to 37.7° C (100° F) as part of the inflammatory response to tissue trauma. If the temperature is higher or continues after this period, it is most likely a sign of an infection and immediate investigation is indicated. Blankets should be removed from the child to keep the temperature from increasing. Hypothermia blanket is not indicated for this level of temperature. The temperature should be recorded, but the physician must be notified for evaluation. Suctioning should be done only as indicated, not on a routine basis. The child should be suctioned for no more than 5 seconds at one time. Symptoms of respiratory distress are avoided by using appropriate technique. DIF: Cognitive Level: Apply REF: p. 763 TOP: Integrated Process: Nursing Process: Implementation MSC: Area of Client Needs: Physiologic Integrity: Physiologic Adaptation

27. The nurse is caring for an 8-year-old child with type 1 diabetes. The nurse should teach the child to monitor for which manifestation of hypoglycemia? a. Lethargy b. Thirst c. Nausea and vomiting d. Shaky feeling and dizziness

d. Shaky feeling and dizziness ANS: D Some of the clinical manifestations of hypoglycemia include shaky feelings; dizziness; difficulty concentrating, speaking, focusing, or coordinating; sweating; and pallor. Lethargy, thirst, and nausea and vomiting are manifestations of hyperglycemia. DIF: Cognitive Level: Apply REF: p. 931 TOP: Integrated Process: Teaching/Learning MSC: Area of Client Needs: Physiologic Integrity: Physiologic Adaptation

14. What should be administered to a child with tetanus? a. Nonsteroidal antiinflammatory drugs (NSAIDs) to reduce inflammation. b. Muscle stimulants to counteract muscle weakness. c. Bronchodilators to prevent respiratory complications. d. Tetanus immunoglobulin therapy.

d. Tetanus immunoglobulin therapy. ANS: D Tetanus immunoglobulin therapy, to neutralize toxins, is the most specific therapy for tetanus. Tetanus toxin acts at the myoneural junction to produce muscular stiffness and lowers the threshold for reflex excitability. NSAIDs are not routinely used. Sedatives or muscle relaxants are used to help reduce titanic spasm and prevent seizures. Respiratory status is carefully evaluated for any signs of distress because muscle relaxants, opioids, and sedatives that may be prescribed may cause respiratory depression. Bronchodilators would not be used unless specifically indicated. DIF: Cognitive Level: Understand REF: p. 997 TOP: Integrated Process: Nursing Process: Implementation MSC: Area of Client Needs: Physiologic Integrity: Pharmacologic and Parenteral Therapies

38. When caring for the child with Kawasaki disease, the nurse should know which information? a. A child's fever is usually responsive to antibiotics within 48 hours. b. The principal area of involvement is the joints. c. Aspirin is contraindicated. d. Therapeutic management includes administration of gamma globulin and aspirin.

d. Therapeutic management includes administration of gamma globulin and aspirin. ANS: D High-dose IV gamma globulin and aspirin therapy is indicated to reduce the incidence of coronary artery abnormalities when given within the first 10 days of the illness. The fever of Kawasaki disease is unresponsive to antibiotics and antipyretics. Mucous membranes, conjunctiva, changes in the extremities, and cardiac involvement are seen. Aspirin is part of the therapy. DIF: Cognitive Level: Apply REF: p. 776 TOP: Integrated Process: Nursing Process: Implementation MSC: Area of Client Needs: Physiologic Integrity: Physiologic Adaptation

24. The mother of a 1-month-old infant tells the nurse she worries that her baby will get meningitis like her oldest son did when he was an infant. The nurse should base her response on which statement? a. Meningitis rarely occurs during infancy. b. Often a genetic predisposition to meningitis is found. c. Vaccination to prevent all types of meningitis is now available. d. Vaccination to prevent Haemophilus influenzae type B meningitis has decreased the frequency of this disease in children.

d. Vaccination to prevent Haemophilus influenzae type B meningitis has decreased the frequency of this disease in children. ANS: D H. influenzae type B meningitis has been virtually eradicated in areas of the world where the vaccine is administered routinely. Bacterial meningitis remains a serious illness in children. It is significant because of the residual damage caused by undiagnosed and untreated or inadequately treated cases. The leading causes of neonatal meningitis are the group B streptococci and Escherichia coli organisms. Meningitis is an extension of a variety of bacterial infections. No genetic predisposition exists. Vaccinations are not available for all of the potential causative organisms. DIF: Cognitive Level: Apply REF: p. 890 TOP: Integrated Process: Teaching/Learning MSC: Area of Client Needs: Physiologic Integrity: Reduction of Risk Potential

3. The nurse is preparing to admit a newborn with myelomeningocele to the neonatal intensive care nursery. Which describes this newborn's defect? a. Fissure in the spinal column that leaves the meninges and the spinal cord exposed b. Herniation of the brain and meninges through a defect in the skull c. Hernial protrusion of a saclike cyst of meninges with spinal fluid but no neural elements d. Visible defect with an external saclike protrusion containing meninges, spinal fluid, and nerves

d. Visible defect with an external saclike protrusion containing meninges, spinal fluid, and nerves ANS: D A myelomeningocele is a visible defect with an external saclike protrusion, containing meninges, spinal fluid, and nerves. Rachischisis is a fissure in the spinal column that leaves the meninges and the spinal cord exposed. Encephalocele is a herniation of brain and meninges through a defect in the skull, producing a fluid-filled sac. Meningocele is a hernial protrusion of a saclike cyst of meninges with spinal fluid, but no neural elements.

4. Which explanation regarding cardiac catheterization is appropriate for a preschool child? a. Postural drainage will be performed every 4 to 6 hours after the test. b. It is necessary to be completely "asleep" during the test. c. The test is short, usually taking less than 1 hour. d. When the procedure is done, you will have to keep your leg straight for at least 4 hours.

d. When the procedure is done, you will have to keep your leg straight for at least 4 hours. ANS: D The child's leg will have to be maintained in a straight position for approximately 4 hours. Younger children can be held in the parent's lap with the leg maintained in the correct position. Postural drainage will not be performed unless the child has corresponding pulmonary problems. The child should be sedated to lie still, but being completely asleep is not necessary. The test will vary in length of time from start to finish.

31. A family wants to begin oral feeding of their 4-year-old son, who is ventilator- dependent and currently tube-fed. They ask the home health nurse to feed him the baby food orally. The nurse recognizes a high risk of aspiration and an already compromised respiratory status. The most appropriate nursing action is to: a. refuse to feed him orally because the risk is too high. b. explain the risks involved, and then let the family decide what should be done. c. feed him orally because the family has the right to make this decision for their child. d. acknowledge their request, explain the risks, and explore with the family the available options.

d. acknowledge their request, explain the risks, and explore with the family the available options. ANS: D Parents want to be included in the decision making for their child's care. The nurse should discuss the request with the family to ensure this is the issue of concern, and then they can explore potential options together. Merely refusing to feed the child orally does not determine why the parents wish the oral feedings to begin and does not involve them in the problem solving. The decision to begin or not change feedings should be a collaborative one, made in consultation with the family, nurse, and appropriate member of the health care team. DIF: Cognitive Level: Analyze REF: p. 823 TOP: Integrated Process: Teaching/Learning MSC: Area of Client Needs: Safe and Effective Care Environment: Management of Care

37. Children taking phenobarbital (phenobarbital sodium) and/or phenytoin (Dilantin) may experience a deficiency of: a. calcium. b. vitamin C. c. fat-soluble vitamins. d. vitamin D and folic acid.

d. vitamin D and folic acid. ANS: D Deficiencies of vitamin D and folic acid have been reported in children taking phenobarbital and phenytoin. Calcium, vitamin C, and fat-soluble vitamin deficiencies are not associated with phenobarbital or phenytoin. DIF: Cognitive Level: Understand REF: p. 905 TOP: Integrated Process: Nursing Process: Implementation MSC: Area of Client Needs: Physiologic Integrity: Pharmacologic and Parenteral Therapies


Conjuntos de estudio relacionados

8 The individual life insurance contract

View Set

PrepU #1.1 - Anatomy and Physiology of Reproductive System

View Set

MD Accident, Health & Life Producer Exam A

View Set

Study Stack Muscle Groups 4 Head and Neck

View Set